Revision (2023 24) 2

You might also like

Download as pdf or txt
Download as pdf or txt
You are on page 1of 63

CBSE Class 12th Board Exams Feb- Mar- 2024

CBSE Class 12th Board Exams are approaching soon and will commence in Feb-March 2024.
Mathematics is a crucial subject and students keen on getting full marks must understand the
marking scheme for Mathematics to give their best shot within the stipulated duration of 3 hours.
CBSE has also uploaded the Sample Question Paper for Mathematics and candidates gearing up
for Class 12th Board Exams must practice this paper among others to understand the level of
difficulty and prepare accordingly.
CBSE Class 12th Board Mathematics Exam Pattern:
1. The CBSE Class 12th Board Mathematics exam will comprise of 36 questions viz
Ques. : 1-20 will carry 1 mark each (20 Marks)
Ques. : 21-25 will carry 2 mark each (10 Marks)
Ques. : 26-31 will carry 3 mark each (18 Marks)
Ques. : 32-35 will carry 5 mark each (20 Marks)
Ques. : 36-38 will carry 4 mark each (12 Marks)(Case base Study)
2. Candidates will be given internal choices in the long answer type questions in Section B, C & D.
3. Marking scheme will be followed for each step given by the candidate and the conclusion statement
and figure will carry a mark too.
Unit-wise allocation of marks of CBSE Class 12 Mathematics:

MATHEMATICS BLUE PRINT-I


SESSION-2023-24
There will be no overall choice in the question paper.
However, 33% internal choices will be given.
Internal Assessement:- 20 Marks

Assessment of Activity Work:


Throughout the year any 10 activities shall be performed by the student from the
activities given in the NCERT Laboratory Manual for the respective class ( XI or XII)
which is available on the link : http://www.ncert.nic.in/exemplar/labmanuals.html A
record of the same may be kept by the student. An year end test on the activity may be
conducted at the School Level.
The weight age are as under:
(a) The activities performed by the student through out the year and record keeping : 5 marks
(b) Assessment of the activity performed during the year end test: 3 marks
(c) Viva-voce : 2 marks
CBSE Class 12 Maths Exam 2020: 7 topics which can fetch you 90 percent
Topics you have to strongly revise to score 90 per cent above:
1.Continuity & Differentiability 2.Three Dimensional Geometry 3.Probability (Bayes Theorem)
4.Linear Programming 5.Matrices and determinants 6.Application of Derivatives (Word Problem)
7.Differential equations
Deleted portion for Class XII (2023-24)(OLD BOOK)
Chapter-1 (Relations and Functions)
Example No.- 15, 16, 17, 18, 19, 20,21, 22, 23, 24, 25, 26, 27, 28.
Exercise:1.3 : - 1, 2, 3, 4, 5, 6(2nd part), 7(2nd part), 8(2nd part), 9, 10, 11, 12, 13, 14.
Example No. – 29 to 40
Exercise 1.4 :- Full Deleted
Example No. :- 45, 49
Miscellaneous Exercise:- 1, 2, 3, 6, 7, 9, 11, 12, 13, 14,19.
Chapter-2 (Inverse Trigonometric Functions)
Example No. – 3, 4, 5, 6, 7
Exercise 2.2:- 1,2,3,4,5,6,7,8,9,10,11,12,13,14,15,18.
Example No. – 10, 11, 12, 13.
Miscellaneous Exercise:- 3,4,5,6,7,8,9,10,11,12,13, 14, 15, 16,17.
Chapter-3 (Matrices)
Example No. – 23, 24 25.
Exercise :- 3.4 :- 1 to 17
Chapter-4 (Determinants)
Example No. – 6,7,8,9,10,11,12,13,14,15, 16.
Exercise 4.2:- 1 to 14
Example No. – 30,31,34.
Miscellaneous Exercises:- 1, 2, 3, 4, 5, 6, 9, 10, 11, 12, 13, 14, 15, 17.
Chapter-5 (Continuity and Differentiability)
Exercise 5.1 : -31, 32, 33.
Example No.- 42, 43.
Exercise 5.8:- Full Deleted
Miscellaneous Exercise: - 19
Chapter-6 (Application of Derivatives)
Example No.- 14, 15, 16, 17, 18, 19, 20, 45 & 46
Exercise 6.3:- Full Deleted
Example No. -21, 22, 23, 24 & 25
Exercise 6.4:- Full Deleted
Miscellaneous Exercise:- 1, 4, 5, 20, 21, 22 & 23
Chapter-7 (Integrals)
Example No:- 25, 26.
Exercise 7.8 :- Full deleted
Miscellaneous Exercise:- 40.
Supplementary Exercise.
Chapter-8 (Application of Integrals)
Example No.:- 7, 8, 10.
Exercise :- 8.2 :- 1,2.
Example No.:- 13, 14.
Miscellaneous Exercise:- 12,15,18,19.
Chapter-9 (Differential Equations)
Example No.:- 4, 5, 6, 7, 8.
Exercise 9.3 :- Full deleted
Miscellaneous Exercise:- 3 & 5
Chapter-10 (Vector Algebra)
Supplementary exercise.
Chapter-11 (Three Dimentional Geometry)
Example No.:- 13, 14, 15, 16, 17, 18, 19, 20, 21, 22, 23, 24, 25 , 26, 27, 28 & 29
Exercise 11.3 :- Full Deleted
Miscellaneous Exercise:- 7, 8, 10, 11, 12, 13, 14, 15, 16, 17, 18, 19, 21, 22 and 23.
Chapter-12 (Linear Programming)
Example No.:- 6, 7, 8.
Exercise 12.2 :- Full deleted
Example No.:- 9, 10, 11.
Miscellaneous Exercise:- Full deleted.
Chapter-13 (Probability)
nd nd
Example No.:- 27(2 part), 28, 29(2 part).
Exercise 13.4 :- 10, 11, 12, 13, 14, 15(2nd part of all question), 16, 17.
Example No.:- 31, 32, 34, 35.
Exercise 13.5:- Full deleted.
Miscellaneous Exercise:- 4, 5, 6, 7, 9, 10, 11.
REVISION PAPER FOR CBSE BOARD EXAMINATION
RELATIONS AND FUNCTIONS
1. Let T be the set of all triangles in a plane with R a relation in T given by R = {(T1, T2) : T1 is congruent to T2}.
Show that R is an equivalence relation.
2. Let L be the set of all lines in a plane and R be the relation in L defined as R = {(L1, L2) : L1 is perpendicular to L2}.
Show that R is symmetric but neither reflexive nor transitive.
3. Determine whether this relations are reflexive, symmetric and transitive R in the set A = {1, 2, 3, 4, 5, 6} as
R = {(x, y) : y is divisible by x}
4. Check whether the relation R defined in the set {1, 2, 3, 4, 5, 6} as R = {(a, b) : b = a + 1} is reflexive, symmetric or transitive.
5. Check whether the relation R in R defined by R = {(a, b) : a ≤ b2} is reflexive, symmetric or transitive.
6. Show that the relation S in the set R of real numbers defined as S={(a,b): a,b ∈ R and a≤ 𝑏 3 } is neither reflexive nor
symmetric nor transitive.
7. Let Z be the set of all integers and R is the relation on Z defined by as
R= {(a,b):a,b∈ Z and a-b is divisible by 5}.Prove that R is equivalence relation.
8. Show that the relation R in the set A={x:x∈ Z,0≤ 𝑥 ≤ 12}given by R={(a,b):|𝑎 − 𝑏| is divisible by 4} is an
equivalence relation. Find the set of all elements related to 1.
9. Let N denotes the set of all natural numbers and R be the relation on N X N defined by
(a,b)R(c,d), if ad(b+c) =bc(a+d). Show that R is an equivalence relation.
10. Show that the relation R defined in the set A of all triangles as R = {(T1, T2) : T1 is similar to T2}, is equivalence
relation. Consider three right angle triangles T1 with sides 3, 4, 5, T2 with sides 5, 12, 13 and T3 with sides 6, 8, 10.
Which triangles among T1, T2 and T3 are related?
11. Let L be the set of all lines in XY plane and R be the relation in L defined as R = {(L1, L2) : L1 is parallel to L2}.
Show that R is an equivalence relation. Find the set of all lines related to the line y = 2x + 4.
12. Show that the function f : N → N, given by f (x) = 2x, is one-one but not onto.
13. Prove that the function f : R → R, given by f (x) = 2x, is one-one and onto.
14. Show that the function f : R → R, defined as f (x) = x2, is neither one-one nor onto.
15. Prove that the Greatest Integer Function f : R → R, given by f (x) = [x], is neither one-one nor onto, where [x] denotes the
greatest integer less than or equal to x.
𝑥−2
16. Let A=R-{3} and B=R-{1}.Consider the function f : A→B defined by f(x)= . Show that f is one-one and onto.
𝑥−3
2 4𝑥+3
17. Show that the function f in A=R - { } defined as f(x)= is one-one and onto
3 6𝑥−4
18. Consider f:R→ [4, ∞)given by f(x)=x2+4. Show that f is one-one and onto.
19. If f: R→R be the function defined by f(x)=4x3+7, Show that f is a bijection.
20. Consider f : R → R given by f (x) = 4x + 3. Show that f is invertible. Find the inverse of f.
𝑥 + 1 𝑖𝑓 𝑥 𝑖𝑠 𝑜𝑑𝑑
21. Show that f:N→N given by f(x)={ is bijective (both one-one and onto).
𝑥 − 1 𝑖𝑓 𝑥 𝑖𝑠 𝑒𝑣𝑒𝑛
𝑛+1
, 𝑖𝑓 𝑛 𝑖𝑠 𝑜𝑑𝑑
22. Let f:N→N be defined by f(n)={ 𝑛2 for all n∈N.Find whether the function f is bijective..
, 𝑖𝑓 𝑛 𝑖𝑠 𝑒𝑣𝑒𝑛
2
Choose the correct answer.
23. Let f : R → R be defined as f(x) = x4. Choose the correct answer.
(A) f is one-one onto (B) f is many-one onto (C) f is one-one but not onto (D) f is neither one-one nor onto.
24. Let f : R → R be defined as f (x) = 3x. Choose the correct answer.
(A) f is one-one onto (B) f is many-one onto (C) f is one-one but not onto (D) f is neither one-one nor onto.
25. Let A = {1, 2, 3}. Then number of relations containing (1, 2) and (1, 3) which are reflexive and symmetric but not
transitive is
(A) 1 (B) 2 (C) 3 (D) 4
26. Let A = {1, 2, 3}. Then number of equivalence relations containing (1, 2) is
(A) 1 (B) 2 (C) 3 (D) 4
27. Let R be the relation in the set {1, 2, 3, 4} given by R = {(1, 2), (2, 2), (1, 1), (4,4), (1, 3), (3, 3), (3, 2)}.
(A) R is reflexive and symmetric but not transitive. (B) R is reflexive and transitive but not symmetric.
(C) R is symmetric and transitive but not reflexive. (D) R is an equivalence relation.
28. Let R be the relation in the set N given by R = {(a, b) : a = b – 2, b > 6}. Choose the correct answer.
(A) (2, 4) ∈ R (B) (3, 8) ∈ R (C) (6, 8) ∈ R (D) (8, 7) ∈ R
INVERSE TRIGONOMETRIC FUNCTIONS
−1 11𝜋
1. Write the principal value of tan-1( 1 ) + cos-1( )(Ans: )
2 12
−1 1
2. Write the principal value of cos-1( ) + 2sin-1( ). (Ans: 𝜋 )
2 2
3𝜋 −𝜋
3. Write the value of tan-1(tan )(Ans: )
4 4
2𝜋
4. Write the value of Sin-1(Sin ).
4
3𝜋
5. Write the value of Sin-1(Sin )
5
13𝜋
6. Write the value of Cos-1(Cos )
6
7𝜋
7. Write the value of tan-1(tan )
6
7𝜋
8. Write the value of Cos-1(Cos )
6
𝜋 −1 1
9. Find the Value of Sin( - 𝑆𝑖𝑛 (− )).
3 2
7𝜋 5𝜋
10.Write the value of cos-1(cos )(Ans: ).
6 6
11.Find the principal value of the following:
1 1 1 2
(a)𝑆𝑖𝑛−1 (− ) (b) 𝑡𝑎𝑛−1 (− √3 ) (c) 𝐶𝑜𝑠 −1 (− ) (d) 𝐶𝑜𝑠 −1 (− )(e) 𝑆𝑒𝑐 −1 ( )
2 2 √2 √3
1 1
12.Find the value of 𝑡𝑎𝑛−1 (1 ) + 𝐶𝑜𝑠 −1 (− ) + 𝑆𝑖𝑛 −1
(− ).
2 2
1 1
13.Find the value of 𝑐𝑜𝑠 −1 ( ) + 3𝑆𝑖𝑛−1 ( ).
2 2
−1 1
14.Prove that sin-1(2x √1 − 𝑥2 ) = 2sin-1x , where ≤x≤ .
√2 √2
1
15.Prove that sin-1(2x √1 − 𝑥 2 ) = 2cos-1x , where ≤ x ≤ 1.
√2
𝑐𝑜𝑠𝑥 −3𝜋 𝜋
16.Express tan-1 ( ), < x < in the simplest form.
1−𝑠𝑖𝑛𝑥 2 2
1
17.Find the value of tan-1[2cos(2sin-1 ) ].
2
Choose the correct answer.
𝝅 𝟏
18. Sin( 𝟑 – sin-1(- 𝟐 ) ) is equal to:
𝟏 𝟏 𝟏
(a)𝟐 (b)𝟑 (c)𝟒 (d)1
19. If sin-1x = y, then
−𝝅 𝝅 −𝝅 𝝅
(a)0≤y≤ 𝝅 (b) ≤y≤ (c) 0<y< 𝝅 (d) <y<
𝟐 𝟐 𝟐 𝟐
15. Tan-1(√𝟑 )– sec-1(-2) is equal to
−𝝅 𝝅 𝟐𝝅
(a)𝝅 (b) 𝟑 (c) 𝟑 (d) 𝟑
16. Tan-1 √𝟑 – cot-1(- √𝟑 ) is equal to
−𝝅
(a)𝝅 (b) 𝟐 (c)0 (d)2√𝟑
17. Sin(tan-1x) , |𝒙| < 1 is equal to
𝒙 𝟏 𝟏 𝒙
(a) (b) (c) (d)
√𝟏−𝒙𝟐 √𝟏−𝒙𝟐 √𝟏+𝒙𝟐 √𝟏+𝒙𝟐
𝝅
18. Sin-1(1-x) - 2 sin-1x = 𝟐 , then x is equal to:
𝟏 𝟏 𝟏
(a)0, 𝟐 (b)1, 𝟐 (c)0 (d)𝟐
MATRICES
2 0 1 1 −1 −1
1. If A= [2 1 3] ,Find the value of A2-3A+2I. (Ans :- A =[ 3 −3 −4] ).
1 −1 0 −3 2 0
1 0 2
2. If A= [0 2 1] , Prove that A3 – 6A2 + 7A + 2I =0.
2 0 3
𝑥 + 3 𝑧 + 4 2𝑦 − 7 0 6 3𝑦 − 2
3. If [ −6 𝑎 − 1 0 ] = [ −6 −3 2𝑐 + 2 ] .(Ans:- a = – 2, b = – 7, c = – 1, x = – 3, y = –5, z = 2).
𝑏 − 3 −21 0 2𝑏 + 4 −21 0
4. Find the values of a, b, c, and d from the following equation:
2𝑎 + 𝑏 𝑎 − 2𝑏 4 −3
[ ] =[ ] (Ans:- a = 1, b = 2, c = 3 and d = 4)
5𝑐 − 𝑑 4𝑐 + 3𝑑 11 24
5. Find the value of a, b, c and d from the equation:
𝑎 − 𝑏 2𝑎 + 𝑐 −1 5
[ ] =[ ].
2𝑎 − 𝑏 3𝑐 + 𝑑 0 13
5 2 3 6 4 4 1 −2
6. Find the value of x and y if x + y = [ ] and x – y = [ ].Ans:- x = [ ] and y = [ ])
0 9 0 −1 0 4 0 5
1 3 5
7. Let A=[−6 8 3].Express A as sum of two matrices such that one is symmetric and other is skew symmetric.
−4 6 5
2 −2 −4
8. Let A=[−1 3 4 ].Express A as sum of two matrices such that one is symmetric and other is skew symmetric.
1 −2 −3
9. If A and B are symmetric matrices, prove that AB – BA is a skew symmetric matrix.
10. Show that the matrix B′AB is symmetric or skew symmetric according as A is symmetric or skew symmetric.
(𝑖+2𝑗)2
11. Construct a 2 × 2 matrix, A = [aij], whose elements are given by: aij = .
2
1
12. Construct a 3 × 4 matrix, whose elements are given by: aij = 2 |−3𝑖 + 𝑗|.
13. A school wants to award its students for the values of honesty, regularity and hard work with a total cash award of Rs
6000.Three times the award money for hard work added to that given for honesty amounts to Rs 11000.The award
money given for honesty and hard work together is double the one given for regularity. Represent the above situation
algebraically and find the award money for each value, using matrix method. Apart from these values, namely
honesty, regularity and hard work suggest one more value which the school must include for award.
(Ans: Rs 500,Rs 2000 and Rs 3500).
14. The management committee of a residential colony decided to award some of its members (say x) for honesty, some
(say y) for helping others and some others (say z) for supervising the workers to keep the colony neat and clean. The
sum of all the awardees is 12. Three times the sum of awardees for cooperation and supervision added to two times
the number of awardees for honesty is 33.If the sum of the number of awardees for honesty and supervision is twice
the number of awardees for helping others,using matrix method ,find the number of awardees of each category. Apart
from these values namely honesty, cooperation and supervision suggest one more value which the management of the
colony must include for awards. (Ans:3,4,5)
15. Two schools P and Q want to award their selected students on the values of discipline, politeness and punctuality.
The school P wants to award Rs x each, Rs y each and Rs z each for the three respective values to its 3,2 and 1
students with a total award money of Rs1000.School Q wants to spend Rs 15oo to award its 4,1
and 3 students on the respective values. If the total amount of award for one prize on each value is Rs 600, using
matrices, find the award money for each value. (Ans:100,200,300)
16. The cost of 4 Kg onion,3kg wheat and 2kg rice is Rs 60. The cost of 2 kg onion,4kg wheat and 6kg rice is Rs 90. The
cost of 6kg onion,2kg wheat and 3kg rice is Rs 70.Find the cost of each per kg by matrix method.(Ans:Rs5,Rs8,Rs 8).
17. The sum of three numbers is 6. If we multiply third number by 3 and add second number to it, we get 11. By adding
first and third numbers, we get double of the second number. Represent algebraically and find the numbers using
matrix method. (Ans 1,2,3).
𝛼
0 −𝑡𝑎𝑛 2 𝑐𝑜𝑠𝛼 −𝑠𝑖𝑛𝛼
18. Let A=[ 𝛼 ] and I the identity matrix of order 2X2 show that I+A=(I-A)[ ]..
𝑡𝑎𝑛 0 𝑠𝑖𝑛𝛼 𝑐𝑜𝑠𝛼
2
2 −1 5 2 2 5 −191 −110
19. Let A = [ ] , B=[ ] ,C= [ ]. Find a matrix D such that CD-AB=0.(Ans: [ ])
3 4 7 4 3 8 77 44
2 −1 −1 −8 −10
1 −2 −5
20. If [ 1 0 ] A = [ 1 −2 −5 ], Find A. (Ans: A= [ ])
3 4 0
−3 4 9 22 15
1 2 3 −7 −8 −9 1 −2
21. Find the matrix A such that A [ ]=[ ].(Ans: A = [ ])
4 5 6 2 4 6 2 0
1 1 3 3 5 2 3 4
22. Find the matrix A such that [ ]A=[ ].(Ans: A= [ ]).
0 1 1 0 1 1 0 1
1 2 0 0
23. For what values of x: [1 2 1] [2 0 1] [2] = O. (Ans:- -1)
1 0 2 𝑥
1 0 2 𝑥
24. For what values of x: [𝑥 −5 −1] [0 2 1] [4] = O ( Ans: ±4√3 )
2 0 3 1
Choose the correct answer
25. A = [𝒂𝒊𝒋 ]mxn is a square matrix , if
(a) m< n (b)m>n (c)m=n (d)None of these
26. Which of the given values of x and y make the following pair of matrices equal
𝟑𝒙 + 𝟕 𝟓 𝟎 𝒚−𝟐
[ ]=[ ]
𝒚 + 𝟏 𝟐 − 𝟑𝒙 𝟖 𝟒
−𝟏 −𝟐 −𝟏 −𝟐
(a)x= 𝟑 , y=7 (b)Not possible to find (c) y= 7 , x= 𝟑 (d) x= 𝟑 , y = 𝟑 .
27. The number of all possible matrices of order 3 × 3 with each entry 0 or 1 is:
(a) 27 (b) 18 (c) 81 (d) 512
Assume X, Y, Z, W and P are matrices of order 2 × n, 3 × k, 2 × p, n × 3 and p × k, respectively.
Choose the correct answer.
28. The restriction on n, k and p so that PY + WY will be defined are:
(a) k = 3, p = n (b) k is arbitrary, p = 2 (c) p is arbitrary, k = 3 (d) k = 2, p = 3
29. If n = p, then the order of the matrix 7X – 5Z is:
(a) p × 2 (b) 2 × n (c) n × 3 (d) p × n.
30. If A, B are symmetric matrices of same order, then AB – BA is a
(a) Skew symmetric matrix (b) Symmetric matrix (c) Zero matrix (d) Identity matrix
𝒄𝒐𝒔𝜶 −𝒔𝒊𝒏𝜶
29. If A = [ ] and A + A’ = I then the value of 𝜶 is
𝒔𝒊𝒏𝜶 𝒄𝒐𝒔𝜶
𝝅 𝝅 𝟑𝝅
(a) (b) (c)𝝅 (d)
𝟔 𝟑 𝟐
30. Matrices A and B will be inverse of each other only if
(a) AB = BA (b) AB = BA = 0 (c) AB = 0, BA = I (d) AB = BA = I
𝜶 𝜷
31. If A = [ ] is such that A2 = I, then
𝜸 −𝜶
(a)1 + 𝜶𝟐 + 𝜷𝜸 = 0 (b) 1 - 𝜶𝟐 + 𝜷𝜸 = 0 (c) 1 - 𝜶𝟐 - 𝜷𝜸 = 0 (d) 1 + 𝜶𝟐 - 𝜷𝜸 = 0]
32. If the matrix A is both symmetric and skew symmetric, then
(a) A is a diagonal matrix (b) A is a zero matrix (c) A is a square matrix (d) None of
these
33. If A is square matrix such that A2 = A, then (I + A)3 – 7 A is equal to
(a) A (b) I – A (c) I (d) 3A
DERTEMINANTS
2 −3 5
1. Find the minors and cofactors of the elements of the determinant |6 0 4 | and
1 5 −7
Verify that a11A31 + a12A31 + a13A33 = 0
2. Find the area of the triangle whose vertices are (3, 8), (– 4, 2) and (5, 1). (Ans:- 61/2)
3. Find values of k if area of triangle is 4 sq. units and vertices are
(i) (k, 0), (4, 0), (0, 2) (ii) (–2, 0), (0, 4), (0, k)
𝑥 + 1 𝑥 − 1 4 −1
4. if | |=| | ,then write the value of x.(Ans:2)
𝑥−3 𝑥+2 1 3
2 3 1 −2
5. If A = [ ] and B = [ ] then verify that (AB)-1 = B-1. A-1 .
1 −4 −1 3
2 3
6. Show that the matrix A = [ ] , satisfies the equation A2 – 4A + I = O, where I is 2 × 2 identity matrix and O is 2 ×
1 2
2 zero matrix. Using this equation, find A-1.
3 1
7. Show that the matrix A = [ ] , satisfies the equation A2 – 5A + 7I = O, where I is 2 × 2 identity matrix and O is
1 2
2 × 2 zero matrix. Using this equation, find A-1.
2(𝑥 + 1) 2𝑥
8. For what value of x, A=[ ] is a singular matrix?(Ans:-2)
𝑥 𝑥−2
2𝑥 + 4 4
9. For what value of x, A=[ ] is a singular matrix?(Ans:4)
𝑥+5 3
1 −1 0 2 2 −4
10. Given that A=[2 3 4] and B=[−4 2 −4].find AB. Use this to solve the following system of equations
0 1 2 2 −1 5
x-y=3, 2x+3y+4z=17, y+2z=7.(Ans:2,-1,4)
0 1 2
-1
11. Compute A for the matrix A= [1 2 3] .Hence solve the system of equations y+2z+8=0, x+2y+3z+14=0 and
3 1 1
3x+y+z+8=0. (Ans:-1,-2,-3)
12. Using matrix method solve the following system of equations:
2 3 10 4 6 5 6 9 20
+𝑦+ = 4, 𝑥 − 𝑦 + 𝑧 = 1., 𝑥 + 𝑦 − = 2.(Ans;2,3,5).
𝑥 𝑧 𝑧
13. Using matrices, solve the following system of linear equations
x+2y-3z=-4, 2x+3y+2z=2, 3x-3y-4z=11. (Ans:3, -2, 1)

Choose the correct answer


𝒙 𝟐 𝟔 𝟐
14. If | |=| | then x is equal to
𝟏𝟖 𝒙 𝟏𝟖 𝟔
(a)6 (b)±6 (c)-6 (d)0
15. If area of triangle is 35 sq units with vertices (2, – 6), (5, 4) and (k, 4). Then k is
(a) 12 (b) –2 (c) –12, –2 (d) 12, –2
16. Let A be a non singular square matrix of order 3 × 3. Then | adj A| is equal to
(a) | A| (b) |𝑨|𝟐 (c) |𝑨|𝟑 (d) 3|A|
17. If A is an invertible matrix of order 2, then det(A-1) is equal to
𝟏
(a)det(A) (b)𝐝𝐞𝐭 (𝑨) (c)1 (d)0
𝟏 𝒔𝒊𝒏𝜽 𝟏
18. Let A = [−𝒔𝒊𝒏𝜽 𝟏 𝒔𝒊𝒏𝜽], where 0 ≤ 𝜽 ≤ 2𝝅. Then
−𝟏 −𝒔𝒊𝒏𝜽 𝟏
(a)Det(A) = 0 (b)Det(A) ∈ (2, ∞) (c)Det(A) ∈ (2, 4) (d) Det(A) ∈ [2, 4]
CONTINUITY AND DIFFERENTIABILITY
√1+𝑝𝑥−√1−𝑝𝑥
∶ −1 ≤ 𝑥 < 0
1. Find p so that f(x)= { 𝑥 is continuous at x = 0. (Ans:p=-1/2)
2𝑥+1
∶0≤𝑥≤1
𝑥−2
𝑘𝑐𝑜𝑠𝑥 𝜋
𝐼𝑓 𝑥 ≠
2. f(x) = is continuous at x = 0, If f(x) = { 𝜋−2𝑥 2
𝜋 is continuous at x = π / 2.(Ans:k=6)
3 𝐼𝑓 𝑥 = 2
1−𝑐𝑜𝑠4𝑥
,𝑥 < 0
𝑥2
3. Let f(x)= 𝑎 ,𝑥 = 0 for what value of a f is continuous at x=0.(Ans:a=8).
√𝑥
,𝑥 > 0
{√16+√𝑥−4
𝜋
𝑎𝑠𝑖𝑛 2 (𝑥 + 1), 𝑥 ≤ 0 1
4. Find the value of a for which the function f is defined as f(x)={ 𝑡𝑎𝑛𝑥−𝑠𝑖𝑛𝑥 is continuous at x=0.(Ans:a=𝜋)
, 𝑥>0
𝑥3
3𝑎𝑥 + 𝑏 𝑖𝑓 𝑥 > 1
5. If the function f(x) ={ 11 𝑖𝑓 𝑥 = 1 is continuous at x=1.Find the values of a and b.(Ans:a=3,b=2).
5𝑎𝑥 − 2𝑏 𝑖𝑓 𝑥 < 1
5 𝑖𝑓 𝑥 ≤ 2
6. If the function f(x) ={𝑎𝑥 + 𝑏 𝑖𝑓 2 < 𝑥 < 10 is continuous .Find the values of a and b.(Ans:a=2,b=1).
21 𝑖𝑓 𝑥 ≥ 10
2𝑥 − 1 , 𝑖𝑓 𝑥 < 0
7. Discuss the continuity of f(x) at x=0 if f(x)={ .
2𝑥 + 1, 𝑖𝑓 𝑥 ≥ 0.
𝑆𝑖𝑛(𝑎+1)𝑥+𝑆𝑖𝑛𝑥
𝑥<0
𝑥
𝑐 𝑥=0 −3 1
8. Find a and b so that f(x) = is continuous at x = 0.(Ans:a= 2 ,c = 2 ,b=any real number.)
√𝑥+𝑏𝑥 2 − √𝑥
3 𝑥>0
{ 𝑏(𝑥)2
1−𝑠𝑖𝑛3 𝑥 𝜋
𝑖𝑥 𝑥 <
3𝑐𝑜𝑠2 𝑥 2
9. Let f(x) 𝑎 𝑖𝑓 𝑥 = 𝜋/2 .if f(x)is continuous at x=𝜋/2 find a and b.
𝑏(1−𝑠𝑖𝑛𝑥) 𝜋
𝑖𝑓 𝑥 >
{ (𝜋−2𝑥)2 2
|𝑥−4|
10. Show that function is defined is everywhere continuous except at x = 4 f(x)={ 𝑥−4 ∶ 𝑥 ≠ 4.
0 ∶𝑥=4
𝑥−5
|𝑥−5⌋
+𝑎 𝑖𝑓𝑥 < 5
11. If f(x)= 𝑎 + 𝑏 𝑖𝑓 𝑥 = 5 is continuous function find a and b.
𝑥−5
{ |𝑥−5⌋
+ 𝑏 𝑖𝑓 𝑥 > 5
𝑐𝑜𝑠2 𝑥−𝑠𝑖𝑛2 𝑥−1
12. Find k so that f(x)={ √𝑥 2 +1−1
𝑥 ≠ 0 is continuous at x = 0.
𝑘 𝑥=0
2𝑥 + 3 𝑖𝑓 𝑥 ≤ 2
13. Find all points of discontinuity of f, where f is defined by f(x) = { .
2𝑥 − 3 𝑖𝑓 𝑥 > 2
𝑎𝑥 + 1 𝑖𝑓 𝑥 ≤ 0
14. Find the relationship between a and b so that the function f defined by f(x) = { continuous at x = 0?
𝑏𝑥 + 3 𝑖𝑓 𝑥 > 0
What about continuity at x = 1?
15. Prove that the function f given by f(x) = |𝑥 − 1| , x ∈ 𝑅 is not differentiable at x =1.
16. Prove that the greatest integer function defined by f(x) = [x] , 0 < x < 3 is not differentiable at x = 1 and x = 2.
DIFFERENTIABILITY
𝑑𝑦 𝑑𝑦 −(2𝑥+𝑦)
1. Find in the following: x2 + xy + y2 = 100.(Ans: = )
𝑑𝑥 𝑑𝑥 𝑥+2𝑦
𝑑𝑦 𝑑𝑦 𝑠𝑖𝑛2𝑥
2. Find in the following: sin2x + cos2y = 1. (Ans: = )
𝑑𝑥 𝑑𝑥 𝑠𝑖𝑛2𝑦
𝑑𝑦 𝑑𝑦 −𝑎
3. Find in the following: ax + by2 = cosy. (Ans: = )
𝑑𝑥 𝑑𝑥 2𝑏𝑦+ 𝑠𝑖𝑛𝑦
𝑑𝑦 𝑐𝑜𝑠 2 (𝑎+𝑦)
4. If cosy = x cos(a+y), with cosa ≠ ±1 prove that = .
𝑑𝑥 𝑠𝑖𝑛𝑎
𝑑𝑦 𝑠𝑒𝑐 2 𝑥−𝑦
5. Find the derivative of the following : xy+y2 = tanx+y.(Ans: = ).
𝑑𝑥 𝑥+2𝑦−1
𝑑𝑦 sinx 𝑠𝑖𝑛𝑥
6. Find for the function y = xsinx , x > 0 w.r.t x. (Ans:- x [ + cosx logx]
𝑑𝑥 𝑥
1 𝑙𝑜𝑔𝑥
7. Differentiate the function w.r.t x (logx)x + xlogx.(Ans:(logx)x{ +log(logx)}+2( )xlogx).
𝑙𝑜𝑔𝑥 𝑥
𝑑𝑦 𝑙𝑜𝑔𝑠𝑖𝑛𝑦+𝑦𝑡𝑎𝑛𝑥
8. If (cosx)y=(siny)x,find the value of .(Ans: ).
𝑑𝑥 𝑙𝑜𝑔𝑐𝑜𝑠𝑥−𝑥𝑐𝑜𝑡𝑦
𝑑𝑦 (1+𝑙𝑜𝑔𝑦)2
9. If yx = ey-x , prove that = .
𝑑𝑥 𝑙𝑜𝑔𝑦
𝑑𝑦
10.If y = cos x . cos 2x . cos 3x, find (Ans: cos x . cos 2x . cos 3x [-tanx -2tan2x- 3tan3x]
𝑑𝑥
𝑥 2 +1 𝑑𝑦 4𝑥
11.If y=(x)xcosx + , find .(Ans: =(x)xcosx (cosx –x logx sinx+logxcosx) - ).
𝑥 2 −1 𝑑𝑥 (𝑥 2 −1)2
1 1
𝑑𝑦 x x 𝑐𝑜𝑡𝑥 log (𝑠𝑖𝑛𝑥)
12.Find , if y= (cosx) + (sinx) (Ans:cosx (-xtanx+logcosx)+ (sinx) (
𝑥 𝑥 − ).
𝑑𝑥 𝑥 𝑥2
𝑥 2 +1 𝑑𝑦 𝑐𝑜𝑠𝑥 4𝑥
13.If y=(logx)cosx + ,find .(Ans: (logx)cosx ( –sinxlog(logx)) - ).
𝑥 2 −1 𝑑𝑥 𝑥𝑙𝑜𝑔𝑥 (𝑥 2 −1)2
𝑑𝑦 −[𝑥 𝑦−1 +𝑦 𝑥 𝑙𝑜𝑔𝑦
14.If xy + yx = ab, find (Ans: ).
𝑑𝑥 𝑥 𝑦 𝑙𝑜𝑔𝑥+𝑦 𝑥−1 𝑥
𝑑2𝑦 𝑠𝑒𝑐 4 𝜃𝑐𝑜𝑠𝑒𝑐𝜃
15.If x=acos3𝜃 ; y=asin3𝜃 ,find .(Ans: ).
𝑑𝑥 2 3𝑎
𝑑𝑦 −𝜋 𝜋 6 𝑡
16.Find , if y = 12(1 – cost), x= 10( t – sint ) , < t< (Ans:- cot )
𝑑𝑥 2 2 5 2
𝜃 𝑑𝑦 𝜋
17.If x=a(cos𝜃 +logtan ) and y=asin𝜃 ,find the value of at 𝜃= .(Ans:1)
2 𝑑𝑥 4
𝑑2 𝑦 𝑏𝑠𝑒𝑐 3 𝑡
18.If x = a(cost + tsint) and y = b(sint - tcost),find [Ans: ].
𝑑𝑥 2 𝑎2 𝑡
𝑑2𝑦 −1 𝜃
19.If x=a(𝜃 –sin𝜃) and y=a(1-cos𝜃),find . (Ans: cosec4 ).
𝑑𝑥 2 4𝑎 2
−1 𝑡 −1 𝑡 𝑑𝑦 −𝑦
20.If x=√𝑎 𝑠𝑖𝑛 , y = √𝑎𝑐𝑜𝑠 , show that = .
𝑑𝑥 𝑥
2 1+𝑥 2
-1 1−𝑥 𝑑𝑢
21.Differentiate sin-1(2x√1 − 𝑥 2 ) w.r.t cos ( 2
).(Ans: = ).
1+𝑥 𝑑𝑣 √1−𝑥 2
𝑑𝑦 2𝑧 1−𝑧 2 1
22.Find when y=√𝑠𝑖𝑛−1 ( 2
) and x= cos-1( ).(Ans: ).
𝑑𝑥 1+𝑧 1+𝑧 2 2𝑦
−2𝑐𝑜𝑠𝑥
23.Differentiate sin2x w.r.t ecosx .(Ans: )
𝑒 𝑐𝑜𝑠𝑥
√1+𝑥 2 −√1−𝑥 2 −1
24.Differentiate tan-1[ ] with respect to cos-1x2.(Ans: ).
√1+𝑥 2 +√1−𝑥 2 2
𝑑2𝑦 𝑑𝑥
25.If y=log(x+√𝑥 2 + 𝑎2 ), show that(x2+a2) +x =0.
𝑑𝑥 2 𝑑𝑦
𝑑2𝑦 𝑑𝑦
26.If y = (tan-1x)2 , show that (x2+1)2 + 2x(x2+1) = 2.
𝑑𝑥 2 𝑑𝑥
1 𝑑2𝑦 𝑑𝑦
27.If x=tan ( log y) , show that (x2+1) 2 +(2x- a) = 0.
𝑎 𝑑𝑥 𝑑𝑥
𝑑2𝑦 𝑑𝑦
28.Y = log(x+ √𝑥 2 + 1), prove that (x2+1) +x = 0.
𝑑𝑥 2 𝑑𝑥
𝑑𝑦 2
29.If y = asinx + bcosx, prove that y2 + ( ) =a + b2.
2
𝑑𝑥
2
2𝑑 𝑦 𝑑𝑦
30.If y= (cot-1x)2 , then show that (x2+1) 2
+ 2x(x +1) = 2.
𝑑𝑥 2 𝑑𝑥
𝑑2𝑦 𝑑𝑦
31.If y=exsinx, prove that -2 +2y=0.
𝑑𝑥 2 𝑑𝑥
32.If y=3cos(logx)+4sin(logx) show that x2y2+xy1+y=0
𝑑2𝑦 𝑑𝑦
33.If y=ex(sinx+cosx), then prove that -2 +2y=0.
𝑑𝑥 2 𝑑𝑥
𝑎𝑐𝑜𝑠 −1 𝑥 𝑑2𝑦 𝑑𝑦
34.If y=𝑒 show that (1-x2) 2 -x -a2y=0
𝑑𝑥 𝑑𝑥
35.If y=(x+√1 + 𝑥 2 )m , then show that (x2+1)y2+xy1-m2y=0.
𝑠𝑖𝑛−1 𝑥
36.If y= ,show that (1-x2)y2-3xy1-y=0.
√1−𝑥 2
2𝑥+1 .3𝑥 2𝑥+1 .3𝑥
37.Differentiate the function w.r.t x sin-1[ 𝑥
](Ans: log6).
1+36 1+36𝑥
𝑑𝑦 2
𝑠𝑖𝑛 (𝑎+𝑦)
38.If xsin(a+y) + sinacos(a+y) = 0,prove that = .
𝑑𝑥 𝑠𝑖𝑛𝑎
2𝑥−3√1−𝑥 2 𝑑𝑦 −1
39.If y=cos-1[ ] ;find .(Ans: ).
√13 𝑑𝑥 √1−𝑥 2
3𝑥+4√1−𝑥 2 𝑑𝑦 1
40.If y=cos-1[ ] ;find .(Ans: ).
5 𝑑𝑥 √1−𝑥 2
𝑑𝑦 1 1
41.Find , if y =sin-1[x√1 − 𝑥 - √𝑥√1 − 𝑥 2 ](Ans: - .
𝑑𝑥 √1−𝑥 2 2√𝑥−𝑥 2
2𝑥+1 2𝑥+1 𝑙𝑜𝑔2
42.Find the derivatives of the function w.r.t x: f(x)=sin-1( 𝑥
). (Ans: ).
1+4 1+4 𝑥
𝑑𝑦 1−𝑦 2
43.Given y √1 − 𝑥 2 + x √1 − 𝑦 2 =k. show that = −√ .
𝑑𝑥 1−𝑥 2
𝑑𝑦 −1
44.If x√1 + 𝑦 +y√1 + 𝑥=0, prove that = .
𝑑𝑥 (1+𝑥)2
√1+𝑥 2 −√1−𝑥 2 𝑑𝑦 −𝑥
45.If y= tan-1[ ]; show that = .
√1+𝑥 2 +√1−𝑥 2 𝑑𝑥 √1−𝑥 4
𝑥 2 −𝑦 2 𝑑𝑦 𝑦
46.If tan [ ]=a, then prove that = .
𝑥 2 +𝑦 2 𝑑𝑥 𝑥
𝑑𝑦 𝑥 2 √1−𝑦 6
47.if √1 − 𝑥 6 + √1 − 𝑦 6 =a3(x3-y3), prove that = .
𝑑𝑥 𝑦 2 √1−𝑥 6
APPLICATION OF DERIVATIVES
1. Find the rate of change of the area of a circle per second with respect to its radius r when r = 5 cm.
(Ans:- 10𝜋 cm2/sec)
2. The volume of a cube is increasing at a rate of 9 cubic centimetres per second. How fast is the surface area increasing
when the length of an edge is 10 centimetres ?(Ans:- 3.6cm2/sec).
3. A stone is dropped into a quiet lake and waves move in circles at a speed of 4cm per second. At the instant, when the
radius of the circular wave is 10 cm, how fast is the enclosed area increasing?(Ans:- 80𝜋 cm2/sec)
4. The length x of a rectangle is decreasing at the rate of 3 cm/minute and the width y is increasing at the rate of
2cm/minute. When x =10cm and y = 6cm, find the rates of change of
(a) the perimeter and (b) the area of the rectangle. (Ans:- -2cm/min and 2cm2/min).
5. The length x of a rectangle is decreasing at the rate of 5cm/min and the width y is increasing at the rate of
4cm/min.When x=8cm and y=6cm ,find the rate of change of
(a)The perimeter (b)Area of rectangle. (Ans:-2cm/min and 2cm/min).
6. The total cost C(x) in Rupees, associated with the production of x units of an item is given by
C(x) = 0.005x3 – 0.02x2 + 30x + 5000 (Ans:- Rs 30.02).
7. A ladder 5m long is leaning against a wall. Bottom of ladder is pulled along the ground away from wall at the rate of
8
2m/sec.How fast is the height on the wall decreasing when the foot of ladder is 4m away from the wall?(Ans: 3m/sec.)
8. Sand is pouring from the pipe at the rate of 12cm3/sec.The falling sand forms a cone on a ground in such a way that
the height of the cone is always one-sixth of radius of the base. How fast is the height of sand cone increasing when
1
the height is 4cm.(Ans : 48𝜋cm/sec).
9. A stone is dropped into a quiet lake and waves move in circles at the speed of 5 cm/s. At the instant when the radius
of the circular wave is 8 cm, how fast is the enclosed area increasing?(Ans: 80 𝜋 cm2/sec)
10. A balloon, which always remains spherical on inflation, is being inflated by pumping in 900 cubic centimetres of gas
1
per second. Find the rate at which the radius of the balloon increases when the radius is 15 cm.(Ans: 𝜋 cm/sec)
11. A particle moves along the curve 6y = x3 +2. Find the points on the curve at which the y-coordinate is changing 8
−31
times as fast as the x-coordinate. (Ans: (4, 11) & (-4, 3 )
12. Two equal sides of an isosceles triangle with fixed base ‘a’ are decreasing at the rate of 9cm/sec.How fast is the area
of the triangle decreasing when the two sides are equal to ‘a’.(Ans:3√3acm2/sec).
13. Show that the function given by f (x) = 7x – 3 is increasing on R.
14. Show that the function f(x()=x3-3x2+3x , x∈R is increasing on R.
15. Prove that the function given by f (x) = x3 – 3x2 + 3x – 100 is increasing in R.
2𝑥
16. Show that y=log(1+x) - 2+𝑥 , x > -1 is an increasing function of x, throughout its domain.
4𝑠𝑖𝑛𝜃 𝜋
17. Prove that y = 2+𝑐𝑜𝑠𝜃 - 𝜃 is an increasing function in [0, 2 ].
18. Find the intervals in which the function f given by f(x)=2x2-3x is (a)strictly increasing(b)strictly decreasing.
3 3
(Ans:( 4,∞) and (-∞,4).
19. Find the interval in which the function f(x)=2x3+9x2+12x+20 is (a)Increasing (b) Decreasing
(Ans: : Increasing (-∞,-2] U [-1,∞) and Decreasing on [-2,-1].
20. Find the intervals in which the following functions is strictly increasing or strictly decreasing
f(x)=20 - 9x + 6x2 - x3.(Ans: st. increasing (1,3) and st. decreasing (-∞,1) U(3,∞)).
21. Find the interval in which the function f(x)=(x-1)3(x-2)2 is (a)Increasing (b) Decreasing.
8
(Ans: Increasing (-∞,1) U (1,5) U (2, ∞) and decreasing on (8/5 ,2).
22. Find the interval in which the function f(x)=2x3+9x2+12x+20 is (a)Increasing (b) Decreasing
(Ans: : Increasing (-∞,-2] U [-1,∞) and Decreasing on [-2,-1].
23. Find the intervals in which the following functions is strictly increasing or strictly decreasing
f(x)=20-9x+6x2-x3.(Ans: st. increasing (1,3) and st. decreasing (-∞,1) U(3,∞)).
1
24. Find the interval in which the function f is given by f(x)= x3 + 𝑥 3 , x≠ 0 is (a)Increasing (b) Decreasing.
(Ans: Increasing (-∞ ,-1] U [1, ∞) and Decreasing [-1,1] –{0}).
25. Find the value of x for which f(x)=[x(x-2)]2 is an increasing function. (Ans: Increasing 0<x<1 and x>2 )
26. Find the intervals in which the function given by f(x)= sinx + cosx, 0< x< 2𝜋 is (a)Increasing and (b)Decreasing.
𝜋 5𝜋 𝜋 5𝜋
(Ans: Increasing [0, 4 ] and [ 4 , −2𝜋] and Decreasing [ 4 , ].
4
27. Find the intervals in which the function given by f(x) = sinx + cosx, 0≤ x≤ 2𝜋 is strictly increasing or strictly
𝜋 5𝜋 𝜋 5𝜋
decreasing. (Ans: strictly Increasing [0, 4 ) and ( 4 , 2𝜋] and strictly Decreasing ( 4 , ).
4
28. A square piece of tin of side 18 cm is to be made into a box without top, by cutting a square from each corner and
folding up the flaps to form the box. What should be the side of the square to be cut off so that the volume of the box
is the maximum possible.(Ans: 3 cm)
29. A rectangular sheet of tin 45 cm by 24 cm is to be made into a box without top, by cutting off square from each
corner and folding up the flaps. What should be the side of the square to be cut off so that the volume of the box is
maximum ?(Ans: 5 cm)
30. An open topped box is to be constructed by removing equal squares from each corner of a 3 metre by 8 metre
rectangular sheet of aluminium and folding up the sides. Find the volume of the largest such box.
(Ans: Side is 2/3 cm and Volume is 200/27 m3)
31. A wire of length 36m is to be cut into two pieces. One of the piece is to be made into a square and the other into a
circle. What should be the lengths of the two pieces so that the combined area of the square and circle is minimum.
144 36𝜋
(Ans: 𝜋+4 ,𝜋+4).
32. A window has the shape of a rectangle surmounted by an equilateral triangle. If the perimeter of the window is 12m,
12 18−6√3
find the dimensions of the rectangle that will produce the largest area of the window. (Ans:6−√3 and ).
6−√3
2𝑅
33. Show that the height of the cylinder of maximum volume that can be inscribed in a sphere of radius R is .
√3
4𝜋𝑅 3
Also, find the maximum volume.(Ans: 3√3 cu. Units)
𝑥2 𝑦2
34. Find the area of the greatest rectangle that can be inscribed in an ellipse 𝑎2 + 𝑏2 =1.
35. Show that the height of closed right circular cylinder of given surface and maximum volume is equal to diameter of
base.
36. Prove that radius of right circular cylinder of greatest curved surface area which can be inscribed in a given cone if
half of that of the cone.
37. If the length of three sides of a trapezium, other than the base are equal to 10cm each, find the area of trapezium when
it is maximum. (Ans:75√3 cm2).
38. The sum of the perimeter of a circle and square is k, where k is some constant .Prove that the sum of their areas is
𝑘
least when the side of square is double the radius of the circle.(Ans: 𝜋+4)
39. A window is in the form of a rectangle surmounted by a semicircular opening. The total perimeter of the window is
10 m. Find the dimensions of the window to admit maximum light through the whole opening.
40. A helicopter is flying along the curve y= x2+2.A soldier is placed at the point (3,2). Find the nearest distance between
the soldier and the helicopter. (Ans:√5𝑢𝑛𝑖𝑡𝑠)
41. Show that the volume of the greatest cylinder which can be inscribed in a cone of height h and semi vertical angle 𝛼
4
,is 27 𝜋 h3tan2 𝛼.
42. A rectangle is inscribed in a semicircle of radius ‘a’ with one of its sides on the diameter of semi-circle. Find the
dimensions of the rectangle so that its area is maximum. Find the area also. (Ans:a2 sq. units)
43. If the sum of the length of the hypotenuse and a side of a right-angled triangle is given, Show that the area of the
𝜋
triangle is maximum when the angle between them is 3 .
44. Show that the right circular cone of least curved surface and given volume has an altitude equal to 2 time the radius
of the base.
45. Show that the semi-verticle angle of right circular cone of maximum volume and given slant height is tan-1√2.
1
46. Show that the height of the cylinder of maximum volume that can be inscribed in a cone of height h is 3 h.
47. A tank with rectangular base and rectangular sides, open at the top is to be constructed so that its depth is 2m and
volume is 8m3.If building of tank cost Rs 70 per sq. m or the base and Rs 45 per sq. m for sides. What is the cost of
least expensive tank.(Ans:Rs 1000)
8
48. Show that the volume of the largest cone that can be inscribed in a sphere of radius R is 27 of the volume of sphere.
Choose the correct answer for questions:
49. The rate of change of the area of a circle with respect to its radius r at r = 6 cm is
(A) 10𝝅 (B) 12 𝝅 (C) 8 𝝅 (D) 11 𝝅
50. The total revenue in Rupees received from the sale of x units of a product is given by R(x) = 3x2 + 36x + 5. The
marginal revenue, when x = 15 is
(A) 116 (B) 96 (C) 90 (D) 126
2 -x
51. The interval in which y = x e is increasing is:
(a)(- ∞ , ∞) (b)(-2, 0) (c)(2, ∞) (d)(0, 2)
2
52. The point on the curve x = 2y which is nearest to the point (0 , 5) is
(a)(2√𝟐 , 4) (b) (2√𝟐 , 0) (c)(0, 0) (d) (2, 2)
𝟏−𝒙+𝒙𝟐 𝟏
53. For all real values of x, the minimum value of 𝟏+𝒙+𝒙𝟐 is (a)0 (b)1 (c)3 (d)𝟑
𝟏
54. The maximum value of [x(x-1) + ]𝟑 , 0 ≤ x ≤ 1 is
𝟏 𝟏
(a)( 𝟑)𝟑 (b)1/2 (c)1 (d)0
55. A cylindrical tank of radius 10 m is being filled with wheat at the rate of 314 cubic metre per hour. Then the
depth of the wheat is increasing at the rate of
(a) 1 m/h (b) 0.1 m/h (c) 1.1 m/h (d) 0.5 m/h
INTEGRALS
1−𝑠𝑖𝑛𝑥
1. Evaluate: ∫ 𝑑𝑥 (Ans:- tanx – secx +c)
𝑐𝑜𝑠2 𝑥
2−3𝑠𝑖𝑛𝑥
2. Evaluate: ∫ 𝑑𝑥 (Ans: 2tanx – 3secx)
𝑐𝑜𝑠2 𝑥
1 𝑥 1
3. Evaluate: ∫ 1 + 𝑡𝑎𝑛𝑥 𝑑𝑥 (Ans:- 2 + log|𝑐𝑜𝑠𝑥 + 𝑠𝑖𝑛𝑥| + c)
2
1 𝑥 1
4. Evaluate: ∫ 1 + 𝑐𝑜𝑡𝑥 𝑑𝑥 (Ans:- 2 − log|𝑐𝑜𝑠𝑥 + 𝑠𝑖𝑛𝑥| + c)
2
𝑑𝑥
5. Evaluate: ∫ 𝑠𝑖𝑛2 𝑥𝑐𝑜𝑠2 𝑥 . (Ans:- -cotx + tanx + c)
𝑐𝑜𝑠2𝑥−𝑐𝑜𝑠2𝛼
6. Evaluate: ∫ dx(Ans:2(sinx+xcos𝛼)+C)
𝑐𝑜𝑠𝑥−𝑐𝑜𝑠𝛼
1 1 cos (𝑥−𝑎)
7. Evaluate ∫ cos(𝑥−𝑎)cos (𝑥−𝑏) 𝑑𝑥. (Ans:sin (𝑎−𝑏) log |cos (𝑥−𝑏)| +C).
1 1 sin (𝑥−𝑎)
8. Integrate the following ∫ sin(𝑥−𝑎)sin (𝑥−𝑏) 𝑑𝑥 (Ans:sin (𝑎−𝑏)log|sin (𝑥−𝑏)|)
𝑠𝑖𝑛2 𝑥 − 𝑐𝑜𝑠2 𝑥
9. Evaluate: ∫ 𝑑𝑥 (Ans:- tanx + cotx + c)
𝑠𝑖𝑛2 𝑥𝑐𝑜𝑠2 𝑥
𝑑𝑥 1 𝑥−3
10. Evaluate: ∫ 𝑥 2 −6𝑥+13 (𝐴𝑛𝑠: 𝑡𝑎𝑛−1 ( ))
2 2
𝑑𝑥 1 3𝑥−2
11. Evaluate: ∫ 3𝑥 2 +13𝑥−10 ( Ans:- 17 log | 𝑥+5 | )
𝑥+2
12. Evaluate: ∫ 2𝑥 2 +6𝑥+5 dx
5𝑥−2 5 11 3𝑥+1
13. Evaluate: ∫ 1+2𝑥+3𝑥 2 dx(Ans: 6 𝑙𝑜𝑔|1 + 2𝑥 + 3𝑥 2 | +3√2tan-1( )+C)
√2
𝑥+2
14. Evaluate: ∫ √𝑥 2 dx.(Ans: √𝑥 2 + 2𝑥 + 3 +log|(𝑥 + 1) + √𝑥 2 + 2𝑥 + 3 | +C)
+2𝑥+3
3𝑥+5
15. Evaluate: ∫ √𝑥 2 dx(Ans:3√𝑥 2 − 8𝑥 + 7 +17log(|𝑥 − 4|+√𝑥 2 − 8𝑥 + 7)+C)
−8𝑥+7
𝑑𝑥
16. Evaluate: ∫ (𝑥+1)(𝑥+2)
(3𝑥−2) 𝑑𝑥
17. Evaluate: ∫ (𝑥+1)2 (𝑥+3)
2𝑥−1 −1 1 1
18. Evaluate ∫ (𝑥−1)(𝑥+2)(𝑥−3) dx(Ans: 6 log(x-1)-3log(x+2)+2log(x-3)+C)
𝑥2 3 𝑥 2 𝑥
19. Evaluate: ∫ dx.(Ans: tan-1( ) - tan-1( )+C)
(𝑥 2 +4)(𝑥 2 +9) 5 3 5 2
2 1
20. Evaluate: ∫ (1−𝑥)(1+𝑥 2)dx(Ans:-log|1 − 𝑥|+2 log|1 + 𝑥 2 | +tan-1x +C)
𝑥
21. Evaluate: ∫ (1+𝑥)(1+𝑥 2) dx
1
22. Evaluate: ∫ 𝑥 (1+𝑥 2) dx
2𝑥 1 1+𝑥 2
23. Evaluate : ∫ (𝑥 2+1)(𝑥 2+3)dx(Ans:2log|3+𝑥 2|+C)
𝑥 2 +1 3 1 5
24. Evaluate: ∫ (𝑥−1)2 (𝑥+3)dx(Ans:8log(x-1) - 2(𝑥−1) + 8log(x+3) +C)
(𝑥 2 +1)(𝑥 2 +2) 2 𝑥 𝑥
25. Evaluate ∫ (𝑥 2+3)(𝑥 2+4) dx (Ans:x+ tan-1 -3tan-12 +C).
√3 √3
𝑥 1 𝑥−1
26. Evaluate ∫ 𝑥 3 +𝑥 2+𝑥+1 dx(Ans:x+log(x2-2x+3)- tan-1( )+C).
√2 √2
𝑥 2 +1 −1 𝑥 8 𝑥
27. Evaluate: ∫ dx.(Ans: tan-1( ) + tan-1( )+C)
(𝑥 2 +4)(𝑥 2 +25) 14 2 35 5
𝑥𝑠𝑖𝑛−1 𝑥
28. Evaluate: ∫ √1−𝑥2
𝑑𝑥 (Ans:- x - √1 − 𝑥 2 sin-1x + c)
−1
29. Evaluate: ∫ 𝑥𝑡𝑎𝑛 𝑥 dx
4 2𝑠𝑖𝑛𝑥−𝑐𝑜𝑠𝑥
30. Evaluate: ∫ 𝑒 2𝑥 𝑠𝑖𝑛𝑥𝑑𝑥(Ans:5 e2x( )
4
(1+𝑠𝑖𝑛𝑥) 𝑥
31. Evaluate: ∫ (1+𝑐𝑜𝑠𝑥) 𝑒 𝑥 dx.(Ans: extan2 +C)
(1+𝑠𝑖𝑛2𝑥) 1
32. Evaluate ∫ 𝑒 2𝑥 dx(Ans:2e2xtanx +C).
1+𝑐𝑜𝑠2𝑥)
𝑥 (𝑠𝑖𝑛4𝑥−4)
33. Evaluate : ∫ 𝑒 dx.(Ans:excot2x+C).
(1−𝑐𝑜𝑠4𝑥)
𝑥 2 +1 𝑥−1
34. Evaluate ∫ 𝑒 𝑥 ((𝑥+1)2 ) 𝑑𝑥 (Ans:𝑒 𝑥 ( 𝑥+1) + 𝑐).
1 1
35. Evaluate ∫ 𝑒 𝑥 (logx + 𝑥 2 )dx (Ans:ex(logx-𝑥).
5
36. Evaluate: ∫2 |𝑥 − 2| + |𝑥 − 3|+|𝑥 − 5|dx(Ans:23/2)
4
37. Evaluate: ∫0 |𝑥|+ |𝑥 − 2|+ |𝑥 − 4|dx(Ans:20)
2
38. Evaluate ∫0 |𝑥 2 + 2𝑥 − 3| dx(Ans:4).
1
2 |𝑥𝑐𝑜𝑠(𝜋𝑥)|dx (Ans:
3 1
39. Evaluate ∫−1 - 𝜋2).
2𝜋
2
40. Evaluate: ∫−1|𝑥 3 − 𝑥| dx(Ans: 11/4).
𝑠𝑖𝑛𝑥−𝑐𝑜𝑠 𝑥
41. Evaluate : ∫ dx(Ans:-log|(𝑠𝑖𝑛𝑥 + 𝑐𝑜𝑠𝑥) + √(𝑠𝑖𝑛𝑥 + 𝑐𝑜𝑠𝑥)2 − 1|+C)
√𝑠𝑖𝑛2𝑥
𝑡𝑎𝑛𝑥−1
42. Evaluate ∫ √𝑐𝑜𝑡𝑥 + √𝑡𝑎𝑛𝑥 dx(Ans:√2 tan-1( )+C).
√2𝑡𝑎𝑛𝑥
𝑠𝑖𝑛𝑥+𝑐𝑜𝑠𝑥
43. Evaluate ∫ dx (Ans:√2 sin-1(sinx-cosx)+c.
√𝑠𝑖𝑛𝑥𝑐𝑜𝑠𝑥
𝜋
𝑠𝑖𝑛𝑥+𝑐𝑜𝑠𝑥 √3−1
44. Evaluate ∫𝜋3 dx(Ans:2sin-1 ).
√𝑠𝑖𝑛2𝑥 2
6
𝜋
𝑠𝑖𝑛𝑥+𝑐𝑜𝑠𝑥 1
45. Evaluate ∫0 4 dx (Ans:40 log9).
9+16𝑠𝑖𝑛2𝑥
𝜋
46. Evaluate ∫02 √𝑐𝑜𝑡𝑥 + √𝑡𝑎𝑛𝑥 dx (Ans:𝜋√2).
𝜋
𝑠𝑖𝑛2 𝑥 1
47. Evaluate: ∫02 𝑠𝑖𝑛𝑥+𝑐𝑜𝑠𝑥 dx (Ans: 2 log(√2 + 1)).
𝜋
𝜋
48. Evaluate ∫04 log(1 + 𝑡𝑎𝑛𝑥) 𝑑𝑥 ,using properties of definite integrals.(Ans: 8 log2)
𝜋 𝑥𝑡𝑎𝑛𝑥 𝜋
49. Evaluate ∫0 dx (Ans: 2 (𝜋 -2)).
𝑠𝑒𝑐𝑥+𝑡𝑎𝑛𝑥
𝜋
𝑐𝑜𝑠2 𝑥 𝜋
50. Evaluate ∫0 2 dx(Ans: ).
𝑐𝑜𝑠2 𝑥+4𝑠𝑖𝑛2 𝑥 6
𝜋
51. Evaluate ∫0 log(1 + 𝑐𝑜𝑠𝑥) 𝑑𝑥 (Ans:-𝜋log2).
𝜋 𝑥 𝜋2
52. Evaluate ∫0 𝑎2 𝑐𝑜𝑠2 𝑥+𝑏2𝑠𝑖𝑛2 𝑥 dx(Ans:2𝑎𝑏).
𝜋
𝑥𝑠𝑖𝑛𝑥𝑐𝑜𝑠𝑥 𝜋2
53. Evaluate ∫02 𝑠𝑖𝑛4 𝑥+𝑐𝑜𝑠4 𝑥 dx(Ans:16).
𝜋
𝜋
54. Prove that ∫04 log (1 + 𝑡𝑎𝑛𝜃) d𝜃 = 8 log2.
𝜋
𝜋 1
55. Evaluate the integrals :∫02 (2𝑙𝑜𝑔𝑠𝑖𝑛𝑥 − 𝑙𝑜𝑔𝑠𝑖𝑛2𝑥)𝑑𝑥. (Ans: 2 log2).
𝜋
𝜋
56. Evaluate ∫02 𝑠𝑖𝑛2𝑥 𝑡𝑎𝑛−1 (𝑠𝑖𝑛𝑥)𝑑𝑥 (Ans: 2 -1).
𝜋
𝑐𝑜𝑠𝑥 𝜋 1
57. Evaluate ∫02 dx(Ans: - log2)
1+𝑐𝑜𝑠𝑥+𝑠𝑖𝑛𝑥 4 2
𝜋
3 𝑑𝑥 𝜋
58. Evaluate:- ∫ 𝜋 (Ans:- 12 )
1+√𝑡𝑎𝑛𝑥
6
3 5
2 2
59. Evaluate: ∫(1 − 𝑥)√𝑥dx.(Ans:3 𝑥 2 − 𝑥 2 + 𝑐)
5
(𝑙𝑜𝑔𝑥)2 (𝑙𝑜𝑔𝑥)3
60. Evaluate : ∫ dx.(Ans: +c)
𝑥 3
sin (𝑥−𝑎)
61. Evaluate: ∫ sin (𝑥+𝑎) 𝑑𝑥.(Ans: (x+a)cos2a-sin2alog|sin (𝑥 + 𝑎)|+c)
2𝑥 2 +1 −1 7 𝑥
62. Evaluate: ∫ 𝑥 2 (𝑥 2 +4) dx(Ans: 4𝑥 + 8tan-1(2)+C)
𝑑𝑥 1 𝑥5
63. Evaluate: ∫ 𝑥(𝑥 5 +3)(Ans:15log|𝑥 5 +3|+C)
𝑑𝑥 𝑥
64. Evaluate: ∫ 𝑥(𝑥 3 +1)(Ans: log| 1 |+C)
(𝑥 3 +1)3
−𝑐𝑜𝑠2𝑥 𝑐𝑜𝑠6𝑥 𝑐𝑜𝑠4𝑥
65. Evaluate: ∫ 𝑠𝑖𝑛𝑥 𝑠𝑖𝑛2𝑥 𝑠𝑖𝑛3𝑥 𝑑𝑥(Ans: + − +C)
8 24 16
𝑥2 −𝑥𝑠𝑒𝑐𝑥
66. Evaluate: ∫ (𝑥𝑠𝑖𝑛𝑥+𝑐𝑜𝑠𝑥)2 dx(Ans:𝑥𝑠𝑖𝑛𝑥+𝑐𝑜𝑠𝑥+tanx +C)
𝑥 2 +4 1 𝑥 2 −4
67. Evaluate : ∫ 𝑥 4 +16 dx(Ans:2√2tan-1( 𝑥√2 )+C
𝑥 2 +1 1 𝑥 2 −1
68. Evaluate : ∫ 𝑥 4 +1 dx(Ans: tan-1(𝑥2√2)+C)
√2
1 𝑥
69. Evaluate: ∫[log(𝑙𝑜𝑔𝑥) + ] dx(Ans:xlog(logx) - 𝑙𝑜𝑔𝑥+C).
(𝑙𝑜𝑔𝑥)2
𝑥2 𝑥 1
70. Evaluate : ∫ 𝑥𝑠𝑖𝑛−1 𝑥dx(Ans: sin-1x + 4 √1 − 𝑥 2 - 4sin-1x +C
2
1 𝑥2 𝑥
71. Evaluate: ∫ 𝑥𝑙𝑜𝑔(𝑥 + 1)dx.(Ans:2(x2-1)log(x+1) - + 2 +C).
4
𝑡𝑎𝑛∅+𝑡𝑎𝑛3 ∅ −1 1 1 2𝑡𝑎𝑛∅−1
72. Evaluate ∫ d∅ .(Ans: 3 log(tan∅ +1) +6 log(tan2∅ -tan∅ +1 ) + tan-1( )+C.
1+𝑡𝑎𝑛3 ∅ √3 √3
𝑥2 1 𝑥 2 −1 1 𝑥 2 −𝑥√2+1
73. Evaluate ∫ 𝑥 4 +1 dx(Ans:2√2 tan-1( 𝑥√2 )+4√2 log |𝑥 2 +𝑥√2+1| +C.)
2𝜋 1
74. Evaluate: ∫0 1+𝑒 𝑠𝑖𝑛𝑥
dx(Ans: 𝜋)
1 𝜋
75. Evaluate ∫0 𝑠𝑖𝑛−1 (𝑥√1 − 𝑥 - √𝑥√1 − 𝑥 2 )dx .(Ans:-1+4 ).
1 −1 1 4
76. Evaluate ∫ 𝑠𝑖𝑛𝑥(5−4𝑐𝑜𝑠𝑥) dx(Ans: 18 log(cosx-1) -18 log(1+cosx)-9 log(5-4cosx)+c).
1−√𝑥
77. Evaluate ∫ √1+ 𝑥 dx (Ans:cos-1√𝑥 +√𝑥 √1 − 𝑥 -2√1 − 𝑥 +C).

𝜋 𝑥𝑑𝑥 𝜋2
78. Using properties of definite integrals ,Evaluate ∫0 (Ans:4√3).
4−𝑐𝑜𝑠2 𝑥
𝑥2 𝑠𝑖𝑛𝑥−𝑥𝑐𝑜𝑠𝑥
79. Find ∫ (𝑥𝑠𝑖𝑛𝑥+𝑐𝑜𝑠𝑥)2 dx (Ans: 𝑥𝑠𝑖𝑛𝑥+𝑐𝑜𝑠𝑥 +C).
1 𝑥
80. Evaluate ∫0 𝑐𝑜𝑡-1(1-x+x2)dx (Ans: − 𝑙𝑜𝑔2).
2
1 2𝑥−1
81. Find the value of ∫0 𝑡𝑎𝑛−1 (1−𝑥−𝑥 2 ) dx(Ans:0).
𝑥2 1 𝑥 2 −𝑥+1 1 𝑥 2 −1
82. Evaluate ∫ dx(Ans: log( )+ tan-1( )+C).
𝑥 4 +𝑥 2 +1 4 𝑥 2 +𝑥+1 2√3 𝑥 √3
𝜋
𝑑𝑥 1 3+√5
83. ∫0 2 (Ans: log( )).
2𝑐𝑜𝑠𝑥+4𝑠𝑖𝑛𝑥 2√5 3−√5
𝑎 𝑥 𝜋−2
84. Evaluate: ∫0 𝑠𝑖𝑛−1 √ dx(Ans:a( ).
𝑥+𝑎 2

Choose the correct answer


𝟏
85. The anti derivative of √𝒙 + equals
√𝒙
𝟏 𝟏 𝟏 𝟐 𝟐 𝟏 𝟐 𝟑 𝟏 𝟑 𝟑 𝟏 𝟏
(a) 𝒙𝟑 + 𝟐𝒙𝟐 + C (b) 𝒙𝟑 + 𝒙𝟐 + C (c) 𝒙𝟐 + 𝟐𝒙𝟐 + C (d) 𝒙𝟐 + 𝒙𝟐 + C
𝟑 𝟑 𝟐 𝟑 𝟐 𝟐
𝒅 𝟑
86. If 𝒇(𝒙) = 4x3 - such that f(2) = 0. Then f(x) is
𝒅𝒙 𝒙𝟒
𝟏 𝟏𝟐𝟗 𝟏 𝟏𝟐𝟗 𝟏 𝟏𝟐𝟗 𝟏 𝟏𝟐𝟗
(a)x4 + − (b) x3 + + (c) x4 + + (d) x3 + −
𝟖𝒙𝟑 𝒙𝟒 𝟖 𝒙𝟑 𝟖 𝒙𝟒 𝟖
𝟏𝟎𝒙𝟗 + 𝟏𝟎𝒙 𝒍𝒐𝒈𝟏𝟎
87. ∫ 𝒅𝒙 equals
𝒙𝟏𝟎 +𝟏𝟎𝒙
(a)10x – x10 + C (b)10x + x10 + C (c)(10x – x10 )-1 +C (d) log(10x + x10 ) + C
𝒅𝒙
88. ∫ equals
𝒔𝒊𝒏𝟐 𝒙𝒄𝒐𝒔𝟐 𝒙
(a)tanx + cotx + C (b)tanx- cotx + C (c)tanxcotx + C (d)tanx – cot2x + C
𝒔𝒊𝒏𝟐 𝒙− 𝒄𝒐𝒔𝟐 𝒙
89. ∫ 𝒅𝒙 is equal to
𝒔𝒊𝒏𝟐 𝒙𝒄𝒐𝒔𝟐 𝒙
(a)tanx + cotx + C (b)tanx + cosecx + C (c)-tanx + cotx + C (d)tanx + secx + C
𝒆𝒙 (𝟏+𝒙)
90. ∫ 𝒅𝒙 equals to
𝒄𝒐𝒔𝟐 (𝒆𝒙 𝒙)
(a) -cot(exx) + C (b) tan(xex) + C (c)tanex + C (d) cotex + C
𝒅𝒙
91. ∫ equals to
𝒙𝟐 +𝟐𝒙+𝟐
(a)xtan-1(x+1) + C (b)tan-1(x+1) +C (c)(x+1)tan-1x-1+ C (d)tan-1x + C
𝒅𝒙
92. ∫ equals to
√𝟗𝒙−𝟒𝒙𝟐
𝟏 −𝟏 𝟗𝒙−𝟖 𝟏 𝟖𝒙−𝟗 𝟏 𝟗𝒙−𝟖 𝟏 𝟗𝒙−𝟖
(a) 𝒔𝒊𝒏 ( )+C (b) 𝒔𝒊𝒏−𝟏 ( ) + C(c) 𝒔𝒊𝒏−𝟏 ( )+C (d) 𝒔𝒊𝒏−𝟏 ( )+C
𝟗 𝟖 𝟐 𝟗 𝟑 𝟖 𝟐 𝟗
𝒙𝒅𝒙
93. ∫ (𝒙−𝟏)(𝒙−𝟐) equals
(𝒙−𝟏)𝟐 (𝒙−𝟐)𝟐 (𝒙−𝟏)𝟐
(a)log | |+C (b) log | | + C (c) log | |+C (d) log |(𝒙 − 𝟏)(𝒙 − 𝟐)| + C
𝒙−𝟐 𝒙−𝟏 (𝒙−𝟐)𝟐
𝒅𝒙
94. ∫ equals
𝒙(𝒙𝟐 +𝟏)
𝟏 𝟏
(a)log|𝒙| - log(x2+1) + C (b) log|𝒙| + log(x2+1) + C
𝟐 𝟐
𝟏 𝟏
(c) -log|𝒙| + log(x2+1) + C (d) log|𝒙| + log(x2+1) + C
𝟐 𝟐
𝒙𝟑
95. ∫ 𝒙𝟐 𝒆 𝒅𝒙 equals
𝟏 𝟑 𝟏 𝟐 𝟏 𝟑 𝟏 𝟐
(a) 𝒆𝒙 + 𝑪 (b) 𝒆𝒙 + 𝑪 (𝒄) 𝒆𝒙 + 𝑪 (d) 𝒆𝒙 + 𝑪
𝟑 𝟑 𝟐 𝟐
96. ∫ 𝐞𝐱 𝐬𝐞𝐜𝐱(𝟏 + 𝐭𝐚𝐧𝐱)dx equals
(a)𝐞𝐱 𝐜𝐨𝐬𝐱 + 𝐂 (b) 𝐞𝐱 𝐬𝐞𝐜𝐱 + 𝐂 (c) 𝐞𝐱 𝐬𝐢𝐧𝐱 + 𝐂 (d) 𝐞𝐱 𝐭𝐚𝐧𝐱 + 𝐂
97. ∫ √𝟏 + 𝒙𝟐 dx
𝒙 𝟏 𝟐 𝟑
(a) √𝟏 + 𝒙𝟐 + log |𝒙 + √𝟏 + 𝒙𝟐 | + C (b) ( 𝟏 + 𝒙𝟐 )𝟐 + C
𝟐 𝟐 𝟑
𝟐 𝟑 𝒙𝟐 𝟏
𝟐 𝟐
(c) x ( 𝟏 + 𝒙 ) + C (d) √𝟏 + 𝒙𝟐 + 𝒙𝟐 log |𝒙 + √𝟏 + 𝒙𝟐 | + C
𝟑 𝟐 𝟐
98. ∫ √𝒙𝟐 − 𝟖𝒙 + 𝟕 dx
𝒙−𝟒
(a) √𝒙𝟐 − 𝟖𝒙 + 𝟕 + 9log|𝒙 − 𝟒 + √𝒙𝟐 − 𝟖𝒙 + 𝟕| + C
𝟐
𝒙+𝟒
(b) √𝒙𝟐 − 𝟖𝒙 + 𝟕 + 9log|𝒙 + 𝟒 + √𝒙𝟐 − 𝟖𝒙 + 𝟕| + C
𝟐
𝒙−𝟒
(c) √𝒙𝟐 − 𝟖𝒙 + 𝟕 -3√𝟐log|𝒙 − 𝟒 + √𝒙𝟐 − 𝟖𝒙 + 𝟕| + C
𝟐
𝒙−𝟒 𝟗
(d) √𝒙𝟐 − 𝟖𝒙 + 𝟕 - log|𝒙 − 𝟒 + √𝒙𝟐 − 𝟖𝒙 + 𝟕| + C
𝟐 𝟐
√𝟑 𝒅𝒙
99. ∫𝟏 equals
𝟏+𝒙𝟐
𝝅 𝟐𝝅 𝝅 𝝅
(a) (b) (c) (d)
𝟑 𝟑 𝟔 𝟏𝟐
𝟐
𝒅𝒙
100. . ∫𝟎𝟑 𝟗 equals
𝟒+𝒙𝟐
𝝅 𝝅 𝝅 𝝅
(a) (b) (c) (d)
𝟔 𝟏𝟐 𝟐𝟒 𝟒
𝟏
𝟏 (𝒙−𝒙𝟑 )𝟑
101. The value of the integral ∫ 𝟏 dx is
𝟑 𝒙𝟒
(a)6 (b)0 (c)3 (d)4
𝒙
102. If f(x) = ∫𝟎 𝒕𝒔𝒊𝒏𝒕𝒅𝒕 , then f ‘ (x) is
(a)cos+xsinx (b)xsinx (c)xcosx (d)sinx+xcosx
𝝅
103. The value of ∫ 𝟐
−𝝅 (𝒙𝟑 + 𝒙𝒄𝒐𝒔𝒙 + 𝒕𝒂𝒏𝟓 𝒙 + 𝟏) dx
𝟐
(a)0 (b)2 (c)𝝅 (d)1
𝝅
𝟒+𝟑𝒔𝒊𝒏𝒙
104. The value of ∫𝟎 𝐥𝐨𝐠(𝟐 )𝒅𝒙 is
𝟒+𝟑𝒄𝒐𝒔𝒙
(a)2 (b)3/4 (c)0 (d)-2
𝒅𝒙
105. ∫ 𝒆𝒙+ 𝒆−𝒙 is equal to
(a)tan-1(ex) + C (b)tan-1(e-x) + C (c)log(ex – e-x) +C (d)log(ex + e-x) +C
𝒄𝒐𝒔𝟐𝒙
106. ∫ (𝒔𝒊𝒏𝒙+𝒄𝒐𝒔𝒙)𝟐 dx is equal to
−𝟏 𝟏
(a) +C (b)log|𝒔𝒊𝒏𝒙 + 𝒄𝒐𝒔𝒙| +C (c)log|𝒔𝒊𝒏𝒙 − 𝒄𝒐𝒔𝒙| +C (d) .
𝒔𝒊𝒏𝒙+𝒄𝒐𝒔𝒙 (𝒔𝒊𝒏𝒙+𝒄𝒐𝒔𝒙)𝟐
𝒃
107. If f(a+b-x) = f(x) , then ∫𝒂 𝒙𝒇(𝒙)𝒅𝒙 is equal to
𝒂+𝒃 𝒃 𝒂+𝒃 𝒃 𝒃−𝒂 𝒃 𝒂+𝒃 𝒃
(a)
𝟐
∫𝒂 𝒇(𝒃 − 𝒙)𝒅𝒙 (b)
𝟐
∫𝒂 𝒇(𝒃 + 𝒙)𝒅𝒙 (c)
𝟐
∫𝒂 𝒇(𝒙)𝒅𝒙 (d)
𝟐
∫𝒂 𝒇(𝒙)𝒅𝒙
APPLICATION OF INTEGRALS
1. Find the area enclosed by the circle x2 + y2= a2.(Ans:- 𝜋𝑎2 )
𝑥2 𝑦2
2. Find the area enclosed by the ellipse 2
+ = 1 (Ans:- 𝜋𝑎𝑏)
𝑎 𝑏2
𝑥2 𝑦2
3. Find the area of the region bounded by the ellipse + = 1. (Ans:- 12𝜋)
16 9
𝑥2 𝑦2
4. Find the area of the region bounded by the ellipse + = 1. (Ans:- 6𝜋)
4 9
5. Find the area of the region bounded by the line y = 3x + 2, the x-axis and the ordinates x = –1 and x =
1.(Ans:- 13/3)
6. Find the area bounded by the curve y = cos x between x = 0 and x = 2p. (Ans:- 4 sq. Units)
7. Find the area under the given curves and given lines:
(i)y = x2 , x=1 , x= 2 and x-axis (ii)y = x4 , x= 1, x = 5 and x-axis
8. Sketch the graph of y= |𝑥 + 3| and evaluate the area under the curve y= |𝑥 + 3| above x-axis and
between x = -6 to x=0.(Ans:9 sq. units)
9. Find the area bounded by the curve y = sin x between x = 0 and x = 2p.(Ans: 4 sq. units)
Choose the correct answer
10. Area of the region bounded by the curve y2 = 4x, y axis and the line y =3 is
𝝅 𝝅
(a)2 (b) 9/4 (c) (d)
𝟑 𝟒
11. Area lying in the first quadrant and bounded by the circle x2 + y2 = 4 and the lines x =0 and x =2 is:
𝝅 𝝅 𝝅
(a)𝝅 (b) (c) (d)
𝟐 𝟑 𝟒
12. Area bounded by the curve y = x3, the axis and the ordinates x = -2 and x = 1 is
(a)- 9 (b)-15/4 (c)15/4 (d)17/4
13. The area bounded by the curve y = x | x| , x-axis and the ordinates x = – 1 and x = 1 is given by
(a)0 (b)1/3 (c)2/3 (d)4/3
[Hint: y = x2 if x > 0 and y = - x2 if x < 0]

14. Make a rough sketch of the region :{(x,y):0≤y≤x2,0≤y≤x+2;0≤x ≤ 3} and find the area of the region,
43
using the method of integration.(Ans: 𝑠𝑞. 𝑢𝑛𝑖𝑡𝑠).
6
15. Make a rough sketch of the region :{(x,y):0≤y≤x2,+1,0≤y≤x+1;0≤x ≤ 2} and find the area of the region,
23
using the method of integration.(Ans: 𝑠𝑞. 𝑢𝑛𝑖𝑡𝑠).
6
16. Make a rough sketch of the region :{(x,y):0≤y≤x2, 0≤y≤x; 0≤x ≤ 2} and find the area of the region, using
11
the method of integration.(Ans: 𝑠𝑞. 𝑢𝑛𝑖𝑡𝑠).
6
17. Using integration, find the area of triangle ABC the co-ordinates of whose vertices are A(2,5), B(4,7) and
C(6,2).(Ans:7 sq. units).
18. Using integration, find the area of triangle ABC the co-ordinates of whose vertices are A(-1, 2),B(1,5) and
C(3,4).(Ans: 4 sq. units)
19. Using integration, find the area of triangle ABC the co-ordinates of whose vertices are A(-1, 0),B(1,3) and
C(3,2).(Ans: 4 sq. units)
20. Using integration ,find the area of the triangular region whose sides have the equations y=2x+1, y=3x+1 and
x=4.(Ans:8 sq. units).
21. Using method of integration find the area of region bounded by the lines 2x+y=4,3x-2y=6 and x-3y+5=0.
(Ans: 3.5 sq. units)
22. Using method of integration find the area of region bounded by the lines 3x-2y+1=0,2x+3y-21 and x-
13
5y+9=0.(Ans: sq. units)
2
23. Find the area bounded by the curve y = 3x, x-axis and between the ordinates x =1 and x =3.
(Ans: - 12 sq. units)
24. Find the area bounded by the curve y = 5x. x-axis and between the ordinates x = 2 and x =5.
105
(Ans:- sq. units)
2
25. Find the area of the region bounded by the curves x2 + y2 =4, y = √3 x and x-axis in the first quadrant.
2𝜋
(Ans:- )
3
26. The area between x = y2 and x =4 is divided into two equal parts by line x = a. Find the value of a.
2
(Ans:- a = (4)3 )
27. Using integration, find the area bounded by the curves.(Draw rough sketch also)
Y = 1 + |𝑥 + 1|, x = -3 , x = 3, y = 0.(Ans:- 16 sq. units)
𝑥2 𝑦2 𝑥 𝑦
28. Find the area of smaller region bounded by the ellipse + =1 and the straight line + =1.
16 9 4 3
(Ans:3(𝜋-2)).
𝑎 𝑎2
29. Find the area of the smaller part of the circle x2 + y2 = a2 cut off by the line x = .(Ans:- (𝜋 – 2).
√2 4
30. Find the area of the following region using integration: { (x, y) : x2 ≤ y ≤ x}
1
31. Find the area of the region bounded by the parabola y=x2 and y = |𝑥|.(Ans: sq. units)
3
9
32. Using integration, find the area bounded by the curve x2=4y and the line x=4y-2.(Ans: sq. units)
8
33. Find the area of the region {(x,y):x2+y2≤4 ,x+y≥2}(Ans:𝜋-2 sq units)
3𝑥 2
34. Find the area of the region included between the parabola y = and the line 3x-2y+12=0.(Ans:27 sq.units)
4
35. Using integration, find the area of the region bounded by the curve x2 = 4y and the line x=4y-2.
9
(Ans: sq .units)
8
36. Draw a rough sketch of the curve y=sinx and y=cosx in first quadrant , then find the area of the region
enclosed by the curves and y-axis.(Ans:√2 - 1 sq. units )
37. Draw the rough sketch of the curves y2=x+1 and y2=-x+1 and determine the area enclosed by the two curves.
8
(Ans: sq units)
3
𝜋
38. Find the area of the region bounded by the curve y=√1 − 𝑥 2 ,the line y=x and the positive x-axis.(Ans: ).
8
2 32
39. Find the area of the region bounded by the curve y = x and the line y = 4.(Ans:- )
3
40. Find the area of the region bounded by the curve y2 = x and the lines x = 1, x = 4 and the x-axis in the
first quadrant.
41. Find the area of the region bounded by y2= 9x, x = 2, x = 4 and the x-axis in the first quadrant.
42. Find the area of the region bounded by the parabola y = x2 and y = x .
43. Find the area bounded by the curve x2 = 4y and the line x = 4y – 2.
44. Find the area of the region bounded by the curve y2 = 4x and the line x = 3.
45. Find the area of the region bounded by the two parabolas y = x2 and y2 = x.(Ans:- 1/3)
8
46. Find the area of the parabola y2= 4ax bounded by its latus rectum.(Ans:- 𝑎2 )
3
47. Using the method of integration find the area bounded by the curve |𝑥| + |𝑦| =1. (Ans:- 4)
DIFFERENTIAL EQUATIONS
𝑑2 𝑦 𝑑𝑦
1. Write the degree of the D.E x3 (𝑑𝑥 2 )2 +x(𝑑𝑥 )4 =0.(Ans:2)
𝑑2 𝑦 2 𝑑2 𝑥 𝑑𝑦
2. Write the degree of the D.E( ) -2 - +1=0.(Ans:2).
𝑑𝑥 2 𝑑𝑥 2 𝑑𝑥
𝑑𝑦 −2𝑥
3. Solve the D.E 2x2 𝑑𝑥 -2xy +y2 =0(Ans:y = −𝑙𝑜𝑔𝑥+𝑐)
𝑑𝑦 𝑥3 𝜋
4. Solve the D.E 𝑑𝑥 = 1+x2+y2+x2y2 , given that y=1 when x=0.(Ans:y = tan(x+ 3 + 4 )
𝑑𝑦 2
5. Solve x(x2 – 1) 𝑑𝑥 =1; y=0 when x=2.(Ans:c=log )
√3
𝜋
6. Find the particular solution of the D.E (1+e2x)dy+(1+y2)exdx=0 given that y=1 when x=0.(Ans:tan-1y+tan-1ex=2 ).
𝑑𝑦 𝜋 −𝜋 2
7. Solve D.E +ycotx = 4xcosecx, given that y=0 when x= 2 .(Ans:c= )
𝑑𝑥 2
𝑑𝑦 1 −𝜋
8. Solve the following D.E ( 1+x2) 𝑑𝑥 +2xy = 1+𝑥 2 ,given y=o when x=1(Ans:c = )
4
9. Solve the following D.E xdy -y dx = √𝑥 2 + 𝑦 2 dx(Ans: y+ √𝑥 2 + 𝑦 2 =Ax2.
𝑑𝑥
10. Solve the following D.E (y+3x2)𝑑𝑦 = x.(Ans: y= 3x2+C)
2𝑥 2 𝑐
11. Solve the D.E xdy –(y+2x2)dx =0(Ans: y= +𝑥)
3
𝑥3 𝑐
12. Solve the D.E xdy +(y – x )dx =0(Ans:y =
3
+𝑥)
4
𝑒 𝑥 −1
13. Solve the following D.E extany dx +(1-ex)sec2ydy =0(Ans: y= tan-1( )).
𝑐
𝑑𝑦
14. Solve D.E cos2x𝑑𝑥 +y =tanx(Ans: y= tanx -1 +ce-tanx)
(𝑙𝑜𝑔𝑥)2
15. Solve the D.E (1+y2)(1+logx)dx +x dy=0(Ans:y= tan( -logx – -c)
2
log (𝑠𝑖𝑛𝑥) 𝑐
16. Solve (1+x2) dy +2xydx =cotx dx(Ans: y= + 1+𝑥 2 )
1+𝑥 2
𝑑𝑦 𝑦 𝑦 𝑦
17. Solve the D.E:𝑑𝑥 - 𝑥 + 𝑐𝑜𝑠𝑒𝑐(𝑥 )=1,when x=1 and y=0.(Ans:cos𝑥 =logx+1.)
𝑦 𝑦
18. Solve the following D.E [xsin2(𝑥 ) –y]dx +xdy =0(Ans: 𝑥 =cot-1(logx –C).
𝑑𝑦 1 𝑐
19. Solve the following D.E x 𝑑𝑥 +y – x +xycotx =0(Ans:y= -cotx +𝑥 + 𝑥𝑠𝑖𝑛𝑥)
𝑦
𝑦 𝑦 𝑦 𝑦 𝑑𝑦 𝑠𝑒𝑐
20. Solve the following D.E (xcos𝑥 + ysin𝑥 )y –(ysin𝑥 -xcos𝑥 )x𝑑𝑥 =0.(Ans: log| 𝑥
|=C).
𝑥𝑦
𝑦
𝑦 𝑙𝑜𝑔 −1
𝑥
21. Solve the D.E ydx +x log𝑥 dy-2xydy =0(Ans: log | |=C).
𝑦
22. Find the particular solution of the D.E (tan-1 –x)dy =(1+y2)dx, given that when x=0, y=0
−1 𝑦
(Ans:x=tan-1y -1 +𝑒 𝑡𝑎𝑛 )
𝑑𝑦 √𝑥 2 +1−1
23. Solve the following D.E √1 + 𝑥 2 + 𝑦 2 + 𝑥 2 𝑦 2 +xy𝑑𝑥 =0.(Ans:√1 + 𝑥 2 + √1 + 𝑦 2 +log( )=C).
𝑥
𝑑𝑦 1 3 1
24. Solve the following D.E (x3+x2+x+1)𝑑𝑥 =2x2+x(Ans:y=2 log(x+1)+4 log (x2+1)-2tan-1x+c).
25. Find the particular solution of the D.E (x-siny)dy+(tanydx=0:given that y=0 whenx=0.(Ans:c=1/4).
𝑥 𝑥 𝑥
26. Solve the following D.E : y𝑒 𝑦 dx=(x𝑒 𝑦 + 𝑦)dy.(Ans:𝑒 𝑦 = 𝑙𝑜𝑔𝑐𝑦. )
𝜋
27. Solve the following D.E(1+y+x2y)dx+(x+x3)dy=0,where y=0 when x=1.(Ans:C= 4 ).
𝑦 𝑦 3𝜋
28. Find the particular solution of the D.E (xdy-ydx)ysin(𝑥 )=(ydx+xdy)xcos𝑥 ,giventhat y=𝜋 when x=3.(Ans:C= 2 ).
29. Solve xdy-ydx=√𝑥 2 + 𝑦 2dx(Ans:{y+√𝑥 2 + 𝑦 2}2=C2x4).
𝑑𝑦 −3
30. Find the general solution of the D.E 𝑑𝑥 -3y=sin2x.(Ans:y= 13 (2𝑐𝑜𝑠2𝑥 + 𝑠𝑖𝑛2𝑥) + 𝐶𝑒3x).
𝑑𝑦
31. Solve the D.E 𝑑𝑥 +ysec2x=tanxsec2x;y(0)=1.(Ans:y=tanx-1+2e-tanx.)
𝑑𝑦
32. Solve the D.E x𝑑𝑥 -y=√𝑥 2 + 𝑦 2 .(Ans:y+√𝑥 2 + 𝑦 2 =Cx2).
𝑑𝑦
33. Find the particular solution of the D.E log(𝑑𝑥 )=3x+4y’given that y=0 when x=0.(Ans:4e3x+3e-4y-7=0).
𝑦 𝑦
34. Show that the following D.E is homogeneous and then solve it: ydx+xlog(𝑥 )dy-2xdy=0.(Ans:log(𝑥 )-1=cy.
𝑑𝑦 2 −2(1+𝑙𝑜𝑔𝑥)
35. Solve the following D.E:xlogx𝑑𝑥 +y= 𝑥logx(Ans:ylogx= +C).
𝑥
36. Solve the D.E(x2-y2)dx+2xydy=0 given that y=1,when x=1.(Ans:x2+y2=cx).
37. Solve the D.E (3xy+y2)dx+(x2+xy)dy=0(Ans:y=-4logx+c).
𝑑𝑦 𝑥4
38. Solve the D.E : 𝑑𝑥 =ex-y+x3e-y.(Ans:ey=ex+ 4 +C)
𝑑𝑦 𝑥(2𝑙𝑜𝑔𝑥+1)
39. Solve the following D.E: 𝑑𝑥 = 𝑠𝑖𝑛𝑦+𝑦𝑐𝑜𝑠𝑦.(Ans:siny=x2logx+c).
𝑦
𝑦 𝑑𝑦 𝑦
40. Solve the initial value problem: x𝑒 𝑥 -ysin(𝑥 )+x𝑑𝑥 sin(𝑥 )=0;y(1)=0.(Ans:c=1+logx2).
𝑑𝑦 −2𝑥
41. Solve the D.E (x-y)𝑑𝑥 =x+3y.(Ans:𝑥+𝑦=log(c(x+y)).
𝑑𝑦 𝜋 2
42. Solve the D.E sinx 𝑑𝑥 +ycosx=2sin-1xcosx given that y=1 when x= 2 .(Ans:ysinx=3sin3x+c).
43. For the following D.E,find a particular solution satisfying the given condition:
𝑑𝑦 𝜋
-3ycotx=sin2x; y=2 when x= 2 (Ans:y=4sin3x-2sin2x).
𝑑𝑥
𝒅𝟐 𝒚 𝒅𝒚 𝒅𝒚
44. The degree of the differential equation (𝒅𝒙𝟐 )𝟑 + (𝒅𝒙)𝟐 + sin (𝒅𝒙) + 1 = 0 is
(a)3 (b)2 (c)1 (d)Not defined
𝒅𝟐 𝒚 𝒅𝒚
45. The order of the differential equation 2x2 𝒅𝒙𝟐 -3 𝒅𝒙 + y = 0
(a)2 (b)1 (c)0 (d)Not defined
46. The number of arbitrary constants in the general solution of a differential equation of fourth order are:
(a)0 (b) 2 (c) 3 (d) 4
47. The number of arbitrary constants in the particular solution of a differential equation
of third order are:
(a) 3 (b) 2 (c) 1 (d) 0
𝒅𝒚
48. The general solution of the D.E 𝒅𝒙 = e x + y is
(a)ex + e-y = C (b)ex + ey = C (c)e-x + ey = C (d)e-x + e-y = C
𝒅𝒙 𝒙
49. A homogeneous differential equation of the from = 𝒉( ) can be solved by making the substitution.
𝒅𝒚 𝒚
(a) y = vx (b) v = yx (c) x = vy (d) x = v
50. Which of the following is a homogeneous differential equation?
(a) (4x + 6y + 5) dy – (3y + 2x + 4) dx = 0 (b) (xy) dx – (x3 + y3) dy = 0
3 2
(c) (x + 2y ) dx + 2xy dy = 0 (d) y2 dx + (x2 – xy – y2) dy = 0
𝒅𝒚
51. The Integrating Factor of the differential equation x𝒅𝒙 – y = 2x2 is
𝟏
(a)e-x (b)e-y (c)𝒙 (d)x
𝒅𝒙
52. The Integrating Factor of the differential equation ( 1 – y2)𝒅𝒚 + yx = ay.
𝟏 𝟏 𝟏 𝟏
(a)𝟏−𝒚𝟐 (b) (c) 𝒚𝟐−𝟏 (d)
√𝟏−𝒚𝟐 √𝒚𝟐 −𝟏
𝒚𝒅𝒙−𝒙𝒅𝒚
53. The general solution of the differential equation 𝒚
= 0 is
(a)
xy = C (b) x = Cy2 (c) y = Cx (d) y = Cx2
x x
54. The general solution of the differential equation e dy + (ye + 2x )dx = 0 is
(a)xey + x2 = C (b)xey + y2 = C (c)yex + x2 = C (d)yey + x2 = C
VECTOR ALGEBRA
𝑖 5
1. Write a unit vector in the direction of the sum of vectors 𝑎⃗ =2i-j+2k and 𝑏⃗⃗= -i+j+3k(Ans: + k)
√26 √26
2. If 𝑎⃗=xi+2j-zk and 𝑏⃗⃗= 3i-yj+k are two equal vectors then write the value of x+y+z(Ans: 0)
3. Find |𝑥⃗| if for a unit vector a , (𝑥⃗-𝑎⃗)( 𝑥⃗+𝑎⃗)=15.(Ans:4)
15 5
4. Find a vector of magnitude 5 units and parallel to the resultant of 𝑎⃗ =2i+3j-k and 𝑏⃗⃗= i-2j+k(Ans: i + j)
√10 √10
5. Let 𝑎⃗ = i+j+k, 𝑏⃗⃗=4i-2j+3k and 𝑐⃗= i-2j+k.Find a vector of magnitude 6 units which is parallel to the vector 2𝑎⃗-𝑏⃗⃗ +3𝑐⃗
(Ans:2i-4j+4k)
6. Find the position vector of a point R which divides the line joining two points P and Q whose position vectors are
2𝑎⃗+𝑏⃗⃗ and 𝑎⃗-3𝑏⃗⃗ respectively, externally in the ratio 1:2.Also show that P is in the mid-point of line segment
1
RQ.(Ans:3𝑎⃗+5𝑏⃗⃗, (1,2)
5 2 2
7. If 𝑎⃗= i+j+k and 𝑏⃗⃗=j-k find a vector 𝑐⃗ such that 𝑎⃗ X c=𝑏⃗⃗ and 𝑎⃗. 𝑐⃗=3(Ans: 3 i + 3 j + 3 k)
1
8. Using vectors, find the area of the triangle ABC whose vertices are A(1,2,3), B(2,-1,4) and C(4,5,-1)(Ans:2 √274)
9. If 𝑎⃗=i-j+7k and 𝑏⃗⃗=5i-j+λk then find a value of λ ,so that 𝑎⃗+𝑏⃗⃗ and 𝑎⃗-𝑏⃗⃗ are perpendicular vectors.(Ans: 5)
10. If 𝑝⃗= 5i+λj -3k and 𝑞⃗=i+3j-5k, then find the value of λ so that 𝑝⃗+𝑞⃗ and 𝑝⃗-𝑞⃗ are perpendicular vectors.(Ans: 1
11. If 𝑎⃗, 𝑏⃗⃗, 𝑐⃗ are three vectors such that |𝑎⃗|=5, |𝑏⃗⃗|=12 and |𝑐⃗|=13 and 𝑎⃗+𝑏⃗⃗+𝑐⃗=0, find the value of 𝑎⃗. 𝑏⃗⃗+𝑏⃗⃗. 𝑐⃗+𝑐⃗. 𝑎⃗.
(Ans:-169)
12. Let 𝑎⃗= i+4j+2k , 𝑏⃗⃗=3i-2j+7k and 𝑐⃗=2i-j+4k .Find a vector p which is perpendicular to both 𝑎⃗ and 𝑏⃗⃗ and 𝑝⃗. 𝑐⃗=18.
(Ans:64i-2j-28k)
13. Let 𝑎⃗ = 4𝑖̂ + 5𝑗̂ − 𝑘̂, 𝑏⃗⃗ = 𝑖̂ − 4𝑗̂ + 5𝑘̂ and 𝑐⃗ = 3𝑖̂ + 𝑗̂ − 𝑘̂. Find a vector 𝑑⃗ which is perpendicular to both 𝑐⃗ and 𝑏⃗⃗ and
𝑑⃗. 𝑎⃗ = 21.(Ans:( i-16j-13k) /3)
2𝑖−2𝑗−𝑘
14. Find a unit vector perpendicular to each of the vector 𝑎⃗+𝑏⃗⃗ and 𝑎⃗-𝑏⃗⃗, ,where 𝑎⃗=3i+2j+2k and 𝑏⃗⃗=i+2j-2k.(Ans: 3 )
15. If two vectors a and 𝑏⃗⃗ are such that |𝑎⃗|=2, |𝑏⃗⃗|=1 and 𝑎⃗. 𝑏⃗⃗=1 then find (3𝑎⃗-5𝑏⃗⃗).(2𝑎⃗+7𝑏⃗⃗).(Ans:0)
16. If vectors 𝑎⃗= 2i+2j+3k , 𝑏⃗⃗=-i+2j+k and 𝑐⃗ = 3i+j are such that 𝑎⃗+λ𝑏⃗⃗ is perpendicular to 𝑐⃗, then
find the value of λ.(Ans:8)
√61
17. Using vectors find the area of triangle with vertices A(1,1,2) ,B(2,3,5) and C(1,5,5).(Ans: sq. units)
2
18. If 𝑎⃗,b,c are three vectors such that |𝑎⃗|=3, |𝑏⃗⃗|=4 and |𝑐⃗|=5 and each one of these is perpendicular to the sum of other
two, find |𝑎⃗ + 𝑏⃗⃗ + 𝑐⃗|.(Ans:5√2)
19. The scalar product of the vector 𝑎⃗ = 𝑖̂ + 𝑗̂ + 𝑘̂with a unit vector along the sum of vectors 𝑏⃗⃗ = 2𝑖̂ + 4𝑗̂ − 5𝑘̂ and 𝑐⃗ =
𝜆𝑖̂ + 2𝑗̂ + 3𝑘̂ is equal to one. Find the value of 𝜆 and hence find the unit vector along 𝑏⃗⃗ + 𝑐⃗.
̂
3𝑖̂+6𝑗̂ −2𝑘
[Ans:- 1, ]
7
20. Find 𝜆, if 𝑎⃗ = 𝑖̂ + 3𝑗̂ + 𝑘̂ , 𝑏⃗⃗ = 2𝑖̂ − 𝑗̂ − 𝑘̂ and 𝑐⃗ = 𝜆𝑗̂ + 3𝑘̂ are coplanar. [Ans: 7]
21. If 𝑎⃗ 𝑋 𝑏⃗⃗ =𝑐⃗ x 𝑑⃗ and 𝑎⃗ x 𝑐⃗ =𝑏⃗⃗ x d .show that 𝑎-d is parallel to b-c, where a≠d and b ≠c.
22. Three vectors 𝑎⃗ , 𝑏⃗⃗ and 𝑐⃗ satisfy the condition 𝑎⃗+𝑏⃗⃗+𝑐⃗=0.Find the value of 𝑎⃗. 𝑏⃗⃗+𝑏⃗⃗. 𝑐⃗+ 𝑐⃗. 𝑎⃗, if |𝑎⃗|=1, |𝑏⃗⃗|=4 and
−21
|𝑐⃗|=2.(Ans: ).
2
23. Prove that, for any three vectors 𝑎⃗, 𝑏⃗⃗ and 𝑐⃗ . Prove that [ 𝑎⃗ + 𝑏⃗⃗
𝑏⃗⃗ + 𝑐⃗⃗⃗ 𝑐⃗⃗⃗ + 𝑎⃗ ] = 2 [𝑎⃗ 𝑏⃗⃗ 𝑐⃗].
24. Show that the points A, B and C with position vectors, 𝑎⃗ =3i -4k-4j, 𝑏⃗⃗ = 2i – j +k and 𝑐⃗ = I -3j -5k, respectively form
the vertices of a right angled triangle.
25. Show that the vectors 2i – j +k, i -3j-5k and 3i -4j -4k form the vertices of a right angled triangle.
26. If the vertices A, B and C of a triangle ABC are (1, 2, 3) , (-1, 0, 0) , (0, 1, 2) respectively, then find the <ABC.
10
(Ans: cos-1 )
√102
27. Find a unit vector perpendicular to each of the vectors (𝑎⃗ + 𝑏⃗⃗) and (𝑎⃗ - 𝑏⃗⃗) , where 𝑎⃗= i + j + k and 𝑏⃗⃗ = i + 2j +3k.
−𝑖+2𝑗−𝑘
(Ans: )
√6
√21
28. Find the area of a triangle having the points A(1, 1, 1), B(1, 2, 3) and C(2, 3, 1) as its vertices.(Ans: )
2
29. The two adjacent sides of a parallelogram are 2i -4j +5k and i -2j -3k . Find the unit vector parallel to its diagonal.
3𝑖−6𝑗+2𝑘
Also find its area. (Ans: and 11 √5)
7
30. Find a vector of magnitude 5 units, and parallel to the resultant of the vectors 𝑎⃗ = 2𝑖 + 3𝑗 − 𝑘, 𝑏⃗⃗= i - 2j + k.
3√10𝑖 √10𝑗
(Ans: + )
2 2
31. Show that the four points A,B,C and D with PV 4𝑖̂ + 5𝑗̂ + 𝑘̂ , −𝑗̂ − 𝑘̂ ,3𝑖̂ + 9𝑗̂ + 4𝑘̂ and 4(−𝑖̂ + 𝑗̂ + 𝑘̂) are coplanar.
32. Let 𝑎⃗ = 𝑖̂ + 4𝑗̂ + 2𝑘̂, 𝑏⃗⃗ = 3𝑖̂ − 2𝑗̂ + 7𝑘̂ and 𝑐⃗ = 2𝑖̂ − 𝑗̂ + 4𝑘̂. Find a vector 𝑃⃗⃗ which is perpendicular to both 𝑎⃗ and 𝑏⃗⃗
and 𝑃⃗⃗ . 𝑐⃗ = 18.[Ans: 64𝑖̂ − 2𝑗̂ − 28𝑘̂]
33. Find the value of λ ,if four points with PV 3𝑖̂ + 6𝑗̂ + 9𝑘̂, 𝑖̂ + 2𝑗̂ + 3𝑘̂, 2𝑖̂ + 3𝑗̂ + 𝑘̂ and 4𝑖̂ + 6𝑗̂ + 𝜆𝑘̂ are coplanar.
34. In triangle ABC (Fig.), which of the following is not true:
⃗⃗⃗⃗⃗⃗⃗ + 𝑩𝑪
(a)𝑨𝑩 ⃗⃗⃗⃗⃗⃗⃗ + 𝑪𝑨
⃗⃗⃗⃗⃗⃗ = 0 (b) 𝑨𝑩
⃗⃗⃗⃗⃗⃗⃗ + 𝑩𝑪
⃗⃗⃗⃗⃗⃗⃗ - 𝑨𝑪
⃗⃗⃗⃗⃗⃗ = 0(c) 𝑨𝑩
⃗⃗⃗⃗⃗⃗⃗ + 𝑩𝑪
⃗⃗⃗⃗⃗⃗⃗ + 𝑪𝑨
⃗⃗⃗⃗⃗⃗ = 0(d) 𝑨𝑩
⃗⃗⃗⃗⃗⃗⃗ - 𝑪𝑩
⃗⃗⃗⃗⃗⃗⃗ + 𝑪𝑨
⃗⃗⃗⃗⃗⃗ = 0
⃗⃗ and ⃗𝒃⃗ are two collinear vectors, then which of the following are incorrect:
35. If 𝒂
⃗⃗ = 𝝀𝒂
(a)𝒃 ⃗⃗ for some scaler λ. ⃗⃗ = ± ⃗𝒃⃗
(b)𝒂
(c) the respective components of 𝒂 ⃗⃗ are not proportional.
⃗⃗ and 𝒃
⃗⃗ and ⃗𝒃⃗ have same direction, but different magnitudes.
(d) both the vectors 𝒂
⃗⃗ is a nonzero vector of magnitude ‘a’ and  a nonzero scalar, then λ𝒂
39. If 𝒂 ⃗⃗ is unit vector if
𝟏
(a)λ =1 (b)λ = -1 (c) a = |𝝀| (d) a = |𝝀| .
𝟐
40. Let the vectors 𝒂 ⃗⃗ be such that |𝒂
⃗⃗ and 𝒃 ⃗⃗| =
⃗⃗| = 3 and |𝒃 ⃗⃗ is a unit vector, if the angle between 𝒂
⃗⃗ x 𝒃
, then 𝒂 ⃗⃗ is
⃗⃗ and 𝒃
√𝟑
𝝅 𝝅 𝝅 𝝅
(a) 𝟔 (b) 𝟒 (c) 𝟑 (d) 𝟐
𝟏 𝟏 𝟏
̂ , 𝒊̂ + 𝒋̂ + 4𝒌
41. Area of a rectangle having vertices A, B, C and D with position vectors -𝒊̂ + 𝒋̂ + 4𝒌 ̂ , 𝒊̂ - 𝒋̂ + 4𝒌
̂
𝟐 𝟐 𝟐
𝟏
̂ respectively is
And -𝒊̂ - 𝒋̂ + 4𝒌
𝟐
(a)1/2 (b)1 (c)2 (d)4
⃗⃗
⃗⃗ and 𝒃 , then 𝒂
42. If 𝜽 is the angle between two vectors 𝒂 ⃗⃗
⃗⃗ . 𝒃 ≥ 0 only when
𝝅 𝝅
(a) 0 < 𝜽 < (b) 0 ≤ 𝜽 ≤ (c) 0 < 𝜽 < 𝝅 (d) ) 0 ≤ 𝜽 < 𝝅
𝟐 𝟐
⃗⃗ and ⃗𝒃⃗ be two unit vectors and 𝜽 is the angle between them. Then 𝒂
43. Let 𝒂 ⃗⃗ + ⃗𝒃⃗ is a unit vector if
𝝅 𝝅 𝝅 𝟐𝝅
(a) 𝜽 = 𝟒 (b) 𝜽 = 𝟑 (c) 𝜽 = 𝟐 (d) 𝜽 = 𝟑
̂ ) + 𝒋̂ . (𝒊̂ 𝑿 𝒌
44. The value of 𝒊̂ . (𝒋̂ 𝑿 𝒌 ̂)+𝒌
̂ . (𝒊̂ 𝑿 𝒋̂ ) is
(a) 0 (b)-1 (c)1 (d)3
45. If 𝜽 is the angle between any two vectors 𝒂 ⃗⃗
⃗⃗ and 𝒃 then |𝒂 ⃗⃗
⃗⃗ . 𝒃| = |𝒂 ⃗⃗
⃗⃗ 𝑿 𝒃| when 𝜽 is equal to
𝝅 𝝅
(a)0 (b) 𝟒 (c) 𝟐 (d)𝝅
THREE DIMENSIONAL GEOMETRY
1. Find the Cartesian equation of the line which passes through the point (-2,4,-5) and is parallel to the line
𝑥+3 4−𝑦 𝑧+8 𝑥+2 𝑦−4 𝑧+5
= = .(Ans: = −5 = ).
3 5 6 3 6
2. Show that the lines 𝑟⃗=3i+2j-4k+λ(i+2j+2k) and 𝑟⃗=5i-2j +𝜇(3i+2j+6k) are intersecting . Hence find their point of
intersection.(Ans: (-1,-6,-12))
3. Find the vector and Cartesian equations of the line passing through point (1,2,-4) and perpendicular to the two lines
𝑥−8 𝑦+19 𝑧−10 𝑥−15 𝑦−29 𝑧−5
= −16 = and = = −5 .(Ans: 𝑟⃗=i+2j-4k+λ(2i+3j+6k)).
3 7 3 8
–𝑥+2 𝑦−1 𝑧+3 𝑥+2 2𝑦−8 𝑧−5
4. Find the angle between following pair of lines = = −3 and = = and check whether the lines are
−2 7 −1 4 4
𝜋
parallel or perpendicular.(Ans: 2 ).
5. Find the shortest distance between the lines 𝑟⃗ = (1 − 𝑡)𝑖̂ + (𝑡 − 2)𝑗̂ + (3 − 2𝑡)𝑘̂ and
8
𝑟⃗ = (𝑠 + 1)𝑖̂ + (2𝑠 − 1)𝑗̂ − (2𝑠 + 1)𝑘̂.(Ans: ).
√29
6. Find the shortest distance between the lines whose vector equations are: 𝑟⃗=i+2j+3k+λ(2i+3j+4k) and
5
𝑟⃗=2i+4j+5k+𝜇(4i+6j+8k).(Ans:√29).
7. Find the shortest distance between the lines 𝑟⃗ = 4𝑖̂ − 𝑗̂ + 𝜆(𝑖̂ + 2𝑗̂ − 3𝑘̂) and
𝑟⃗ = 𝑖̂ − 𝑗̂ + 2𝑘̂ + 𝜇(2𝑖̂ + 4𝑗̂ − 5𝑘̂ ).
𝑥−1 𝑦+1 𝑧−1 𝑥+2 𝑦−1 𝑧+1 107
8. Find the shortest distance between the lines : = = and = = .(Ans: ).
3 2 5 4 3 −2 √1038
𝑥 𝑦−2 𝑧−3
9. Find the equation of the perpendicular from point (3,-1,11) to the line 2= = .Also find the co-ordinates of foot
3 4
𝑥−3 𝑦+1 𝑧−7
of perpendicular and the length of perpendicular.(Ans:√53, = = ,(2,5,7)).
−1 6 −4
𝑥+2 𝑦+1 𝑧−3
10. Find the points on the line = = at a distance of 5 units from the point P(1,3,3).
3 2 2
(Ans: (-2,-1,3) and (4,3,7)).
1−𝑥 7𝑦−14 5𝑧−10 7−7𝑥 𝑦−5 6−𝑧
11. Find the value of λ , so that = = and = = are perpendicular to each other.(Ans: 7)
3 2𝜆 11 3𝜆 1 5
𝑥−4 𝑦+3 𝑧+1 𝑥−1 𝑦+1 𝑧+10
12. Show that the lines = = and = = intersect and find the co-ordinate of their point of
1 −4 7 2 −3 8
intersection. (Ans(5,-7,6).
𝑥+1 𝑦+3 𝑧+5 𝑥−2 𝑦−4 𝑧−6
13. Show that the lines = = and = = intersect. Find intersection point.
3 5 7 1 3 5
14. Find the angle between the following pair of lines:
3
𝑟⃗=(3+λ)i+(1-λ)j-(2+2λ)k and 𝑟⃗=(2+3𝜇)i-(1-5𝜇)j-(56+4𝜇)k.(Ans:cos-15√3).
15. Find whether the lines 𝑟⃗=(i-j-k)+λ(2i+j) and 𝑟⃗=2i-j +𝜇(i+j-k) intersect or not.If intersecting ,find their point of
intersection.(Ans:(3,0,-1)).
16. Find the equation of line passing through points A(0,6,-9) and B(-3,-6,3).If D is the foot of perpendicular drawn from
the point C(7,4,-1) on the line AB, then find the co-ordinates of the point D and equation of line CD.
7−𝑥 −𝑥−2
(Ans:D(-1,2,-5) and = 4−𝑦 = ).
4 2
17. Find the vector equation of the line passing through the point (1,2,-4) and perpendicular to the two line :
𝑥−8 𝑦+19 𝑧−10 𝑥−15 𝑦−29 𝑧−5
= = 𝑎𝑛𝑑 = = .(Ans: 𝑟⃗=𝑟⃗=i+2j-4k+λ(2i+3j+6k)).
3 −16 7 3 8 −5
𝑥+5 𝑦+3 𝑧−6
18. Find the equation of the perpendicular drawn from the point P(2,4,-1) to the line = = .
1 4 −9
𝑥−2 𝑦−4 𝑧+1
(Ans: = = ).
6 3 2
𝑥+5 𝑦+3 𝑧−6
19. Find the equation of the perpendicular drawn from the point P(2,4,-1) to the line = = .
1 4 −9
Also, Write down the co-ordinates of the foot of the perpendicular from P to the line. (Ans:--4,1,-3).
𝑥+3 𝑦−1 𝑧+4
20. Find the foot of the perpendicular from the point (0,2,3) on the line = = .Also find the length of
5 2 3
perpendicular. (Ans:(2,3,-1) and √21).
3 −2 8
21. Find the direction cosines of the line passing through the two points (-2,4,-5) and (1,2,3). (Ans: , , ).
√77 √77 √77
22. If the line through the points (4,1,2) and (5,λ,0) is parallel to the line through the points(2,1,1) and (3,3,-1) .
Find λ.(Ans:3)
LINEAR PROGRAMMING
1. Solve the following LPP graphically:
Minimise of Z = 5x +10y
Subject to constraints x + 2 y ≤ 120 , x+y ≥ 60, x- 2y ≥ 0 and x, y ≥ 0(Ans: 300 at (60, 0)
2. Maximize and Minimize Z = x +2 y
Subject to the constraints x + 2y ≥ 100, 2x – y ≤ 0, 2x + y ≤ 200 and x, y ≥ 0.
(Ans: 400 at (0, 200) and 100 at (0, 50) and (20, 40)).
3. Solve the following linear programming problem graphically:
Minimise Z = 200 x + 500 y
subject to the constraints:
x + 2y ≥ 10 , 3x + 4y ≤ 24 , x ≥0, y ≥ 0 (Ans:- 2300 at the point (4, 3))
4. Solve the following problem graphically:
Minimise and Maximise Z = 3x + 9y
subject to the constraints: x + 3y ≤ 60
x + y ≥ 10 , x ≤ y , x ≥0, y ≥ 0
5. Solve the following LPP graphically:
Minimise Z = – 3x + 4 y , subject to x + 2y ≤ 8, 3x + 2y ≤ 12, x ≥0, y ≥ 0
6. Solve the following LPP graphically:
Minimise Z = x + 2y , subject to 2x + y ≥ 3, x + 2y ≥ 6, x, y ≥ 0.
7. Show that the minimum of Z occurs at more than two points.
Minimise and Maximise Z = 5x + 10 y , subject to x + 2y ≤ 120, x + y ≥ 60, x – 2y ≥ 0, x, y ≥ 0

PROBABILITY
1. A speaks truth 75% of the cases, while B in 90% cases. In what percent of cases are they likely to contradict each
other in stating the same fact?Do you think that statement of B is true.(Ans: 30% B is false).
3 5
2. The probabilities of two students A and B coming to the school in time are 7 and 7 respectively. Assuming that the
events ‘A’ coming in time and ‘B’ coming in time are independent, find the probability of only one of them coming
26
to the school in time.(Ans : 49)
3. In a hockey match both teams A and B scored same number of goals up to the end of the game, so to decide the
winner the referee asked both the captains to throw a die alternately and decided that the team, whose captain gets a
six first, will be declared the winner.If the captain of the team A was asked to start , find their respective probabilities
5
of winning the match and start whether the decision of the referee was fair or not.(Ans:11)
4. How many times must a man toss a fair coin, so that the probability of having atleast one head is more than
80%.(Ans:3 or more time)
1 1
5. Probabilities of solving a specific problem independently by A and B are 2 and 3 respectively.If both try to solve
problem independently, find the probability that (a)Problem is solved(b)Exactly one of them solves the
problem.(Ans:2/3 and 1/2)
6. A family has 2 children.Find the probability that both are boys, if it is known that (a)Atleast one of children is
boy.(b)Elder child is a boy.(Ans:1/3 and 1/2)
7. On a multiple choice examination with three possible answers (Out of which only one is correct)for each of the five
questions, what is the probability that a candidate would get four or more correct answers just by
guessing?(Ans:11/243)
8. A die is thrown again and again until three sixes are obtained. Find the probability of obtaining third six in the sixth
throw of die.(Ans:625/23328)
9. Assume that the chances of a patient having a heart attack are 40%. It is also assumed that a meditation and yoga
course reduce the risk of heart attack by 30% and prescription of certain drug reduces its chances by 25%. At a time a
patient can choose any one of the two options with equal probabilities. It is given that after going through one of the
two options the patient selected at random suffers a heart attack. Find the probability that the patient followed a
course of meditation and yoga?(Ans: 14/29)
10. Two cards are drawn simultaneously (Without replacement)from a well shuffled deck of 52 cards.Find the mean and
variance of the number of red cards.(Ans:1 and 25/51)
11. Find the mean number of heads in three tosses of a coin.(Ans:3/2)
12. A random variable X has the following probability distribution:
X 0 1 2 3 4 5 6 7
P(X) 0 k 2k 2k 3k K2 2k2 7k2+k
Determine (i) k (ii) P(X < 3) (iii) P(X > 6) (iv) P(0 < X < 3).(Ans:1/10,3/10,17/100 and 3/10).
13. Find the probability distribution of number of doublets in three tosses of a pair of dice.
(Ans:
X 0 1 2 3
P(X) 125/216 75/216 15/216 1/216
14. From a lot of 30 bulbs which include 6 defective, a sample of 4 bulbs is drawn at random with replacement. Find the
probability distribution of the number of defective bulbs.
Ans:
X 0 1 2 3 4
P(X) 256/625 256/625 96/625 16/625 1/625

15. Two cards are drawn successively with replacement from a well shuffled deck of 52 cards.Find the probability
distribution of number of aces.
(Ans:
X 0 1 2
P(X) 144/169 24/169 1/169
16. Of the students in a college, it is known that 60% reside in hostel and 40% are day scholars (not residing in hostel).
Previous year results report that 30% of all students who reside in hostel attain A grade and 20% of day scholars
attain A grade in their annual examination. At the end of the year, one student is chosen at random from the college
and he has an A grade, what is the probability that the student is a hostlier?(Ans:9/13)
17. Suppose a girl throws a die. If she gets a 5 or 6, she tosses a coin three times and notes the number of heads. If she
gets 1, 2, 3 or 4, she tosses a coin once and notes whether a head or tail is obtained. If she obtained exactly one head,
what is the probability that she threw 1, 2, 3 or 4 with the die?(Ans:8/11)
18. Suppose that 5% of men and 0.25% of women have grey hair. A grey haired person is selected at random. What is the
probability of this person being male? Assume that there are equal number of males and females.(Ans:20/21)
19. Bag I contains 3 red and 4 black balls while another Bag II contains 5 red and 6 black balls. One ball is drawn at
random from one of the bags and it is found to be red. Find the probability that it was drawn from Bag II.(Ans:35/68)
20. A man is known to speak truth 3 out of 4 times. He throws a die and reports that it is a six. Find the probability that it
is actually a six.(Ans:3/8)
21. A factory has two machines A and B. Past record shows that machine A produced 60% of the items of output and
machine B produced 40% of the items. Further, 2% of the items produced by machine A and 1% produced by
machine B were defective. All the items are put into one stockpile and then one item is chosen at random from this
and is found to be defective. What is the probability that it was produced by machine B?(Ans:1/4)
22. There are three coins. One is a two headed coin (having head on both faces), another is a biased coin that comes up
heads 75% of the time and third is an unbiased coin. One of the three coins is chosen at random and tossed, it shows
heads, what is the probability that it was the two headed coin ?(Ans:4/9)
23. Given three identical boxes I, II and III, each containing two coins. In box I, both coins are gold coins, in box II, both
are silver coins and in the box III, there is one gold and one silver coin. A person chooses a box at random and takes
out a coin. If the coin is of gold, what is the probability that the other coin in the box is also of gold?(Ans:2/3)
24. In a class 5% of boys and 10% of girls have an IQ of more than 150.In the class 60% of boys and rest girls.If a
student is selected at random and found to have an IQ of more than 150.Find the probability that the student is a
boy.(Ans:3/7)
25. In a factory which manufactures bolts, machines A, B and C manufacture respectively 25%, 35% and 40% of the
bolts. Of their outputs, 5, 4 and 2 percent are respectively defective bolts. A bolt is drawn at random from the product
and is found to be defective. What is the probability that it is manufactured by the machine B?(Ans:28/69)
26. A card from a pack of 52 cards is lost. From the remaining cards of the pack, two cards are drawn and are found to be
both diamonds. Find the probability of the lost card being a diamond.(Ans:11/50)
27. An insurance company insured 2000 cyclists,4000 scooter drivers and 6000 motorbike drivers.The probability of an
accident involving a cyclist,scooter driver and a motorbike driver are 0.01,0.03 and 0.15 respectively.One of the
insurance persons meets with an accident.Ehat is the probability that he is a scooter driver.Which mode of transport
would you suggest to a student and why?(Ans:6/52.)
28. A doctor is to visit a patient. From the past experience, it is known that the probabilities that he will come by train,
3 1 1 2
bus, scooter or by other means of transport are respectively 10 , 5 , 10 , and 5 . The probabilities that he will be late are
1 1 1
, 3 , 𝑎𝑛𝑑 , if he comes by train, bus and scooter respectively, but if he comes by other means of transport, then he
4 12
will not be late. When he arrives, he is late. What is the probability that he comes by train?(Ans:1/2)
29. Two groups are competing for the position on the Board of directors of a corporation. The probabilities that the first
and the second groups will win are 0.6 and 0.4 respectively. Further, if the first group wins, the probability of
introducing a new product is 0.7 and the corresponding probability is 0.3 if the second group wins. Find the
probability that the new product introduced was by the second group.(Ans:2/9).
30. Bag I contains 3 red and 4 black balls and bag II contains 4 red and 5 black balls. One ball is transferred from bag I to
bag II and then a ball is drawn from bag II. The ball so drawn is found to be red in colour. Find the probability that
the transferred ball is black.(Ans: 16/31)
31. Two numbers are selected at random(without replacement) from the first six positive integers. Let X denote the larger
of the two numbers obtained. Find E(x).(Ans: 14/3)
32. Let a pair of dice be thrown and the random variable X be the sum of the numbers that appear on the two dice. Find
the mean or expectation of X.(Ans:- 7)
33. Find the mean number of heads in three tosses of a fair coin.
34. Two numbers are selected at random (without replacement) from the first six positive integers. Let X denote the
larger of the two numbers obtained. Find E(X).
35. A class has 15 students whose ages are 14, 17, 15, 14, 21, 17, 19, 20, 16, 18, 20, 17, 16, 19 and 20 years. One student
is selected in such a manner that each has the same chance of being chosen and the age X of the selected student is
recorded. What is the probability distribution of the random variable X? Find mean.
36. A random variable X has the following probability distribution:
X 0 1 2 3 4 5 6 7
2 2
P(X) 0 K 2K 2K 3K K 2K 7K2+ K

(a)k (b) P(X <3) (c)P(X>6) (d)P(0<X<3)


𝟏
37. If P(A) = 𝟐 , P(B) = 0, then P(A/B) is
(a)0 (b)1/2 (c)Not defined (d)1
38. If A and B are events such that P(A/B) = P(B/A), then
(a)A ∁ B but A ≠ B (b)A = B (c)A ∩ B = ∅ (d)P(A) = P(B)
39. The probability of obtaining an even prime number on each die, when a pair of dice is rolled is
(a)0 (b)1/3 (c)1/12 (d)1/36
40. Two events A and B will be independent, If
(a)A and B are mutually exclusive. (b)P(A’ B’ ) = [1 – P(A)][1-P(B)]
(c)P(A) = P(B) (d)P(A) + P(B) =1
𝟒
41. Probability that A speaks truth is 𝟓 A coin is tossed. A reports that a head appears. The probability that actually there
was head is
(a)4/5 (b)1/2 (c)1/5 (d)2/5
42. If A and B are two events such that A ∁ B and P(B) ≠ 0, then which of the following is correct?
𝑷(𝑩)
(a)P(A/B) = (b) P(A/B) < P(A) (c)P(A/B) ≥ P(A) (d)None of these
𝑷(𝑨)
43. If A and B are two events such that P(A) ≠ 0 and P(B/A) = 1, then
(a)A ∁ 𝑩 (b)B ∁ 𝑨 (b)B =∅ (d)A=∅
44. If P(A/b)>P(A) then which of the following is correct:
(a)P(B/A) < P(B) (b) P(A ∩ B ) < P(A).P(B) (c)P(B/A) > P(B) (d)P(B/A) =P(B)
45. If A and B are any two events such that P(A) + P(B) – P(A and B) = P(A), then
(a)P(B/A) =1 (b)P(A/B)=1 (c)P(B/A)=0 (d)P(A/B)=0
--------------------------------------------BEST OF LUCK-------------------------------------------------------

CBSE SAMPLE PAPER-2023-24


SECTION A
(Multiple Choice Questions)
Each question carries 1 mark
1 𝑤ℎ𝑒𝑛 𝑖 ≠ 𝑗
1. If A = [𝑎𝑖𝑗 ] is a square matrix of order 2 such that aij = { , then A2 is
0 𝑤ℎ𝑒𝑛 𝑖 = 𝑗
1 0 1 1 1 1 1 0
(a)[ ] (b) [ ] (c) [ ] (d) [ ]
1 0 0 0 1 0 0 1
2. If A and B are invertible square matrices of the same order, then which of the following is not correct?
|𝐴| 1
(a)|𝐴𝐵−1 | = |𝐵| (b) |(𝐴𝐵)−1 | = |𝐴||𝐵| (c)(𝐴𝐵)−1 = B-1A-1 (d)(A+B)-1 = B-1 + A-1
3. If the area of the triangle with vertices (-3, 0), (3, 0) and (0, k) is 9 square units, then the values of k will be
(a)9 (b)±3 (c)-9 (d)6
kx
𝑖𝑓 𝑥 < 0
4. If f(x) = {|x| is continuous at x = 0 , then the value of k is
3 if x ≥ 0
(a)−3 (b) 0 (c) 3 (d) any real number
5. The lines 𝑟⃗ = 𝑖̂ + 𝑗̂ − 𝑘̂ + 𝜆(2𝑖̂ + 3𝑗̂ − 6𝑘̂) and 𝑟⃗ = 2𝑖
̂ − 𝑗̂ − 𝑘̂ + 𝜇(6𝑖̂ + 9𝑗̂ − 18𝑘̂) (where µ and λ are scalers) are
(a) coincident (b) skew (c) intersecting (d) parallel
𝑑𝑦 𝑑2 𝑦 2
6. The degree of the differential equation [ 1 + ( )2 ]3 =( ) is
𝑑𝑥 𝑑𝑥 2
(a)4 (b)3/2 (c)2 (d)Not defined
7. The corner points of the bounded feasible region determined by a system of linear constraints are (0, 3), (1, 1) and (3, 0). Let Z
= px + qy, where p, q > 0. The condition on p and q so that the minimum of Z occurs at (3, 0) and (1, 1) is
(a)p=2q (b)p = q/2 (c)p=3q (d)p=q

8. ABCD is a rhombus whose diagonals intersect at E. Then ⃗⃗⃗⃗⃗⃗𝐸𝐴 + ⃗⃗⃗⃗⃗⃗


𝐸𝐵 + ⃗⃗⃗⃗⃗⃗
𝐸𝐶 + ⃗⃗⃗⃗⃗⃗
𝐸𝐷 equals to
⃗⃗
(𝑎)0 (b) ⃗⃗⃗⃗⃗⃗
𝐴𝐷 ⃗⃗⃗⃗⃗⃗⃗
(c) 2𝐵𝐷 (d)2 ⃗⃗⃗⃗⃗⃗
𝐴𝐷
𝜋 𝐶𝑜𝑠2 𝑥 3 (2𝑛
9. For any integer n, the value of ∫−𝜋 𝑒 𝑆𝑖𝑛 + 1)𝑥 𝑑𝑥 𝑖𝑠
(a)-1 (b)0 (c)1 (d)2
0 2𝑥 − 1 √𝑥
10. The value of |𝐴|, If A = |1 − 2𝑥 0 2√𝑥| , where x ∈ R+ is
−√𝑥 −2√𝑥 0
2
(a)(2x+1) (b)0 (c)(2x+1)3 (d)(2x-1)2
11. The feasible region corresponding to the linear constraints of a Linear Programming Problem is given
Below:

Which of the following is not a constraint to the given Linear Programming Problem?
(a) x + y ≥ 2 (b)x +2y ≤ 10 (c) x – y ≥ 1 (d) x – y ≤ 1.
̂ + 4𝑘̂ , then the vector form of the component of 𝑎⃗ along 𝑏⃗⃗ is
12. If 𝑎⃗ = 4𝑖̂ + 6𝑗̂ and 𝑏⃗⃗ = 3𝑗
18 ̂) 18 ̂) 18 ̂) 18
(a) 5 ( 3𝑖̂ + 4𝑘 (b) 25 ( 3𝑗̂ + 4𝑘 (c) 5 ( 3𝑗̂ + 4𝑘 (d) 25 ( 4𝑖̂ + 6𝑗̂ )
13. Given that A is a square matrix of order 3 and |𝐴| = -2, then |𝑎𝑑𝑗(2𝐴)| is equal to
(a)-26 (b)4 (c)-28 (d)28
1 1 1
14. A problem in Mathematics is given to three students whose chances of solving it are 2 , 3 , 4 respectively. If the events of their
solving the problem are independent then the probability that the problem will be solved, is
(a)1/4 (b)1/3 (c)1/2 (d)3/4
15. The general solution of the differential equation ydx -xdy = 0 (Given x, y > 0) is of the form
(a)xy=c (b)x=cy2 (c)y=cx (d)y=cx2
(Where 'c' is an arbitrary positive constant of integration)
16. The value of λ for which two vectors 2𝑖̂ − 𝑗̂ + 2𝑘 ̂ and 3𝑖 ̂ + 𝜆𝑗̂ + 𝑘̂ are perpendicular is
(a)2 (b) 4 (c) 6 (d) 8
17. The set of all points where the function f(x) = x+ |𝑥| is differential, is
(a)(0, ∞ ) (b) (-∞, 0) ) (c)(-∞ , 0) U (0 , ∞) (d)(-∞ , ∞)
1 1 1
18. If the direction cosines of a line are < 𝑐 , 𝑐 , 𝑐 > then
(a)0<c<1 (b)c>2 (c)c= ± √2 (d) c= ±√3
ASSERTION-REASON BASED QUESTIONS
In the following questions, a statement of Assertion (A) is followed by a statement of Reason (R).
Choose the correct answer out of the following choices.
(a) Both (A) and (R) are true and (R) is the correct explanation of (A).
(b) Both (A) and (R) are true but (R) is not the correct explanation of (A).
(c) (A) is true but (R) is false.
(d) (A) is false but (R) is true.

𝑑
19. Let f(x) be a polynomial function of degree 6 such that (𝑓(𝑥)) = (x-1)3(x-3)2, then
𝑑𝑥
ASSERTION(A): f(x) has minimum at x=1
𝑑 𝑑
REASON(R): When 𝑑𝑥 (f(x)) < 0 for every x ∈ (a -h, a) and 𝑑𝑥 (f(x)) > 0, for every x ∈ (a, a+h); where ‘h’ is an infinitesimally
small positive quantity then f(x) has minimum at x= a, provided f(x) is continuous at x=a.
20. ASSERTION(A): The relation f: {1,2,3,4} → {x,y,z,p} defined by f= {(1,x),(2,y),(3,z)} is a bijective function.
REASON(R): The relation f: {1,2,3} → {x,y,z,p} defined by f= {(1,x),(2,y),(3,z)} is one-one.

Section –B
[This section comprises of very short answer type questions (VSA) of 2 marks each]
33𝜋
21. Find the value of sin-1(cos( )).
5
OR
Find the domain of sin-1(x2 – 4)
22. Find the interval/s in which the function f: R→ R defined by f(x) =xex is increasing.
1
23. If f(x) = 4𝑥 2+2𝑥+1 ; x ∈ R then find the maximum value of f(x).
OR
Find the maximum profit that a company can make, if the profit function is given by P(x) = 72 +42x -x2, where X is the number
of units and P is the profit in rupees.
1 (2−𝑥)
24. Evaluate : ∫−1 log (2+𝑥) dx.
25. Check whether the function f: R → R defined by f(x) = x3 + x, has critical points or not?If yes then find the points.

Section – C
[This section comprises of short answer type questions (SA) of 3 marks each]
2𝑥 2 +3
26. Find:- ∫ 𝑥 2 (𝑥2 +9)dx
27. The random variable X has a probability distribution P(X) of the following form, where 'k ' is some real number:
𝑘, 𝑖𝑓 𝑥 = 0
𝑃(𝑋) = {2𝑘, 𝑖𝑓 𝑥 = 1 and 0 otherwise.
3𝑘, 𝑖𝑓 𝑥 = 2
(i) Determine the value of k. (ii) Find P(X<2) (iii)P(X>2)
𝑥
28. Find: ∫ √ 𝑑𝑥.
1−𝑥 3
OR
𝜋
Evaluate ∫04 log(1 + 𝑡𝑎𝑛𝑥) 𝑑𝑥
𝑥 𝑥
29. Solve the D.E 𝑦𝑒 𝑦 dx = (𝑥𝑒 𝑦 + y2)dy
OR
𝑑𝑦
Solve the D.E (cos2x) 𝑑𝑥 + y = tanx
30. Solve the following Linear Programming Problem graphically:
Minimize: Z = x +2y
subject to the constraints: x + 2y ≥ 100, 2x – y ≤ 0 , 2x + y ≤ 200 , x, y ≥0
OR
Solve the following Linear Programming Problem graphically:
Minimize: Z = -x +2y
subject to the constraints: x ≥ 3, x + y ≥5 , x + 2y ≤ 6 , y ≥0
𝑦
𝑑2 𝑦 𝑎
31. If (a+ bx)𝑒 𝑥 = x then prove that x 𝑑𝑥 2 = ( 𝑎+𝑏𝑥
)2
Section –D
[This section comprises of long answer type questions (LA) of 5 marks each]
32. Make a rough sketch of the region { (x, y) : 0 ≤ y ≤ x2 + 1, 0 ≤ x + 1, 0 ≤ x ≤ 2 } and find the area of the region using the
method of integration.
33. Let N be the set of all natural numbers and R be a relation on N x N defined by (a, b) R (c, d) ↔ ad =bc for all
(a, b), (c, d) ∈ N x N . Show that R is an equivalence relation on N x N. Also find the equivalence class of (2, 6) , i.e [(2, 6)].
OR
𝑥
Show that the function f: R → { x ∈ R : -1 < x < 1} defined by f(x) = 1+ |𝑥| , x ∈ R is one-one and onto function.
34. Using the matrix method, solve the following system of linear equations :
2 3 10 4 6 5 6 9 20
𝑥
+ 𝑦 + 𝑧 = 4 , 𝑥 − 𝑦 + 𝑧 = 1, 𝑥 + 𝑦 − 𝑧 = 2.
35. Find the coordinates of the image of the point (1, 6, 3) with respect to the line
𝑟⃗ = 𝑗̂ + 2𝑘̂ + 𝜆(𝑖̂ + 2𝑗̂ + 3𝑘̂ ) where λ is a scaler. Also find the distance of the image from the y-axis.

𝑂𝑅
̂ ) where λ is a scalar and another aero plane is flying along the line 𝑟⃗
An aeroplane is flying along the line 𝑟⃗ = 𝜆(𝑖̂ − 𝑗̂ + 𝑘
= 𝑖̂ − 𝑗̂ + µ(−2𝑗̂ + 𝑘̂ ) where µ is a scalar. At what points on the lines should they reach so that the distance between
them is the shortest? Find the shortest possible distance between them.
Section –E
[This section comprises of 3 case- study/passage based questions of 4 marks each with sub parts.
The first two case study questions have three sub parts (i), (ii), (iii) of marks 1,1,2 respectively.
The third case study question has two sub parts of 2 marks each.)

36. Read the following passage and answer the questions given below:
In an Office three employees Jayant, Sonia and Oliver process incoming copies of a certain form. Jayant processes 50%of the forms,
Sonia processes 20% and Oliver the remaining 30% of the forms. Jayant has an error rate of 0.06 , Sonia has an error rate of 0.04 and
Oliver has an error rate of 0.03 .
Based on the above information, answer the following questions.

(i) Find the probability that Sonia processed the form and committed an error.
(ii) Find the total probability of committing an error in processing the form.
(iii) The manager of the Company wants to do a quality check. During inspection, he selects a form at random from the days output of
processed form. If the form selected at random has an error, find themprobability that the form is not processed by Jayant.
OR
(iii) Let E be the event of committing an error in processing the form and let E1 , E2 and E3 be the events that Jayant, Sonia and Oliver
processed the form. Find the value of ∑3𝑖=1 𝑃(𝐸𝑖 /𝐸).
37. Read the following passage and answer the questions given below:
Teams A, B,C went for playing a tug of war game. Teams A,B,C have attached a rope to a metal ring and is trying to pull the ring
into their own area.
Team A pulls with force F1 = 6𝑖̂ + 0𝑗̂ 𝑘𝑁
Team B pulls with force F2 =-4 𝑖̂ + 4𝑗̂𝑘𝑁
Team C pulls with force F3 =-3 𝑖̂ − 3𝑗̂𝑘𝑁

(i) What is the magnitude of the force of Team A ?


(ii) Which team will win the game?
(iii) Find the magnitude of the resultant force exerted by the teams.
OR
(iii) In what direction is the ring getting pulled?
38. Read the following passage and answer the questions given below:
The relation between the height of the plant (‘y ‘ cm) with respect to its exposure to the sunlight is governed by the following
1
equation y = 4x - 2 𝑥 2 where x is the number of days exposed to the sunlight for x ≤3.
(i) Find the rate of growth of the plant with respect to the number of days exposed to the sunlight.
(ii) Does the rate of growth of the plant increase or decrease in the first three days? What will be the height of the plant after 2
days?
CBSE BOARD QUESTION PAPER-2023
SECTION A
This section comprises multiple choice questions (MCQs) of 1 mark each.
1. If for a square matrix A, A2 – 3A + I = 0 and A-1 = xA + yI then the value of x +y is
(a)-2 (b)2 (c)3 (d)-3
−1
2. If |𝐴| = 2, where A is a 2 X 2 matrix then |4𝐴 | equals to
1
(a)4 (b)2 (c)8 (d)32
3. Let A be a 3 3 matrix such that |𝑎𝑑𝑗 𝐴|= 64. Then |𝐴| is equal to
(a)8 only (b)-8 only (c) 64 (d) 8 and -8
3 4
4. If A = [ ] and 2A + B is a null matrix, then B is equal to
5 2
6 8 −6 −8 5 8 −5 −8
(a)[ ] (b) [ ] (c) [ ] (d) [ ]
10 4 −10 −4 10 3 −10 −3
𝑑
5. If 𝑑𝑥 (f(x)) = logx , then f(x) equals to
1 1
(a)-𝑥 + C (b)x(logx-1) + C (c)x(logx + x) (d)𝑥 + C
𝜋
𝜋
6. ∫06 𝑆𝑒𝑐 2 ( 𝑥 − ) dx is equal to :
6
1 1
(a) (b)- (c)√3 (d) -√3
√3 √3
7. The sum of the order and the degree of the differential equation
𝑑2 𝑦 𝑑𝑦
𝑑𝑥 2
+ (𝑑𝑥 )3 = siny is:(𝑎)5 (𝑏)2 (𝑐)3 (𝑑)4
8. The value of p for which the vectors 2𝑖̂ + 𝑝𝑗̂ + 𝑘̂ and −4𝑖̂ − 6𝑗̂ + 26𝑘̂ are perpendicular to each other is:
(a)3 (b)-3 (c)-17/3 (d)17/3
9. The value of (𝑖̂ X 𝑗̂ ) . 𝑗̂ + (𝑗̂ X 𝑖̂ ) . 𝑘̂ is
(a)2 (b)0 (c)1 (d)-1
10. If 𝑎⃗ + 𝑏⃗⃗ = 𝑖̂ and 𝑎⃗ = 2𝑖̂ − 2𝑗̂ + 2𝑘̂ then |𝑏⃗⃗| equals to
(a)√14 (b)3 (c)√12 (d)√17
𝑥−1 1−𝑦 2𝑧−1
11. Direction cosines of the line = = are:
2 3 12
2 3 6 2 −3 12 2 −3 −6 2 3 6
(a)7 , 7 , 7 (b) , , (c) 7 , , (d) 7 , − 7 , 7
√157 √157 √157 7 7
12. P(A/B) = 0.3, P(A)=0.4 and P(B)=0.8, then P(B/A) Is equals to :
(a)0.6 (b)0.3 (c)0.06 (d)0.4
3𝑥 + 5 𝑥 ≥ 2
13. The value of k for which f(x) = { is continuous function is
𝑘𝑥 2 𝑥 < 2
(a)-11/4 (b)4/11 (c)11 (d)11/4
0 1
14. If A = [ ] and (3I+ 4A)(3I-4A) = x2I then the value of x is/are:
−1 0
(a)±√7 (b)0(c)±5(d)25
15. The general solution of the differential equation x dy – ( 1+ x2) dx = dx is :-
𝑥3 𝑥3 𝑥2 𝑥2
(a) Y = 2x + +C (b) Y = 2logx + +C (c) Y = + C (d) Y = 2logx + +C
3 3 2 2
16. If f(x) – a(x- cosx) is strictly decreasing in R then a belongs to
(a){0} (b)(0, ∞) (c)(-∞ ,0) (d)(-∞, ∞)
17. The corner points of the feasible region in the graphical representation of a linear programming problem are (2, 72),
(15, 20) and (40, 15). If z = 18x + 9y be the objective function, then :
(a) z is maximum at (2, 72), minimum at (15, 20) (b) z is maximum at (15, 20), minimum at (40, 15)
(c) z is maximum at (40, 15), minimum at (15, 20) (d) z is maximum at (40, 15), minimum at (2, 72)
18. The number of corner points of the feasible region determined by the constraints
x - y ≥ 0 , 2y ≤ x+ 2, x≥ 0 , y ≥ 0 is:
(a)2 (b)3 (c)4 (d)5
Questions number 19 and 20 are Assertion and Reason based questions carrying 1 mark each. Two statements
are given, one labelled Assertion (A) and the other labelled Reason (R). Select the correct answer from the
codes (a), (b), (c) and (d) as given below.

(a) Both Assertion (A) and Reason (R) are true and Reason (R) is the correct explanation of the Assertion (A).
(b) Both Assertion (A) and Reason (R) are true, but Reason (R) is not the correct explanation of the Assertion (A).
(c) Assertion (A) is true and Reason (R) is false.
(d) Assertion (A) is false and Reason (R) is true.
3𝜋 𝜋 5𝜋
19. Assertion(A) : The range of the function f(x) = 2sin-1x + 2 , where x ∈ [-1, 1] is [ 2 , 2 ].
Reason(R): The range of the principal value branch os sin-1x is [0, 𝜋].
𝑥−3 𝑦+1 𝑧−3
20. Assertion(A): Equation of a line passing through the points (1, 2, 3) and (3, 1, 3) is 2 = 3 = 0 .
Reason(R):- Equation of a line passing through points (x1 , y1 ,z1 ), (x2 , y2 ,z2) is given by
𝑥−𝑥1 𝑦−𝑦 𝑧−𝑧
= 𝑦 −𝑦1 = 𝑧 −𝑧1 .
𝑥 −𝑥
2 1 2 1 2 1
SECTION B
This section comprises very short answer (VSA) type questions of 2 marks each.
21. A function f : A → B defined as f(x) = 2x is both one-one and onto. If A = {1, 2, 3, 4}, then find the set B.
OR
3𝜋 3𝜋
Evaluate: Sin-1 ( Sin ) + Cos-1(Cos ) + tan-1(1)
4 4
22. Find all the vectors of magnitude 3 3 which are collinear to vector 𝑖̂ + 𝑗̂ + 𝑘̂ .
23. (a)Position vectors of the points A, B and C as shown in the figure below are 𝑎⃗ , 𝑏⃗⃗ and 𝑐⃗ respectively.

5
⃗⃗⃗⃗⃗⃗ express 𝑐⃗ in terms of 𝑎⃗ and 𝑏⃗⃗ .
⃗⃗⃗⃗⃗⃗ = 𝐴𝐵
If 𝐴𝐶 4
(b) Check whether the lines given by equations x = 2λ +2, y = 7λ +1 , z = -3λ -3 and x = -µ-2, y = 2µ + 8, z = 4µ + 5 are
perpendicular to each other or not.
𝑑𝑦
24. If y = ( x + √𝑥 2 − 1 )2, then show that(x2 -1) ( 𝑑𝑥 )2 = 4y2.
16𝑠𝑖𝑛𝑥 𝜋
25. Show that the function f(x) = 4+𝑐𝑜𝑠𝑥 – x is strictly decreasing in ( 2 , 𝜋 ).
SECTION C
This section comprises short answer (SA) type questions of 3 marks each.
𝜋
26. Evaluate:- I = ∫0 [log(𝑠𝑖𝑛𝑥) − log(2𝑐𝑜𝑠𝑥)]𝑑𝑥
2

1
27. Find:- ∫ dx
√𝑥(√𝑥+1)(√𝑥+2)
𝑑𝑦
28. (a) Find the particular solution of the differential equation 𝑑𝑥 + sec2x . y = tanx . sec2x , given that y(0) = 0.
OR
(b)Solve the differential equation given by x dy – ydx - √𝑥 2 + 𝑦 2 dx = 0.
29. Solve graphically the following linear programming problem :
Maximise z = 6x + 3y,
subject to the constraints
4x + y ≥ 80 3x+2y ≤ 150 x +5y ≥ 115
x,y ≥0
30. (a) The probability distribution of a random variable X is given below :

(i) Find the value of k.


(ii) Find P( 1 ≤ X ≤ 3)
(iii) Find E(X) the mean of X.
OR
1 1
A and B are independent events such that P(A ∩ 𝐵 ) = 4 and P( 𝐴 ∩ B ) = 6. Find P(A) and P(B).
𝜋
31. Evaluate:- ∫02 𝑒 𝑥 𝑠𝑖𝑛𝑥 𝑑𝑥
OR
1
Find ∫ cos(𝑥−𝑎)cos (𝑥−𝑏)dx
SECTION D
This section comprises long answer type questions (LA) of 5 marks each.
32. A relation R is defined on a set of real numbers R as R = {(x, y) : x . y is an irrational number}. Check whether R is
reflexive, symmetric and transitive or not. As R = {(x, y) : x . y is an irrational number}. Check whether R is
reflexive, symmetric and transitive or not.

1 2 −2 3 −1 1
33. (a) If A = [−1 3 0 ] and B-1 = [−15 6 −5] , find (AB)-1.
0 −2 1 5 −2 2
OR
Solve the Solve the following system of equations by matrix method :
x + 2y + 3z = 6
2x y + z = 2
3x + 2y 2z = 3
34. Find the vector and the Cartesian equations of a line passing through the point (1, 2, 4) and parallel to the line joining
the points A(3, 3, 5) and B(1, 0, 11). Hence, find the distance between the two lines.
OR
(b) Find the equations of the line passing through the points A(1, 2, 3) and B(3, 5, 9). Hence, find the coordinates of
the points on this line which are at a distance of 14 units from point B.
35. Find the area of the region bounded by the curves x2 = y, y = x + 2 and x-axis using integration.
SECTION E
This section comprises 3 case study based questions of 4 marks each.
Case Study 1
36. There are different types of Yoga which involve the usage of different poses of Yoga Asanas, Meditation and
Pranayam as shown in the figure below :
The Venn diagram below represents the probabilities of three different types of Yoga, A, B and C performed by the
people of a society. Further, it is given that probability of a member performing type C Yoga is 0·44.

On the basis of the above information, answer the following questions :


(i) Find the value of x.
(ii) Find the value of y.
(iii) Find P(C/B)
OR
Find the that a randomly selected person of the society does Yoga of type A or B but not C.
Case Study 2
37. A tank, as shown in the figure below, formed using a combination of a cylinder and a cone, offers better drainage as
compared to a flat bottomed tank.

A tap is connected to such a tank whose conical part is full of water. Water is dripping out from a tap at the bottom at
the uniform rate of 2 cm3/s. The semi-vertical angle of the conical tank is 45 .
On the basis of given information, answer the following questions :
(i) Find the volume of water in the tank in terms of its radius r.
(ii) Find rate of change of radius at an instant when r = 2 2 cm.
(iii) (a) Find the rate at which the wet surface of the conical tank is decreasing at an instant when radius r = 2√2 cm.
OR
(iii) (b) Find the rate of change of height ‘h’ at any instant when slant heigt is 4cm.
Case Study 3
38. The equation of the path traced by a roller-coaster is given by the
polynomial f(x) = a(x + 9) (x + 1) (x 3). If the roller-coaster crosses y-axis at a point (0, 1), answer the following :

(i) Find the value of ‘a’


(ii) Find f’’(x) at x =1.
CBSE BOARD QUESTION PAPER-2023(SUPPLEMENTRY EXAM.)
SECTION A
This section comprises multiple choice questions (MCQs) of 1 mark each.
1. If A is a square matrix of order 3 and |𝐴| = 6 then the value of |adj A|
(a)6 (b)36 (c)27 (d)216
𝜋
2. The value of ∫06 𝑠𝑖𝑛3𝑥𝑑𝑥
√3 1 √3 1
(a) - (b) - 3 (c) (d) 3
2 2
3. If 𝑎⃗ and 𝑏⃗⃗ and 𝑎⃗ + 𝑏⃗⃗ are all unit vectors and 𝜃 is the angle between 𝑎⃗ and 𝑏⃗⃗ , then the value of 𝜃 is:
2𝜋 5𝜋 𝜋 𝜋
(a) 3 (b) (c) 3 (d) 6
6
4. The projection of vector 𝑖̂ on the vector 𝑖̂ + 𝑗̂ + 2𝑘̂ is:
1 2 3
(a) (b)√6 (c) (d)
√6 √6 √6
5. A family has 2 children and the elder child is a girl. The probability that both children are girls is :
1 1 1 3
(a)4 (b) 8 (c) 2 (d) 4
6. The vector equation of a line which passes through the point (2, 4, 5) and is parallel to the line
𝑥+3 4−𝑦 𝑧+8
= = is
3 2 6
(𝑎)𝑟⃗ = -2𝑖̂ + 4𝑗̂ − 5𝑘̂ + 𝜆(3𝑖̂ + 2𝑗̂ + 6𝑘̂ ) (b) 𝑟⃗ = 2𝑖̂ − 4𝑗̂ + 5𝑘̂ + 𝜆(3𝑖̂ − 2𝑗̂ + 6𝑘̂)
(𝑐)𝑟⃗ = 2𝑖̂ − 4𝑗̂ + 5𝑘̂ + 𝜆(3𝑖̂ + 2𝑗̂ + 6𝑘̂) (c)𝑟⃗ = -2𝑖̂ + 4𝑗̂ − 5𝑘̂ + 𝜆(3𝑖̂ − 2𝑗̂ − 6𝑘̂)
2𝑥 −3 10 1
7. For which value of x are the determinants | | and | | equal?
5 𝑥 −3 2
(a)3 (b)-3 (c)±2 (d)2
4 3 2
8. The value of the cofactor of the element of second row and third column in the matrix [2 −1 0] is:
1 2 3
(a)5 (b)-5 (c)-11 (d)11
𝑑2 𝑦 𝑑𝑥
9. The difference of the order and the degree of the differential equation (𝑑𝑥 2 )2 + (𝑑𝑥)3 + x4 = 0 is
(a)1 (b)2 (c)-1 (d)0
1 −1
10. If the matrix A = [ ] and A2 = kA, then the value of k is:
−1 1
(a)1 (b)2 (c)-1 (d)0
𝑐𝑜𝑠2𝑥
11. ∫ 𝑠𝑖𝑛2 𝑥.𝑐𝑜𝑠2𝑥 𝑑𝑥 is equal to
(a)tanx -cotx +C (b)-cotx-tanx+C (c)cotx+tanx+C (d)tanx-cotx-C
2 𝑑𝑥
12. The integrating factor of the differential equation (3x + y) 𝑑𝑦 = x is
1 1 2 −1
(a)𝑥 (b)𝑥 2 (c)𝑥 (d) 𝑥
13. The point which lies in the half plane 2x + y – 4 ≤ 0 is:
(a)(0,8) (b)(1,1) (c)(5,5) (d)(2,2)
y x 𝑑𝑦
14. If (cosx) = (siny) , then 𝑑𝑥 is equal to:
𝑦𝑡𝑎𝑛𝑥+log (𝑐𝑜𝑠𝑦) 𝑥𝑡𝑎𝑛𝑦+log (𝑐𝑜𝑠𝑥) 𝑦𝑡𝑎𝑛𝑥−log (𝑐𝑜𝑠𝑦) 𝑦𝑡𝑎𝑛𝑥+log (𝑐𝑜𝑠𝑦)
(a) 𝑥𝑡𝑎𝑛𝑥−log (𝑐𝑜𝑠𝑥) (b) 𝑦𝑡𝑎𝑛𝑥+log (𝑐𝑜𝑠𝑦) (c) 𝑥𝑡𝑎𝑛𝑦−log (𝑐𝑜𝑠𝑥) (d) 𝑥𝑡𝑎𝑛𝑦+log (𝑐𝑜𝑠𝑥)
3 2 4 1
15. It is given that X [ ]=[ ]. Then matrix X is
1 −1 2 3
1 0 0 1 1 1 1 −1
(a)[ ] (b) [ ] (c) [ ] (d) [ ]
0 1 1 1 1 −1 1 −1
16. If ABCD is a parallelogram and AC and BD are its diagonals, then 𝐴𝐶 ⃗⃗⃗⃗⃗⃗ + 𝐵𝐷
⃗⃗⃗⃗⃗⃗⃗ is:
⃗⃗⃗⃗⃗⃗
(a)2𝐷𝐴 ⃗⃗⃗⃗⃗⃗
(b)2𝐴𝐵 ⃗⃗⃗⃗⃗⃗
(c)2𝐵𝐶 (d)2𝐵𝐷⃗⃗⃗⃗⃗⃗⃗
17. If x = a cos𝜃 + b sin𝜃 , y = a sin𝜃 − b cos𝜃 , then which one of the following is true ?
𝑑2 𝑦 𝑑𝑦 𝑑2 𝑦 𝑑𝑦 𝑑2 𝑦 𝑑𝑦 𝑑2 𝑦 𝑑𝑦
(a)y2 𝑑𝑥 2 - x𝑑𝑥 + y =0 (b) y2 𝑑𝑥 2 + x𝑑𝑥 + y =0(c) y2 𝑑𝑥 2 - x𝑑𝑥 - y =0 (d) y2 𝑑𝑥 2 - x𝑑𝑥 - y =0
18. The corner points of the bounded feasible region of an LPP are O(0, 0), A(250, 0), B(200, 50) and C(0, 175). If the
maximum value of the objective function Z = 2ax + by occurs at the points A(250, 0) and B(200, 50), then the
relation between a and b is :

(a) 2a = b (b) 2a = 3b (c) a = b (d) a = 2b


Questions number 19 and 20 are Assertion and Reason based questions carrying 1 mark each. Two statements
are given, one labelled Assertion (A) and the other labelled Reason (R). Select the correct answer from the
codes (a), (b), (c) and (d) as given below.
(b) Both Assertion (A) and Reason (R) are true and Reason (R) is the correct explanation of the Assertion (A).
(c) Both Assertion (A) and Reason ® are true and Reason ® is not the correct explanation of the Assertion (A).
(d) Assertion (A) is true, but Reason ® is false.
(e) Assertion (A) is false, but Reason ® is true.
𝜋
19. Assertion(A) :- The principal value of cot-1(√3 ) is 6 .
Reason (R ): Domain of cot-1x is R –{-1, 1}
20. Assertion(A): Quadrilateral formed by vertices A(0, 0, 0), B(3, 4, 5), C(8, 8, 8) and D(5, 4, 3) is a rhombus.
Reason (R) : ABCD is a rhombus if AB = BC = CD = DA, AC ≠ BD.
SECTION B
This section comprises very short answer (VSA) type questions of 2 marks each.
21. If three non-zero vectors are 𝑎⃗ , 𝑏⃗⃗ and 𝑐⃗ such that 𝑎⃗ . 𝑏⃗⃗ = 𝑎⃗ . 𝑐⃗ and 𝑎⃗ X 𝑏⃗⃗ = 𝑎⃗ X 𝑐⃗, then show that 𝑏⃗⃗ = 𝑐⃗.
𝐶𝑜𝑠𝑥
22. (a) Simplify tan-1 ( 1−𝑠𝑖𝑛𝑥 )
OR
Prove that the greatest integer function f: R → R , given by f(x) = [𝑋] , is neither one-one nor onto.
23. Function f is defined as
2𝑥 + 2 𝑖𝑓 𝑥 < 2
F(x) = { 𝑘 𝑖𝑓 𝑥 = 2
3𝑥 𝑖𝑓 𝑥 > 2
Find the value of k for which the function f is continuous at x =2.
24. Find the intervals in which the function f(x) = x4 – 4x3 + 4x2 + 15 is strictly increasing.
25. (a) If 𝑎⃗ , 𝑏⃗⃗ and 𝑐⃗ are three vectors such that |𝑎⃗| = 7, |𝑏⃗⃗| = 24, |𝑐⃗| = 25 and 𝑎⃗ + 𝑏⃗⃗ + 𝑐⃗ = ⃗0⃗, then find the value of 𝑎⃗ . 𝑏⃗⃗ +
⃗⃗⃗⃗
𝑏. 𝑐⃗ + 𝑐⃗. 𝑎⃗ .
OR
If a line makes angles 𝛼, 𝛽, 𝛾 with x-axis , y-axis and z-axis respectively, then prove that
sin2𝛼 + sin2𝛽 + sin2𝛾 = 2
SECTION C
(This section comprises of short answer type questions (SA) of 3 marks each)
26. (a) Evaluate :
𝜋
𝑥𝑠𝑖𝑛𝑥 𝑐𝑜𝑠𝑥
∫02 𝑆𝑖𝑛4 𝑥+ 𝐶𝑜𝑠4 𝑥 dx
𝑂𝑅
3
Evaluate: ∫1 ( |𝑥 − 1| + |𝑥 − 2| ) dx
𝑑𝑦 𝑥𝑦
27. Find the particular solution of the D.E 𝑑𝑥 = 𝑥 2 +𝑦 2 , given that y= 1 when x = 0.
OR
𝑑𝑦 1
Find the particular solution of the D.E ( 1 + x2) + 2xy = 𝑥 2+1 , given that y= 0 when x = 1.
𝑑𝑥
28. Out of two bags, bag A contains 2 white and 3 red balls and bag B contains 4 white and 5 red balls. One ball is drawn
at random from one of the bags and is found to be red. Find the probability that it was drawn from bag B.
OR
Out of a group of 50 people, 20 always speak the truth. Two persons are selected at random from the group (without
replacement). Find the probability distribution of number of selected persons who always speak the truth.
𝐶𝑜𝑠𝑥
29. Find : ∫ √3−3𝑠𝑖𝑛𝑥−𝑐𝑜𝑠2 𝑑𝑥
𝑥
30. Solve the following Linear programming Problem graphically:
Minimise Z = 3x + 8y
Subject to the constraints
3x + 4y ≥ 8
5x + 2y ≥ 11
x,y≥0
2𝑥 2 +1
31. Find ∫ 𝑥 2( 𝑥 2+4 ) dx
SECTION D
This section comprises long answer type questions (LA) of 5 marks each.
3 2 1
32. If matrix A = [4 1 3] , Find A-1 and hence solve the following system of linear equations :
1 1 1
3x + 2y + z = 2000 4x + y + 3z = 2500 x + y + z = 900
𝑥+1 𝑦+3 𝑧+5 𝑥−2 𝑦−4 𝑧−6
33. (a) Show that the lines 3 = 5 = 7 and 1 = 3 = 5 intersect. Also find their point of intersection.
OR
𝑥−1 𝑦+1 𝑥+1 𝑦−2
(a) Find the shortest distance between the pair of lines 2 = 3 = z and 5 = 1 ; z=2.
34. Find the area of the triangle ABC bounded by the lines represented by the equations 5x - 2y -10 = 0,
x- y- 9 = 0 and 3x - 4y- 6 = 0, using integration method.
35. (a) Show that the relation S in set R of real numbers defined by
S = {(a, b) : a ≤ b3 , a ∈ R, b ∈ R} is neither reflexive, nor symmetric, nor transitive.
OR
Let R be the relation defined in the set A = {1, 2, 3, 4, 5, 6, 7} by R = {(a, b) : both a and b are either odd or even}.
Show that R is an equivalence relation. Hence, find the elements of equivalence class [1].

SECTION E
This section comprises 3 case study based questions of 4 marks each.
Case Study 1
36. In a group activity class, there are 10 students whose ages are 16, 17, 15, 14, 19, 17, 16, 19, 16 and 15 years. One
student is selected at random such that each has equal chance of being chosen and age of the student is recorded.

On the basis of the above information, answer the following questions :


(i) Find the probability that the age of the selected student is a composite number.
(ii) Let X be the age of the selected student. What can be the value of X ?
(iii) (a) Find the probability distribution of random variable X and hence find the mean age.
OR
(iii) (b) A student was selected at random and his age was found to be greater than 15 years. Find the probability that
his age is a prime number.
Case Study 2
37. A housing society wants to commission a swimming pool for its residents. For this, they have to purchase a square
piece of land and dig this to such a depth that its capacity is 250 cubic metres. Cost of land is < 500 per square metre.
The cost of digging increases with the depth and cost for the whole pool is < 4000 (depth)2.

Suppose the side of the square plot is x metres and depth is h metres.
On the basis of the above information, answer the following questions :
(i) Write cost C(h) as a function in terms of h.
(ii) Find critical point.
(iii) (a) Use second derivative test to find the value of h for which cost of constructing the pool is minimum. What is
the minimum cost of construction of the pool ?
OR
(iii) (b) Use first derivative test to find the depth of the pool so that cost of construction is minimum. Also, find
relation between x and h for minimum cost. 2
Case Study 3
38. In an agricultural institute, scientists do experiments with varieties of seeds to grow them in different environments to
produce healthy plants and get more yield. A scientist observed that a particular seed grew very fast after
germination. He had recorded growth of plant since germination and he said that its growth can be defined by the
function.
1
F(x) = 3 x3 – 4x2 + 15x + 2 , 0 ≤ x ≤ 10
where x is the number of days the plant is exposed to sunlight.

On the basis of the above information, answer the following questions :


(i) What are the critical points of the function f(x) ?
(ii) Using second derivative test, find the minimum value of the function.
CBSE SAMPLE PAPER-2022-23
SECTION A
(Multiple Choice Questions)
Each question carries 1 mark
1. If A =[aij] is a skew-symmetric matrix of order n, then
1
(a) aij = ∀ i, j (b)aij ≠ 0 ∀ i, j (c) aij = 0, where i=j (d) aij ≠ 0 where i= j
𝑎𝑖𝑗
2. If A is a square matrix of order 3, |𝐴′| = −3, then |𝐴𝐴′| =
(a) 9 (b) -9 (c) 3 (d) -3
3. The area of a triangle with vertices A, B, C is given by
1 1 1
⃗⃗⃗⃗⃗⃗ 𝑋 ⃗⃗⃗⃗⃗⃗
(a) |𝐴𝐵 𝐴𝐶 | (b) |𝐴𝐵⃗⃗⃗⃗⃗⃗ 𝑋 ⃗⃗⃗⃗⃗⃗
𝐴𝐶 | ⃗⃗⃗⃗⃗⃗ 𝑋 ⃗⃗⃗⃗⃗⃗
(c) 4 |𝐴𝐶 𝐴𝐵 | ⃗⃗⃗⃗⃗⃗ 𝑋 ⃗⃗⃗⃗⃗⃗
(d) 8 |𝐴𝐶 𝐴𝐵 |
2
1−𝑐𝑜𝑠4𝑥
𝑖𝑓 𝑥 ≠ 0
4. The value of ‘k’ for which the function f(x) = { 8𝑥 2 is continuous at x=0 is
𝑘 𝐼𝑓 𝑥 = 0
(a)0 (b)-1 (c)1 (d)2
1
5. If 𝑓 ′ (𝑥) = 𝑥 + 𝑥 , then f(x) is
(a)x2 + log|𝑥| +C (b) x2/2 + log|𝑥| +C (c) x/2 + log|𝑥| +C (d) x/2 - log|𝑥| +C
6. If m and n, respectively, are the order and the degree of the differential equation
𝑑 𝑑𝑦
[ (𝑑𝑥 )]4 = 0, then m + n = (𝑎)1 (𝑏)2 (𝑐)3 (𝑑)4
𝑑𝑥

7. The solution set of the inequality 3x + 5y < 4 is


(a)An open half-plane not containing the origin.
(b)An open half-plane containing the origin.
(c)The whole XY-Plane not containing the line 3x + 5y =4.
(d)A closed half plane containing the origin.
8. The scalar projection of the vectors 3𝑖̂ - 𝑗̂ - 2𝑘̂ on the vector 𝑖̂ + 2𝑗̂ - 3𝑘̂ is
7 7 6 7
(a) (b) 14 (c) 13 (d) 2
√14
3 𝑥
9. The value of ∫2 𝑑𝑥
𝑥 2 +1
3 1 9
(a)log4 (b) log2 (c) 2 𝑙𝑜𝑔2 (d) log4
10. If A, B are non-singular square matrices of the same order, then (𝐴𝐵-1)-1 =
(a)A-1B (b)A-1B-1 (c)BA-1 (d)AB
11. The corner points of the shaded unbounded feasible region of an LPP are (0, 4), (0.6, 1.6) and (3, 0) as shown in the figure.
The minimum value of the objective function Z = 4x + 6y occurs at

(a)(0.6, 1.6) 𝑜𝑛𝑙𝑦 (b) (3, 0) only (c) (0.6, 1.6) and (3, 0) only
(d) at every point of the line-segment joining the points (0.6, 1.6) and (3, 0)
2 4 2𝑥 4
12. If | |=| |, then the possible value(s) of ‘x’ is/are
5 1 6 𝑥
(a)3 (b)√3 (c)- √3 (d) √3 , - √3
13. If A is a square matrix of order 3 and |A| = 5, then |𝑎𝑑𝑗𝐴| =
1
(a) 5 (b) 25 (c) 125 (d)5
14. Given two independent events A and B such that P(A) =0.3, P(B) = 0.6 and P(A’ ∩ B’ ) is
(a) 0.9 (b) 0.18 (c) 0.28 (d) 0.1
15. The general solution of the differential equation 𝑦𝑑𝑥 − 𝑥𝑑𝑦 = 0 𝑖𝑠
(a)xy=C (b)x =Cy2 (c)y=Cx (d)y=Cx2
-1 2
16. If y = sin x, then ( 1- x ) y2 is equal to
(a)xy1 (b)xy (c)xy2 (d)x2
17. If two vectors 𝑎⃗ and 𝑏⃗⃗ are such that |𝑎⃗| = 2 , |𝑏⃗⃗| = 3 𝑎𝑛𝑑 𝑎⃗. 𝑏⃗⃗ = 4, then |𝑎⃗ − 2𝑏⃗⃗| is
(a)√2 (b)2√6 (c)24 (d)2√2
18. P is a point on the line joining the points 𝐴(0, 5, −2) and 𝐵(3, −1,2). If the x-coordinate of P is 6, then its z-coordinate is
(a) 10 (b) 6 (c) -6 (d) -10
ASSERTION-REASON BASED QUESTIONS
In the following questions, a statement of assertion (A) is followed by a statement of Reason (R). Choose the correct
answer out of the following choices.
(a) Both A and R are true and R is the correct explanation of A.
(b) Both A and R are true but R is not the correct explanation of A.
(c) A is true but R is false.
(d) A is false but R is true.

1 1
19. Assertion(A) : The domain of the function sec-12x is ( -∞ , - 2 ] U [ 2 , ∞)
𝜋
Reason(R) : Sec-1(-2 ) = - 4
𝜋
20. Assertion(A) : The acute angle between the line 𝑟⃗ = 𝑖̂ + 𝑗̂ + 2𝑘̂ + λ(𝑖̂ - 𝑗̂ ) and the x-axis is 4 .
Reason(R) : The acute angle 𝜃 between the lines
𝑟⃗ = x1 𝑖̂ + y1 𝑗̂ + z1 𝑘̂ + λ(a1 𝑖̂ + b1 𝑗̂ + c1 𝑘̂) and
|𝑎1 𝑎2 +𝑏1 𝑏2 +𝑐1 𝑐2 |
𝑟⃗ = x2 𝑖̂ + y2 𝑗̂ + z2 𝑘̂ + λ(a2 𝑖̂ + b2 𝑗̂ + c2 𝑘̂) is given by cos𝜃 =
√𝑎1 2 +𝑏1 2 +𝑐1 2 √𝑎2 2 +𝑏2 2 +𝑐2 2

SECTION B
This section comprises of very short answer type-questions (VSA) of 2 marks each

13𝜋
21. Find the value of 𝑠𝑖𝑛−1 [sin( )]
7
OR
𝑛+1
, 𝑖𝑓 𝑛 𝑖𝑠 𝑜𝑑𝑑
Prove that the function f is surjective, where f:N→N such that f(n)={ 𝑛2 for all n∈N.
, 𝑖𝑓 𝑛 𝑖𝑠 𝑒𝑣𝑒𝑛
2
Is the function injective? Justify your answer.

22. A man 1.6 m tall walks at the rate of 0.3 m/sec away from a street light that is 4 m above the ground. At what rate is the tip
of his shadow moving? At what rate is his shadow lengthening?

23. If 𝑎⃗ = 𝑖̂ - 𝑗̂ + 7𝑘̂and 𝑏⃗⃗ = 5𝑖̂ - 𝑗̂ +λ 𝑘̂ then find the value of λ so that the vectors 𝑎⃗+𝑏⃗⃗ and 𝑎⃗-𝑏⃗⃗ are orthogonal.
OR
Find the direction ratio and direction cosines of a line parallel to the line whose equations are
6𝑥 − 12 = 3𝑦 + 9 = 2𝑧 − 2 .
𝑑𝑦 1−𝑦 2
24. If y √1 − 𝑥 2 + 𝑥√1 − 𝑦 2 =1, then prove that = - √1−𝑥 2 .
𝑑𝑥

25. Find |𝑥⃗| if ( 𝑥⃗ - 𝑎⃗ ) (𝑥⃗ + 𝑎⃗ ) 12, where 𝑎⃗ is a unit vector.


SECTION C
(This section comprises of short answer type questions (SA) of 3 marks each)
𝑑𝑥
26. Evaluate: ∫ √3−2𝑥−𝑥 2 .
27. Three friends go for coffee. They decide who will pay the bill, by each tossing a coin and then letting the “odd
person” pay. There is no odd person if all three tosses produce the same result. If there is no odd person in the first
round, they make a second round of tosses and they continue to do so until there is an odd person. What is the
probability that exactly three rounds of tosses are made?
OR
Find the mean number of defective items in a sample of two items drawn one-by-one without replacement from an
urn containing 6 items, which include 2 defective items. Assume that the items are identical in shape and size.
𝜋
3 𝑑𝑥
28. Evaluate:- ∫ 𝜋
1+√𝑡𝑎𝑛𝑥
6
OR
4
Evaluate: ∫0 |𝑥 − 1|dx
29. Solve the D.E 𝑦 𝑑𝑥 + ( x – y2 ) dy = 0
OR
Solve the following D.E x dy - y dx = √𝑥 2 + 𝑦 2 dx

30. Solve the following linear programming problem graphically:


Minimise Z = 400 x + 300 y
subject to the constraints:
x + y ≤ 200 , x ≤ 40 , x ≥ 20, y ≥ 0

𝑥 3 +𝑥+ 1
31. Evaluate: ∫ dx
𝑥 2 −1

SECTION D
(This section comprises of long answer-type questions (LA) of 5 marks each)

32. Make a rough sketch of the region :{(x, y): 0≤y≤x2, 0≤y≤x+2; 0≤x ≤ 2} and find the area of the region using
integration.

33. Define the relation R in the set 𝑁 × 𝑁 as follows: For (a, b), (c, d) ∈ 𝑁 × 𝑁, (a, b) R (c, d) iff ad = bc. Prove that R is
an equivalence relation in 𝑁 × 𝑁
OR
Given a non-empty set X, define the relation R in P(X) as follows: For A, B ∈ P(X), (A, B ) ∈ R Iff A C B. Prove that R is
reflexive, transitive and not symmetric.
34. An insect is crawling along the line 𝑟⃗ = 6𝑖̂ + 2𝑗̂ + 2𝑘̂ + 𝜆(𝑖̂ − 2𝑗̂ + 2𝑘̂) and another insect is crawling along the line
𝑟⃗ = −4𝑖̂ − 𝑘̂ + 𝜇(3𝑖̂ − 2𝑗̂ − 2𝑘̂) . At what points on the lines should they reach so that the distance between them is
the shortest? Find the shortest possible distance between them.
𝑶𝑹
The equations of motion of a rocket are: .
𝑥 = 2𝑡, 𝑦 = −4𝑡, 𝑧 = 4𝑡, where the time t is given in seconds, and the coordinates of a moving point in km. What is the
path of the rocket? At what distances will the rocket be from the starting point O(0, 0, 0) and from the following line in 10
seconds? 𝑟⃗ = 20𝑖̂ − 10𝑗̂ + 40𝑘̂ + 𝜇(10𝑖̂ − 20𝑗̂ + 10𝑘̂)
2 −3 5
35. If A= [3 2 −4] . Find A-1. Use A-1 to solve the following system of equations
1 1 −2
2x – 3y + 5z = 11, 3x + 2y – 4z = -5, x + y – 2z = -3.
SECTION E
(This section comprises of 3 case-study/passage-based questions of 4 marks each with two sub-parts. First two
case study questions have three sub-parts (i), (ii), (iii) of marks 1, 1, 2 respectively. The third case study
question has two Sub-parts of 2 marks each.)
36. Case- Study-I:- Read the passage given below and answer the following questions.

The temperature of a person during an intestinal illness is given by


F(x) = -0.1x2 + mx + 98.6, 0 ≤ x ≤ 12, m being a constant, where f(x) is the temperature in °F at x days.
(i) Is the function differentiable in the interval (0, 12)? Justify your answer.
(ii) If 6 is the critical point of the function, then find the value of the constant m.
(iii) Find the intervals in which the function is strictly increasing/strictly decreasing.
OR
Find the points of local maximum/local minimum, if any, in the interval (0, 12) as well as the points of absolute
maximum/absolute minimum in the interval [0, 12]. Also, find the corresponding local maximum/local minimum and the
absolute maximum/absolute minimum values of the function.

37. Case- Study-2:- Read the passage given below and answer the following questions.

In an elliptical sport field the authority wants to design a rectangular soccer field with the maximum possible area.
𝑥2 𝑦2
The sport field is given by the graph of 𝑎2 + 𝑏2 = 1.
(i) If the length and the breadth of the rectangular field be 2x and 2y respectively, then find the area function in terms
of x.
(ii) Find the critical point of the function.
(iii) Use First derivative Test to find the length 2x and width 2y of the soccer field (in terms of a and b) that maximize
its area.
OR
(iii) Use Second Derivative Test to find the length 2x and width 2y of the soccer field (in terms of a and b) that
maximize its area.
38. Case- Study-3:- Read the passage given below and answer the following questions.
There are two antiaircraft guns, named as A and B. The probabilities that the shell fired from them hits an airplane are
0.3 and 0.2 respectively. Both of them fired one shell at an airplane at the same time.
(i) What is the probability that the shell fired from exactly one of them hit the plane?
(ii) If it is known that the shell fired from exactly one of them hit the plane, then what is the probability that it was
fired from B?

***********************************************************************
CBSE BOARD QUESTION PAPER-2022
SECTION A
Questions number 1 to 6 carry 2 marks each
𝑑 𝑑𝑦
1. Find the product of the order and the degree of the D.E [ 𝑑𝑥 (𝑥𝑦 2 )]. 𝑑𝑥 + y = 0.
𝑠𝑖𝑛3𝑥
2. (a) Find: ∫ dx
𝑠𝑖𝑛𝑥
OR
1
𝑙𝑜𝑔3 𝑒𝑥
(b) Evaluate: ∫0 2 dx
𝑒 2𝑥 +1
3. If 𝑎⃗ and 𝑏⃗⃗ are two-unit vectors such that |2𝑎⃗ + 3𝑏⃗⃗| = |3𝑎⃗ − 2𝑏⃗⃗|. Find the angle between 𝑎⃗ and 𝑏⃗⃗.
4. A pair of dice is thrown. It is given that the sum of numbers appearing on both dice is an even number.
Find the probability that the number appearing on at least one die is 3.
5. Probabilities of A and B solving a specific problem are 2/3 and 3/5 respectively. If both of them try
independently to solve the problem, then find the probability that the problem is solved.
6. Write the cartesian equation of the line PQ passing through points P(2, 2, 1) and Q(5, 1, 2). Hence, find
the y-coordinate of the point on the line PQ whose z-coordinate is -2.
SECTION B
Questions number 7 to 10 carry 3 marks each
7. ABCD is a parallelogram such that ⃗⃗⃗⃗⃗⃗ 𝐵𝐷 = 2𝑖̂ + 𝑗̂ + 𝑘̂ . Find ⃗⃗⃗⃗⃗⃗
𝐴𝐶 = 𝑖̂ + 𝑗̂ and ⃗⃗⃗⃗⃗⃗⃗ 𝐴𝐵 𝑎𝑛𝑑 ⃗⃗⃗⃗⃗⃗
𝐴𝐷. Also find the area of the
parallelogram ABCD.
1
8. (a)Evaluate : ∫0 𝑡𝑎𝑛−1 𝑥 dx
OR
2𝑥
Find:- ∫ dx
𝑥 2 +3𝑥+2
𝑑𝑥
9. Find the particular solution of the differential equation ( y + 3x2 ) = x , given that y = 1 when x =1.
𝑑𝑦
10. (a)Find the equation of the plane passing through points (2, 1, 0), (3, 2, 2) and (1, 1, 7). Also, obtain its
distance from the origin.
OR
𝑦−1 𝑧−2 𝑦+2 𝑧−1
Find the distance between the lines x = = and x+1 = = .
2 3 2 3
SECTION C
Questions number 11 to 14 carry 4 marks each
𝑥−2 𝑦+1 𝑧−2
11. Find the distance of the point (-1, -5, -10) from the point of intersection of the line = = and
3 4 12
the plane x – y + z = 5.
1
12. Evaluate: ∫0 𝑥 ( 1 − 𝑥)𝑛 𝑑𝑥
13. Using integration, find the area of the smaller region enclosed by the curve 4x2 + 4y2 = 9 and the
line 2x + 2y = 3.
OR
If the area of the region bounded by the curve y2 = 4ax and the line x = 4a is 256/3 sq. units, then using
integration, find the value of a, where a > 0.
Case-Study Based Question
14. At the start of a cricket match, a coin is tossed and the team winning the toss has the opportunity to
choose to bat or bowl. Such a coin is unbiased with equal probabilities of getting head and tail.

Based on the above information, answer the following questions:


(a) If such a coin is tossed 2 times, then find the probability distribution of number of tails. 2
(b) Find the probability of getting at least one head in three tosses of such a coin. 2

CBSE BOARD SUPPLEMENTARY QUESTION PAPER-2022


SECTION A
Questions number 1 to 6 carry 2 marks each
𝜋
1
1. Evaluate:- ∫0 2
5 dx
1+ 𝐶𝑜𝑡 2 𝑥
2. If 𝑎⃗ = 𝑖̂ + 𝑗̂ − 2𝑘̂ , 𝑏⃗⃗ = −𝑖̂ + 2𝑗̂ + 2𝑘̂ and 𝑐⃗ = −𝑖̂ + 2𝑗̂ − 𝑘̂ are three vectors, then find a vector perpendicular to
both the vectors (𝑎⃗ + 𝑏⃗⃗ ) and ( 𝑏⃗⃗ - 𝑐⃗ ).
3. A bag contains cards numbered 1 to 25. Two cards are drawn at random, one after the other, without
replacement. Find the probability that the number on each card is a multiple of 7.
4. One bag contains 4 white and 5 black balls. Another bag contains 6 white and 7 black balls. A ball,
drawn at random, is transferred from the first bag to the second bag and then a ball is drawn at random
from the second bag. Find the probability that the ball drawn is white.
5. If 𝑎⃗ , 𝑏⃗⃗ and 𝑐⃗ are unit vectors such that 𝑎⃗ + 𝑏⃗⃗ + 𝑐⃗ = ⃗0⃗, then find the value of 𝑎⃗ .𝑏⃗⃗ + 𝑏⃗⃗ . 𝑐⃗ + 𝑐⃗. 𝑎⃗ .
6. Find the general solution of the differential equation
x cos y dy = (x log x + 1) ex dx
OR
Find the value of (2a 3b), if a and b represent respectively the order and the degree of the
𝑑2𝑦 3 𝑑𝑦 𝑦 𝑑𝑦
differential equation x[y ( ) + 𝑥( )2 − ] =0
𝑑𝑥 2 𝑑𝑥 𝑥 𝑑𝑥

SECTION B
Questions number 7 to 10 carry 3 marks each
7. Find the area of the region {(x,y) : x2 + y2 ≤ 9 , x + y ≥ 3 } using integration
OR
Using integration find the area of the region bounded by the parabola y2 = 4a, the lines x = 0 and x = 3
and the axis.
𝑠𝑖𝑛𝑥
8. Find : ∫ 𝑑𝑥
sin (𝑥−2𝑎)
9. Find the equation of the plane passing through three points whose position vectors are
-j , 3i + 3j and i + j + k.
10. Find the distance between the following parallel lines:
𝑟⃗ = 2𝑖̂ + 𝑗̂ − 𝑘̂ + 𝜆(𝑖̂ + 𝑗̂ − 𝑘̂)
𝑟⃗ = 𝑖̂ − 2𝑗̂ + 𝑘̂ + 𝜇(𝑖̂ + 𝑗̂ − 𝑘̂)
OR
Find the coordinates of the point where the line through the points ( 1, 1, 8) and (5, 2, 10) crosses the
ZX-plane.
SECTION C
Questions number 11 to 14 carry 4 marks each
11. Find the equation of the plane passing through the intersection of the planes
𝑟⃗. (2𝑖̂ + 2𝑗̂ − 3𝑘̂) = 7 and 𝑟⃗. (2𝑖̂ + 5𝑗̂ + 3𝑘̂) = 9 and through the point (2, 1, 3).
12. Find : ∫ 𝐶𝑜𝑠𝑥 . 𝑡𝑎𝑛−1 (𝑠𝑖𝑛𝑥)𝑑𝑥
OR
𝑒𝑥
Find: ∫ 𝑑𝑥
(𝑒 𝑥 +1)(𝑒 𝑥 +3)
13. Find the particular solution of the differential equation:-
𝑑𝑦
x + 2 y = x2 logx, given y(1) =1.
𝑑𝑦
Case-Study Based Question
A biased die is tossed and respective probabilities for various faces to turn up are the following :

Based on the above information, answer the following questions :


(a) What is the value of K ?
(b) If a face showing an even number has turned up, then what is the probability that it is the face with 2
or 4 ?
CBSE BOARD QUESTION PAPER-2020
SECTION A
Question numbers 1 to 20 carry 1 mark each.
Question numbers 1 to 10 are multiple choice type questions. Select the correct option.
1. If A is a square matrix of order 3 and |𝐴| = 5, then the value of |2𝐴′ | is
(a)-10 (b)10 (c)-40 (d)40
2. If A is a square matrix such that A = A, then (I – A) + A is equal to
2 3

(a)I (b)0 (c)I- A (d)I + A


3𝜋
3. The principal value of tan-1 ( tan ) is
5
2𝜋 2𝜋 3𝜋 3𝜋
(a) (b) − (c) (d) −
5 5 5 5
4. If the projection of 𝑎⃗ = 𝑖̂ − 2𝑗̂ + 3𝑘̂ on 𝑏⃗⃗ = 2𝑖̂ + 2𝑗̂ − 3𝑘̂ is zero, then the value of λ is
−2 −3
(a)0 (b)1 (c) 3 (d) 2
5. The vector equation of the line passing through the point (–1, 5, 4) and perpendicular to the plane z = 0 is
(a) 𝑟⃗ = −𝑖̂ + 5𝑗̂ + 4𝑘̂ + 𝜆(𝑖̂ + 𝑗̂) (b) 𝑟⃗ = −𝑖̂ + 5𝑗̂ + (4 + 𝜆)𝑘̂
(b) (c) 𝑟⃗ = −𝑖̂ − 5𝑗̂ − 4𝑘̂ + 𝜆𝑘̂ (d)𝑟̂ = λ𝑘̂
6. The number of arbitrary constants in the particular solution of a differential equation of second order is (are)
(a)0 (b)1 (c)2 (d)3
𝜋
7. ∫ 4
−𝜋 𝑠𝑒𝑐 2 𝑥𝑑𝑥 is equal to (a)-1 (b)0 (c)1 (d)2
4
8. The length of the perpendicular drawn from the point (4, – 7, 3) on the y-axis is
(a)3 units (b)4 units (c)5 units (d)7 units
1 1 ’
9. If A and B are two independent events with P(A) = 3 and P(B) = 4 , then P(B / A) is equal to
(a)1/4 (b)1/3 (c)3/4 (d)1
10. The corner points of the feasible region determined by the system of linear inequalities are (0, 0), (4, 0),
(2, 4) and (0, 5). If the maximum value of z = ax + by, where a, b > 0 occurs at both (2, 4) and (4, 0), then
(a)a=2b (b)2a=b (c)a=b (d)3a=b

Fill in the blanks in question numbers 11 to 15.


11. A relation R in a set A is called __________, If (a1 , a2 ) ∈ R implies (a2 , a1) ∈ R, for all a1 , a2 ∈ A.
12. The greatest integer function defined by f(x) = [𝑥] , 0 < x< 2 is not differentiable at x =______.
13. If A is a matrix of order 3 2, then the order of the matrix A’ is ________
OR
A square matrix A is said to be skew-symmetric, if ___________ .
14. The equation of the normal to the curve y2 = 8x at the origin is ___________.
OR
The radius of a circle is increasing at the uniform rate of 3 cm/sec. At the instant when the radius of the circle is 2
cm, its area increases at the rate of _____________ cm2 /s.
15. The position vectors of two points A and B are ⃗⃗⃗⃗⃗⃗
𝑂𝐴 = 2𝑖̂ − 𝑗̂ − 𝑘̂ and ⃗⃗⃗⃗⃗⃗
𝑂𝐵 = 2𝑖̂ − 𝑗̂ + 2𝑘̂ , respectively. The position
vector of a point P which divides the line segment joining A and B in the ratio 2 : 1 is ______ .

Question numbers 16 to 20 are very short answer type questions


2 0 0
16. If A = [−1 2 3], then find A (adj A).
3 3 5
17. Find ∫ 𝑥 4 𝑙𝑜𝑔𝑥 dx
OR
2𝑥
Find :- ∫ 3 2 dx
√𝑥 +1
3
18.Evaluate:- ∫1 |2𝑥 − 1| dx
19. Two cards are drawn at random and one-by-one without replacement from a well-shuffled pack of 52 playing cards.
Find the probability that one card is red and the other is black.
𝑑𝑥
20. Find ∫ √9−4𝑥 2
SECTION B
Question numbers 21 to 26 carry 2 marks each.
1
21. Prove that sin-1 (2x √1 − 𝑥 2 ) = 2Cos-1 x , ≤ x ≤ 1.
√2
OR
Consider a bijective function: R+ → ( 7, ∞ ) given by f(x) =16x2 + 24x + 7, where R+ is the set of all positive real
numbers. Find the inverse function of f.
𝑑2 𝑦
22. If x = at2 , y = 2at then find 𝑑𝑥 2 .
23. Find the points on the curve y = x3 -3x2 -4x at which the tangent lines are parallel to the line 4x + y – 3= 0.
̂ and
24. Find a unit vector perpendicular to each of the vectors 𝑎⃗ and 𝑏⃗⃗ where 𝑎⃗ = 5𝑖̂ + 6𝑗̂ − 2𝑘
𝑏⃗⃗ = 7𝑖̂ + 6𝑗
̂ + 2𝑘̂ .
𝑂𝑅
Find the volume of the parallelopiped whose adjacent edges are represented by 2𝑎⃗ , - 𝑏⃗⃗ and 3𝑐⃗ , where
𝑎⃗ = 𝑖̂ − 𝑗̂ + 2𝑘̂ , 𝑏 = 3𝑖̂ + 4𝑗̂ − 5𝑘̂ and 𝑐⃗ = 2𝑖̂ − 𝑗̂ + 3𝑘̂ .
25. Find the value of k so that the lines x = – y = kz and x – 2 = 2y + 1 = – z + 1 are perpendicular to each other.
26. The probability of finding a green signal on a busy crossing X is 30%. What is the probability of finding a green
signal on X on two consecutive days out of three ?
SECTION C
Question numbers 27 to 32 carry 4 marks each.
27. Let N be the set of natural numbers and R be the relation on N X N defined by (a, b) R (c, d) iff ad = bc
for all a, b, c, d ∈ N. Show that R is an equivalence relation.
2 𝑑𝑦
28. If y = 𝑒 𝑥 𝑐𝑜𝑠𝑥 + (cosx)x , then find 𝑑𝑥 .
29. Find ∫ 𝑠𝑒𝑐 3 𝑥 dx
30. Find the general solution of the differential equation
Yey dx = ( y3 + 2x ey)dy
OR
Find the general solution of the differential equation
𝑑𝑦 𝑦 𝜋
X 𝑑𝑥 = y – xtan (𝑥 ) , given that y = 4 at x =1.

31. A furniture trader deals in only two items chairs and tables. He has Rs 50,000 to invest and a space to store at most 35
items. A chair costs him Rs 1,000 and a table costs him Rs 2,000. The trader earns a profit of Rs 150 and Rs 250 on a
chair and table, respectively. Formulate the above problem as an LPP to maximise the profit and solve it graphically.
32. There are two bags, I and II. Bag I contains 3 red and 5 black balls and Bag II contains 4 red and 3 black balls. One
ball is transferred randomly from Bag I to Bag II and then a ball is drawn randomly from Bag II. If the ball so drawn
is found to be black in colour, then find the probability that the transferred ball is also black.
OR
An urn contains 5 red, 2 white and 3 black balls. Three balls are drawn, one-by-one, at random without replacement.
Find the probability distribution of the number of white balls. Also, find the mean and the variance of the number of
white balls drawn.
SECTION D
Question numbers 33 to 36 carry 6 marks each.
1 2 −3
33. If A = [3 2 −2] then find A-1 and use it to solve the following system of the equations:
2 −1 1
x + 2y – 3z = 6 3x + 2y – 2z = 3 2x – y + z = 2

OR
Using properties of determinants, prove that
(𝒃 + 𝒄)𝟐 𝒂𝟐 𝒃𝒄
| (𝒄 + 𝒂)𝟐 𝒃𝟐 𝒄𝒂 | = (a – b) (b – c) (c – a) (a + b + c) (a2 +b2 + c2 )
(𝒂 + 𝒃)𝟐 𝒄𝟐 𝒂𝒃

34. Using integration, find the area of the region bounded by the triangle whose vertices are (2, – 2), (4, 5) and (6, 2).
35. Show that the height of the right circular cylinder of greatest volume which can be inscribed in a right circular cone
4
of height h and radius r is one-third of the height of the cone, and the greatest volume of the cylinder is 9 times the
volume of the cone.
36. Find the equation of the plane that contains the point A(2, 1, – 1) and is perpendicular to the line of intersection of the
planes 2x + y – z = 3 and x + 2y + z = 2. Also find the angle between the plane thus obtained and the y-axis.
OR
Find the distance of the point P(– 2, – 4, 7) from the point of intersection
Q of the line 𝑟⃗ = 3𝑖̂ − 2𝑗̂ + 6𝑘̂ + 𝜆(2𝑖̂ − 𝑗̂ + 2𝑘
⃗⃗ ) and the plane 𝑟⃗ . (𝑖̂ − 𝑗̂ + 𝑘̂) = 6. Also write the vector equation of
the line PQ.
CBSE BOARD QUESTION PAPER-2019
SECTION A (1 MARK EACH)
𝐴
1. Form the differential equation representing the family of curves y = 𝑥 + 5, by eliminating the arbitrary constant A.
2. If A is a square matrix of order 3, With |𝐴| =9, then write the value of |2. 𝑎𝑑𝑗 𝐴|.
3. Find the acute angle between the planes 𝑟⃗ .(i– 2j -2k)=1 and 𝑟⃗ .(i – 2j -2k)=0.
OR
Find the length of the intercept, cut off by the planes 2x + y –z =5 on the x-axis.
𝑑𝑦
4. If y = log (cos ex ), then find 𝑑𝑥 .

SECTION B (2 MARKS EACH)


0 1+𝑡𝑎𝑛𝑥
5. Find ∫−𝜋 1−𝑡𝑎𝑛𝑥 𝑑𝑥.
4
6. Let * be an operation defined as * : R X R →R such that a * b =2a +b, a,b ∈R. Check if * is a binary operation. If yes
find if it is associative too.
7. X and Y are two points with position vectors 3𝑎⃗ + 𝑏⃗⃗ and 𝑎⃗ - 3𝑏⃗⃗ respectively. Write the position vector of a point Z
which divides the line segment XY in the ratio 2:1 externally.
OR
Let 𝑎⃗ = i +2j -3k and 𝑏⃗⃗ = 3i ij +2k be two vectors. Show that the vectors 𝑎⃗ + 𝑏⃗⃗ and 𝑎⃗ - 𝑏⃗⃗ are perpendicular to each
other.
8. If A and B are symmetric matrices, such that AB and BA are both defined, then prove that AB- BA is a skew
symmetric matrix.
9. 12 cards numbered 1 to 12 (One number on one card), are placed in a box and mixed up thoroughly. Then a card is
drawn at random from the box. If it is known that the number on the drawn card is greater than 5, find the probability
that the card bears an odd number.
10. Out of 8 outstanding students of a school, in which there are 3 boys and 5 girls, a team of4 students is to be selected
for a quiz competition. Find the probability that 2 boys and 2 girls are selected.
OR
In a multiple choice examination with three possible answers for each of five questions, what is the probability that a
candidate would get four or more correct answers just by guessing?
11. Solve the following differential equation:
𝑑𝑦
+ y = cos x –sinx.
𝑑𝑥
12. Find : ∫ 𝑥 . 𝑡𝑎𝑛−1 𝑥 dx.
OR
𝑑𝑥
Find : ∫ √5−4𝑥−2𝑥 2 .
SECTION C (4 MARKS EACH)

13. Using properties of determinants, Find thevalue of x for which,


4−𝑥 4+𝑥 4+𝑥
|4 + 𝑥 4 − 𝑥 4 + 𝑥|=0.
4+𝑥 4+𝑥 4−𝑥
𝑑𝑦
14. Solve the differential equation 𝑑𝑥 = 1 +x2 +y2 +x2y2, given that y =1 when x=0.
OR
𝑑𝑦 𝑥𝑦
Find the particular solution of the differential equation 𝑑𝑥 = 𝑥 2 + 𝑦 2 , given that y=1 when x=0.
𝑥−1
15. Let A=R-{2} and B=R-{1}.Consider the function f : A→B defined by f(x)= 𝑥−2 .
Show that f is one-one and onto hence find f-1.
OR
Show that the relation S in the set A={x:x∈ Z,0≤ 𝑥 ≤ 12}given by S={(a,b):|𝑎 − 𝑏| is divisible by 3} is an
equivalence relation.
cos (𝑥+𝑎)
16. Integrate the function : w.r.t x.
sin (𝑥+𝑏)
𝑑2 𝑦 𝑑𝑦
17. If x = sint, y = sinpt, prove that ( 1- x2 ) 𝑑𝑥 2 - x𝑑𝑥 +𝑝2 y = 0.
OR
√1+𝑥 2 −√1−𝑥 2
Differentiate tan-1[√1+𝑥 2 ] with respect to cos-1x2.
+√1−𝑥 2
12 -13 -156.
18. Prove that cos-113 +sin =sin
5 65
cosx sinx 𝑑𝑦
19. If y=(x) + (cosx) , find 𝑑𝑥 .
𝑎 𝑎 1
20. Prove that ∫0 𝑓(𝑥)𝑑𝑥 = ∫0 𝑓(𝑎 − 𝑥)𝑑𝑥, 𝑎𝑛𝑑 ℎ𝑒𝑛𝑐𝑒 𝑒𝑣𝑎𝑙𝑢𝑎𝑡𝑒 ∫0 𝑥 2 (1 − 𝑥)𝑛 𝑑𝑥
21. Find x such that the four points A(x, -1, -1) , B(4, 5, 1), C(3, 9, 4) and D(-4, 4, 4) are coplanar.
22. A ladder 13m long is leaning against a vertical wall. The bottom of the ladder is dragged away from the wall along
the ground at the rate of 2cm/sec. How fast is the height on the wall decreasing when the foot of the ladder is 5m
away from the wall?
23. Find the vector equation of the plane determined by the points A(3, -1, 2), B(5, 2, 4) and C(-1, -1, 6). Hence find the
distance of the plane, thus obtained from the origin.
SECTION D (6 MARKS EACH)
𝑥2 𝑦2
24. Using integration, find the area of the greatest rectangle that can be inscribed in an ellipse : 𝑎2 + =1
𝑏2
25. An insurance company insured 3000 cyclists,6000 scooter drivers and 9000 car drivers. The probability of an
accident involving a cyclist, scooter driver and a car driver are 0.3, 0.05 and 0.02 respectively.One of the insurance
persons meets with an accident. What is the probability that he is a cyclist?
2 −3 5
26. Using elementary row transformation, find the inverse of the matrix. A = [3 2 −4] .
1 1 −2
OR
Using matrices, solve the following system of linear equations
x + 2y -3z = -4
2x +3 y +2z =2
3x -3y -4z =11
27. Using integration, find the area of the region bounded by the parabola y2 =4x and the circle
4X 2 +4 Y2 =9.
OR
Using the method of integration, find the area of the region bounded by the lines 3x -2y +1=0,
2x +3y -21=0 and x -5y +9 =0.
28. A dietician wishes to mix two types of foods in such way that the vitamin contents of mixture contains at least 8
units of vitamin A and 10 units of vitamin C. Food I contains 2 units per kg of vitamin A and 1 unit per kg of vitamin
C, while food II contains 1 unit per kg of vitamin A and 2 units per kg of vitamin C. It costs Rs 5 per kg to purchase
food I and Rs 70 per kg to purchase food II. Find the minimum cost of such a mixture. Formulate above as an LPP
and solve it graphically.
29. Find the vector equation of a line passing through the point (2,3,2) and parallel to the line
𝑟⃗ = (-2i +3j ) + λ(2i -3j +6 k). Also find the distance between these two lines.
OR
Find the coordinates of the foot of the perpendicular Q drawn from P(3, 2,1) to the plane 2x – y +z +1=0. Also , find
the distance PQ and the image of the point P treating this plane as a mirror.
CBSE BOARD SUPPLEMENTARY QUESTION PAPER-2019
SECTION A (1 MARK EACH)
5 3 8
1. Find the cofactor of the element a23 of the determinants |2 0 1|.
1 2 3
2. Find the length of the perpendicular drawn from the points P(3,-4,5) on the z-axis.
OR
Find the vector equation of a plane, which is at a distance of 5 units from the origin and whose normal vector is 2i – j
+2k.
3. Form the differential equation representing the family of curves ax+by=0 where a and b are arbitrary constants.
𝑑𝑦
4. Find 𝑑𝑥 , if sin2x + cos2y =1.
SECTION B (2 MARKS EACH)
𝑑𝑥
5. Find ∫ 𝑐𝑜𝑠2 𝑥(1−𝑡𝑎𝑛𝑥)2
OR
1
∫0 𝑥(1 − 𝑥)𝑛 𝑑𝑥
6. Form the differential equation representing the family of curves y =bcos(x+a), where a and b are arbitrary constants.
7. Find a unit vector perpendicular to both 𝑎⃗ and 𝑏⃗⃗, where 𝑎⃗ = 4i – j +8k, 𝑏⃗⃗ = -j +k.
𝑂𝑅
If 𝑎⃗ = 2i+2j +3k, 𝑏⃗⃗ = -I +2j +k and 𝑐⃗ = 3i +j are such that 𝑎⃗ + λ𝑏⃗⃗ is perpendicular to 𝑐⃗ , then find the value
of λ.
8. Given that A and B are two independent events such that P(A)= 0.3 and P(B)= 0.5. Find the value of P(A/B).
OR
A bag contains 3 white and 2 red balls, another bag contains 4 white and 3 red balls. One ball is drawn at random
from each bag. Find the probability that the balls drawn are one white and one red.
1 1 1
9. The probability of A, B and C solving a problem independently are 2, and 4 respectively. If all the three try to solve
3
the problem independently, find the probability that the problem is solved.
1
10. if f:R → R, is given by f(x) = (3 − 𝑥 3 )3 , find fof(x).
1 2 0 0
11. For what value of x is [1 2 1] [2 0 1] [2] = 0.
1 0 2 𝑥
𝑑𝑥
12. Find : ∫ √5−4𝑥−𝑥 2 .
SECTION C (4 MARKS EACH)

𝑐𝑜𝑠𝑥
13. Find ∫ (1+𝑠𝑖𝑛𝑥)(2+𝑠𝑖𝑛𝑥) dx
𝜋 𝑥 𝑠𝑖𝑛𝑥
14. Evaluate: ∫0 dx
1+𝑐𝑜𝑠2 𝑥
15. Find the particular solution of the differential equation :
x dx – 𝑦 𝑒 𝑦 √1 + 𝑥 2 dy =0, given that y=1 when x =0.
OR
𝑦 𝑦
Solve the differential equation x cos (𝑥 ) = y cos (𝑥 ) + x.
16. Find the vector equation of the line passing through the point (2, 3, -1) and parallel to the planes
𝑟⃗ . (3i +4 j+2k) =5 and 𝑟⃗ . (3i -2j -2k) =4.
17. Let A = N x N be the set of all ordered pairs of natural numbers and R be the relations on the set A defined by (a,b) R
(c,d) iff ad =bc. Show that R is an equivalence relations.
OR
𝑥 2𝑥
Show that f: R –{2} → R-{1} defined by f(x) = 𝑥−2 is one one . Also, if g:R-{1} →R-{2} is defined as g(x)= 𝑥−1, find
gof(x).
18. If y=acos(logx)+bsin(logx) show that x2y2+xy1+y=0
√1+𝑥 +√1−𝑥 𝜋 1
19. Prove that tan-1[ ]= + cos-1x.
√1+𝑥 −√1−𝑥 4 2
𝑑2 𝑦
20. x=asec3𝜃 ; y=atan 𝜃 ,find 𝑑𝑥 2 .
3

𝑑𝑦
OR Find 𝑑𝑥 , if xy . yx = xx.
21. Find x such that the four points A(5,x,4) , B(4,4,0), C(5,4,-3) and D(7,7,-2) are coplanar.
22. Find the equation of the tangent and normal to the parabola y2 = 4ax at the points (at2 , 2at).
23. Using properties of determinants, prove the following:
𝛼 𝛽 𝛾
2 2
| 𝛼 𝛽 𝛾 2 | = (𝛼 − 𝛽)(𝛽 − 𝛾(𝛾 − 𝛼)(𝛼 + 𝛽 + 𝛾).
𝛽+𝛾 𝛼+𝛾 𝛼+𝛽

SECTION D (6 MARKS EACH)


𝑥−8 𝑦−1 𝑧−3
24. Find the coordinates of the points where the line = = intersects the plane 2x +2y+z =3. Also find the
4 1 8
angle between the line and the plane.
OR
𝑥−7 𝑦−5 𝑧−3 𝑥−1 𝑦+1 𝑧+1
A line with direction ratio <2, 2,1> intersects the lines = = and = = at the points P and Q
3 2 1 2 4 3
respectively. Find the length and equation of the intercept PQ.
25. A small firm manufactures chairs and tables. Market demand and available resources indicate that the combined
production of chairs and tables should not exceed 50 units per day. It takes 30 minutes to manufacture a chair and 1
hour to manufacture a table. A maximum of 40 man-hours per day are available.
The profit on each chair is Rs 40 and profit on each table is Rs 60. Determine how many each of chairs and tables
should be manufactured per day in order to maximize the profit. What is the maximum profit? Formulate LPP and
solve graphically.
26. Bag I contains 4 red and 2 black balls and bag II contains 3 red and 5 black balls. One ball is transferred from bag I to
bag II and then a ball is drawn from bag II. The ball so drawn is found to be red in colour. Find the probability that
the transferred ball is black.
0 1 2
27. If A = [1 2 3] , Find A-1 Use it to solve the system of equations:
3 1 1
x + 2z =5 , x + 2y +3z = 10 , 3x + y + z = 9
OR
3 −1 1
Using elementary row transformation, find the inverse of the matrix. A = [−15 6 −5] .
5 −2 2
28. A helicopter of the enemy is flying along the curve given by y = x2 +7. A soldier, placed at (3,7), wants to shoot
down the helicopter when it is nearest to him. Find the nearest distance.
29. Using integration, find the area lying above x-axis and included between the circle x2 + y2 =4x and inside of the
parabola y2 = 2x.
OR
Using integration, find the area of triangle ABC the co-ordinates of whose vertices are A(3,2),B(5,7) and C(7,5).
CBSE BOARD QUESTION PAPER-2018
SECTION A (1 MARK EACH)
1. If a*b denotes the larger ‘a’ and ‘b’ and if a o b = (a * b) + 3, then write the value of (5) o 10, where * and o are binary
operations.
2. Find the magnitude of each of the two vectors a and b, having the same magnitude such that the angle between them
is 60o and their scalar product is 9/2.
0 𝑎 −3
3. If matrix A = [2 0 −1] is skew symmetric, find the value of ‘a’ and ‘b’.
𝑏 1 0
4. Find the value of: tan‒1√3 ‒ cot‒1 (‒√3).
SECTION B (2 MARKS EACH)
5. The total cost C (x) associated with the production of x units of an item is given by
C (x) = 0.005 x3‒ 0.02 x2 +30 x + 5000. Find the marginal cost when 3 units are produced, where by marginal cost we
mean the instantaneous rate of change of total cost at any level of output.
6. Differentiate tan‒1 {(1 + cos x)/sinx}.
2 −3
7. Given A = [ ]compute A‒1 and show that 2 A‒1 = 9I ‒ A.
−4 7
8. Prove that 2 sin‒1 x = sin‒1(3x ‒ 4 x3).
9. A black and a red die are rolled together. Find the conditional probability of obtaining the sum 8, given that the red
die in number less than 4.
10. If sin ø is the angle between i ‒ 2 j + 3 k and 3 i ‒ 2 j + k, find sin ø.
11. Find the differential equation representing the family of curves y = a eb x + 5, where a and b are arbitrary constants.
𝑐𝑜𝑠2𝑥+2𝑠𝑖𝑛2 𝑥
12. Evaluate : ∫ 𝑑𝑥.
𝑐𝑜𝑠2 𝑥
SECTION C (4 MARKS EACH)
13. Two numbers are selected at random (Without replacement) from the first five positive integers. Let X denote the
larger of the two numbers obtained. Find the mean and variance of x.
14. An open tank with a square base and vertical sides is to be constructed from a metal sheet so as to hold a given
quantity of water. Show that the cost of material will be least when depth of the tank is half of its width. If the cost is
to be borne by nearby settled lower income families, for whom water will be provided, what kind of value is hidden
in this question?
15. Find the equations of the tangent and the normal, to the curve 16x2 + 9y2 =145 at the point (x1, y1), where x1=2 and
y1> 0.
OR
𝑥4
Find the intervals in which the function f(x) = - x3 -5x2 +24x +12 is (a)Strictly increasing (b)Strictly decreasing.
4
𝑑𝑦
16. If (x2 + y2)2 = xy , find 𝑑𝑥 .
OR
𝑑𝑦 𝜋
If x=a(2𝜃 -sin2𝜃) and y =a(1-cos2 𝜃), find 𝑑𝑥 when 𝜃 = 3 .
𝑑2 𝑦 𝑑𝑦
17. If y= sin(sinx), prove that 𝑑𝑥 2 + tanx +y cos2x =0.
𝑑𝑥
18. Find the particular solution of the differential equation ex tany dx +( 2- ex )sec2y dy=0 given that
𝜋
y = 3 when x=0.
OR
𝑑𝑦 𝜋
Find the particular solution of the differential equation 𝑑𝑥 + 2 y tanx =sinx, given that y=0 when x= = 3 .
SECTION D (6 MARKS EACH)
19. Find the shortest distance between the lines
𝑟⃗ = (4i –j ) + λ(i + 2j -3k) and 𝑟⃗ = i – j +2 k + 𝜇 (2i +4 j- 5k)
2𝑐𝑜𝑠𝑥
20. Find : ( dx.
1−𝑠𝑖𝑛𝑥)(1+𝑠𝑖𝑛2 𝑥)
21. Suppose a girl throws a die. If she gets 1 or 2, she tosses a coin three times and notes the number of tails. If she gets
3, 4, 5 or 6, she tosses a coin once and notes whether a head or tail is obtained. If she obtained exactly one ‘tail’.
What is the probability that she threw 3, 4, 5 or 6 with the die?
22. Let a= 4i+5j-k ,b=i-4j+5k and c=3i+j-k .Find a vector d which is perpendicular to both c and b and 𝑑 ⃗⃗⃗⃗ 𝑎⃗ =21.
1 1 1 + 3𝑥
23. Using properties of determinants, prove that |1 + 3𝑦 1 1 | = 9(3xyz +xy +yz +zx)
1 1 + 3𝑧 1
24. Using integration, find the area of the region in the first quadrant enclosed by the x-axis, the line y= x and the circle
x2 + y2 =32.
25. Let A={x 𝜖 Z: 0≤ x ≤ 12 }. Show that R= {(a, b): a,b 𝜖 A, |𝑎 − 𝑏| is divisible by 4} is an equivalence relation. Find
the set of all elements related to 1. Also write the equivalence class {2}.
OR
𝑥
Show that function f: R → R defined by f(x) = 𝑥 2 +1 , for every x 𝜖 R is neither one-one nor onto.Also, if g:R→R id
defined as g(x) =2x-1, find fog(x).
26. Find the distance of the point ( -1, -5, -10) from the point of intersection of the line
𝑟⃗ = (2i –j +2k ) + λ(3i + 4j +2k) and the plane 𝑟⃗.(i –j +k) =5.

27. A factory manufactures two types of screws A and B, each type requiring the use of two machines, an automatic and
a hand-operated. It takes 4 minutes on the automatic and 6 minutes on the hand-operated machines to manufacture a
packet of screws ‘A’ while it takes 6 minutes on the automatic and 3 minutes on the hand-operated machine to
manufacture packet of screws ‘B’. Each machine is available for at most 4 hours on any day. The manufacturer can
sell a packet of screws ‘A’ at a profit of 70 paise and screws ‘B’ at a profit of 1. Assuming that he can sell all the
screws he manufactures how many packets of each type should the factory owner produce in a day in order to
maximize his profit? Formulate the above LPP and solve it graphically and find the maximum profit.
𝜋
𝑠𝑖𝑛𝑥+𝑐𝑜𝑠 𝑥
28. Evaluate : ∫04 16+9 𝑠𝑖𝑛2𝑥 dx.
OR
3
Evaluate: ∫1 𝑥 2 +3x +𝑒 𝑥 dx as the limit of the sum.
2 −3 5
29. If A = [3 2 −4] , Find A-1 Use it to solve the system of equations:
1 1 −2
2x – 3y + 5z =11 , 3x + 2y -4z = - 5 , x + y -2 z= - 3.
OR
1 2 3
Using elementary row transformation, find the inverse of the matrix. A = [ 2 5 7 ].
−2 −4 −5
CBSE BOARD SUPPLEMENTARY QUESTION PAPER-2018
SECTION A (1 MARK EACH)
1. Find the value of tan-1√3 - sec-1(-2).
1 2 2
2. If A = ( 2 1 𝑥 ) is a matrix satisfying AA’ =9I. find x.
−2 2 −1
3. Find the value of [𝑖̂ 𝑗̂ 𝑘̂].
4. Find the identity element in the set Q+ of all positive rational numbers for the operation * defined by
3𝑎𝑏
a*b= for all a, b 𝜖 Q+.
2
SECTION B (2 MARKS EACH)
-1 -1 3
5. Prove that 3 cos x = cos (4 x - 3x).
2 3
6. If A = [ ] be such that A -1 = kA, then find tha value f k.
5 −2
𝑐𝑜𝑠𝑥−𝑠𝑖𝑛𝑥
7. Differentiate tan-1( cos 𝑥+𝑠𝑖𝑛𝑥 ) with respect to x.
8. The total revenue received from the sale of x units of a product is given by R(x) = 3x2 + 36x +5 in rupees. Find the
marginal revenue when x = 5, where by marginal revenue we mean the rate of change of total revenue with respect to
the number of items sold at any instant.
3−5 sin 𝑥
9. Find ∫ 𝑑𝑥.
𝑐𝑜𝑠2 𝑥
𝑑𝑦
10. Solve the D.E cos( 𝑑𝑥 ) = a, where a 𝜖 R.
11. If 𝑎⃗ + 𝑏⃗⃗ + 𝑐⃗ = 0 , if |𝑎⃗| = 3, |𝑏⃗⃗| = 6 and |𝑐⃗| = 9, then find the angle between 𝑎⃗ and 𝑏⃗⃗ .
5
12. Evaluate P(A U B), if 2 P(A) = P(B) = 13 and P(A/B) = 2/5.
SECTION C (4 MARKS EACH)
13. Using properties of determinants, prove that
5𝑎 −2𝑎 + 𝑏 −2𝑎 + 𝑐
|−2𝑏 + 𝑎 5𝑏 −2𝑏 + 𝑐 |= 12(a+b+c)(ab+bc+ca).
−2𝑐 + 𝑎 −2𝑐 + 𝑏 5𝑐

𝑑𝑦 𝑐𝑜𝑠2 (𝑎+𝑦) 𝑑𝑦
14. If sin y = x cos (a+y), then show that 𝑑𝑥 = . Also show that 𝑑𝑥 = cosa , where a = 0.
cos 𝑎
𝑑2 𝑦 𝜋
15. If x = a sec3𝜃 and y = a tan3𝜃 . find 𝑑𝑥 2 at 𝜃 = 3 .
OR
−1 𝑥 𝑑2 𝑦 𝑑𝑦
If y = 𝑒 𝑡𝑎𝑛 prove that (1 +x2) + (2 x-1)𝑑𝑥 .
𝑑𝑥 2
16. Find the angle of intersection of the curves x2 + y2 =4 and (x -2)2 + y2 = 4, at the point in the first quadrant.
OR
Find the intervals in which the function f(x) = - 2x3 – 9x2 -12x +1 is (i)Strictly increasing (ii)Strictly Decreasing.
17. A window is in the form of a rectangle surmounted by a semicircular opening. The total perimeter of the window is
10m. Find the dimensions of the window to admit maximum light through the whole opening.
4
18. Find ∫ (𝑥−2)(𝑥 2+4) dx.
19. Solve the differential equation (x2 - y2 )dx +2xydy =0.
OR
𝑑𝑦 1
Find the particular solution of the differential equation ( 1+ x2 ) 𝑑𝑥 + 2xy = 1+𝑥 2 given that y= 0 and x =1.
20. Find x such that the four points A(4, 4, 4), B(5, x, 8), C(5, 4, 1) and D(7, 7, 2) are coplanar.
21. Find the shortest distance between the lines :
𝑥−1 𝑦−2 𝑧−3 𝑥−2 𝑦−4 𝑧−5
= = and = = .
2 3 4 3 4 5
22. Two groups are competing for the position on the Board of directors of a corporation. The probabilities that the first
and the second groups will win are 0.6 and 0.4 respectively. Further, if the first group wins, the probability of
introducing a new product is 0.7 and the corresponding probability is 0.3 if the second group wins. Find the
probability that the new product introduced was by the second group.
23. From a lot of 20 bulbs which include 5 defective, a sample of 3 bulbs is drawn at random, one by one with
replacement. Find the probability distribution of the number of defective bulbs. Also find the mean of the
distribution.
SECTION D (6 MARKS EACH)
24. Show that the relation R on the set Z of all integers defined by (x, y) 𝜖 R ↔ x – y is divisible by 3 is an equivalence
relation.
OR
𝑎+𝑏 𝑖𝑓 𝑎 + 𝑏 < 6
Define a binary operation on the set {0,1,2,3,4,5} as a*b={ .
𝑎+𝑏−6 𝑎+𝑏 ≥6
Show that zero is the identity for this operation and each element a of the set is invertible with 6-a being the inverse
of a.
5 0 4 1 3 3
25. Given A = [2 3 2] and B -1 = [1 4 3] find (AB)-1.
1 2 1 1 3 4
OR
1 2 −2
Find the inverse of the matrix A = [−1 3 0 ] by using elementary row transformation.
0 −2 1

26. Using integration find the area of the region. { (x, y) : 0 ≤ 2y≤x2 , 0≤y≤ 𝑥, 0≤x≤ 3}
𝜋
𝑥𝑠𝑖𝑛𝑥𝑐𝑜𝑠𝑥
27. Evaluate: ∫02 𝑠𝑖𝑛4 𝑥+ 𝑐𝑜𝑠4𝑥 dx.
OR
3
Evaluate: ∫1 (3𝑥 2 + 2𝑥 + 1)𝑑𝑥 as the limit of the sum.

28. Find the vector equation of the line passing through (1, 2, 3) and parallel to each of the planes
𝑟⃗ . (i - j + 2 k) = 5 and 𝑟⃗ . (3i + j + k) = 6. Also find the point of intersection of the line thus obtained with the plane
𝑟⃗ . (2i + j + k) = 4 .

29. A company produces two types of goods, A and B, that require gold and silver. Each unit of type A requires 3 g of
silver and 1 g of gold while that of B requires 1 g of silver and 2 g of gold. The company can use atmost 9 g of silver
and 8 g of gold. If each unit of type A brings a profit of 40 and that of type B 50, find the number of units of each
type that the company should produce to maximize the profit. Formulate and solve graphically the LPP and find
the maximum profit.
CBSE BOARD QUESTION PAPER-2017
SECTION A (1 MARK EACH)
1. If A is a 3 X 3 invertible matrix, then what will be the value of k if det(A-1) =(det A)k.
𝑘𝑥
𝑖𝑓 𝑥 < 0
2. Determine the value of the constant ‘k’ so that the function f(x) = { |𝑥| is continuous at x=0.
3 𝑖𝑓 𝑥 ≥ 0
3
3. Evaluate : ∫2 3𝑥 dx.
4. If a line makes angles 900 and 600 respectively with the positive directions of x and y axes, find the angle which it
makes with the positive direction of z-axis.
SECTION B (2 MARKS EACH)
5. Show that all the diagonal elements of a skew symmetric matrix are zero.
𝜋
6. Find dy/dx at x = 1, y = 4 if sin2y + cos xy =K.
7. The volume of a sphere is increasing at the rate of 3 cm3 per second. Find the rate of increase of its surface area,
when the radius is 2 cm.
8. Show that the function f(x) = 4 x3 -18 x2 +27x -7 is always increasing on R.
9. Find the vector equation of the line passing through the point A(1, 2, -1) are parallel to the line
5x – 25 =14 -7y = 35z.
10. Prove that if E and F are independent events, then the events E and F’ are also independent.
11. A small firm manufactures necklaces and bracelets. The total number of necklaces and bracelets that it can handle per
day is at most 24. It takes one hour to make a bracelet and half an hour to make a necklace. The maximum number of
hours available per day is 16. If the profit on a necklace is Rs 100 and that on a bracelet is Rs 300. Formulate on
L.P.P for finding how many of each should be produced daily to maximize the profit? It is being given that at least
one of each must be produced.
𝑑𝑥
12. Find ∫ 𝑥 2+ 4𝑥+8.
SECTION C (4 MARKS EACH)
𝜋 1 -1𝑎 𝜋 1 𝑎 2𝑏
13. Prove that tan { 4 + 2 cos } + tan { 4 - 2 cos-1𝑏} = .
𝑏 𝑎
𝑥 𝑥 + 𝑦 𝑥 + 2𝑦
14. Using properties of determinants, prove that |𝑥 + 2𝑦 𝑥 𝑥 + 𝑦 | = 9y2 (x+y).
𝑥 + 𝑦 𝑥 + 2𝑦 𝑥
OR
2 −1 5 2 2 5
Let A = [ ] , B=[ ] ,C= [ ]. Find a matrix D such that CD-AB=0.
3 4 7 4 3 8
15. Differentiate the function (sinx)x + sin-1√𝑥 with respect to x.
OR
𝑑𝑦 𝑦
If xm.yn =(x+y)m+n , prove that 𝑑𝑥 = 𝑥
2𝑥
16. Find ∫ (𝑥 2+1)(𝑥 2+2)2 dx
3
17. Evaluate : ∫02|𝑥𝑠𝑖𝑛𝜋𝑥| dx.
18. Prove that x2 – y2 = c(x2+ y2)2 is the general solution of the D.E (x3 -3xy2)dx = (y3 -3x2y)dy, where C is a parameter.
19. Let 𝑎⃗ = i + j + k, 𝑏⃗⃗ = i and 𝑐⃗ =c1i +c2j +c3k, then
(a) Let c1 = 1 and c2=2, find c3 which makes 𝑎⃗, 𝑏⃗⃗ and 𝑐⃗ coplanar.
(b) If c2 = -1 and c3= 1, show that no value of c1 can make 𝑎⃗ , 𝑏⃗⃗ and 𝑐⃗ coplanar.
20. If 𝑎⃗ , 𝑏⃗⃗, 𝑐⃗ are mutually perpendicular vectors of equal magnitudes, show that the vectors 𝑎⃗ + 𝑏⃗⃗ + 𝑐⃗ is equally
inclined to 𝑎⃗ , 𝑏⃗⃗ and 𝑐⃗. Also find the angle which 𝑎⃗ + 𝑏⃗⃗ + 𝑐⃗ makes with 𝑎⃗ or 𝑏⃗⃗ or 𝑐⃗.
21. The random variable X can take only the values 0, 1, 2, 3. Given that P(X=0)=P(X=1)=p and P(X)=2=P(X=3) such
that ∑ 𝑝𝑖 𝑥𝑖2 = 2 ∑ 𝑝𝑖 𝑥𝑖 , find the value of p.
22. Often it is taken that a truthful person commands, more respect in the society. A man is known to speak the truth 4
out of 5 times. He throws a die and reports that it is a six. Find the probability that it is actually a six.
23. Solve the following L.P.P graphically
Minimise Z = 5x +10 y
Subject to constraints : x + 2y ≤ 120, x + y ≥ 60, x-2y ≥ 0 and x,y ≥ 0.
SECTION D (6 MARKS EACH)
1 −1 2 −2 0 1
24. Use product of [0 2 −3] [ 9 2 −3] to solve the system of equations
3 −2 4 6 1 −2
X+3z=9, -x +2y-2z=4, 2x-3y+4z = -3.
√𝑦+6−1
25. Consider f:R+→[-5,∞) given by f(x)=9x2+6x-5,Show that f is invertible with f-1(y)= . Hence find
3
4
(i)f-1(10) (ii) y if f-1(y) = 3. Where R+ is the set of all non-negative real numbers.
OR
Discuss the commutatively and associatively of binary operation ‘*’ defined on A = Q – {1} by the rule
a* b = a- b +ab for all a, b 𝜖 A. Also find theidentity element of * in A and hence find the invertible
elements of A.
26. If the sum of the length of the hypotenuse and a side of a right angled triangle is given,Show that the area of the
𝜋
triangle is maximum when the angle between them is 3 .
27. Using integration, find the area of region bounded by the triangle whose vertices are (-2,1), (0,4) and (2,3).
OR
Find the area bounded by the circle x2 + y2 =16 and the line √3 y =x in the first quadrant, using integration.

𝑑𝑦 𝜋
28. Solve the D.E x 𝑑𝑥 + y = x cosx + sin x , given that y =1 when x = 2 .
29.
𝑥−8
30. Find the vector and Cartesian equations of a line passing through (1, 2, - 4) and perpendicular to the lines =
3
𝑦+19 𝑧−10 𝑥−15 𝑦−29 𝑧−5
= and = = .
−16 7 3 8 −5

OR
Find the equation of the plane through the line of intersection of 𝑟⃗ .(2i -3 j +4k) =1 and 𝑟⃗ .(i – j) 4=0 and
perpendicular to the plane 𝑟⃗.(2i – j + k) + 8 = 0. Hence find whether the plane thus obtained contains the line
x-1 = 2y -4 = 3z -12.
CBSE BOARD SUPPLEMENTARY QUESTION PAPER-2017
SECTION A (1 MARK EACH)
1. If the following function f(x) is continuous at x =0, then write the value of k.
3𝑥
𝑠𝑖𝑛
2
f(x) = { , 𝑥=0.
𝑥
𝑘 ,𝑥 = 0
2. If A is an invertible matrix of order 2 and det(A) = 4, then write the value of det (A-1).
3. If ( 𝑎⃗ 𝑋 𝑏⃗⃗ )2 + ( 𝑎⃗ . 𝑏⃗⃗ )2 = 225 and |𝑎⃗| =5, then write the value of |𝑏⃗⃗|.
3𝑥
4. Find : ∫ 3𝑥−1 𝑑𝑥
SECTION B (2 MARKS EACH)
5. Find the values of x and y from the following matrix equation :
𝑥 5 3 −4 7 6
2[ ]+ [ ]= [ ].
7 𝑦−3 1 2 15 14
𝜋
6. If f(x) = sin2x – cos2x , find f’( 6 )
7. A particle moves along the curve 6y=x3 + 2. Find the points on the curve at which y-coordinate is changing 2 times as
fast as x-coordinate.
𝑥
8. Find : ∫ √32−𝑥 2 dx.
𝑑𝑦
9. Find the general solution of the D.E log ( 𝑑𝑥 )= 3x +4y.
𝑑𝑦 1+𝑦
10. Find the integrating factor of the D.E 𝑑𝑥 + y = .
𝑥
11. If the vectors 𝑎⃗ = i +3j +k, 𝑏⃗⃗ = 2i – j –k and 𝑐⃗ = ai + 7 j +3 k are coplanar, then find the value of a.
12. An aero plane can carry a maximum of 250 passengers. A profit of Rs 1500 is made on each executive class ticket
and a profit f Rs 1000 is made on each economy class ticket. The airlines reserves at least 25 seats for executive class.
However, at least 3 times as many passengers prefer to travel by economy class than by executive class. Frame the
linear programming problem to determine how many tickets of each type must be sold in order to maximize the profit
for the airline.
SECTION C (4 MARKS EACH)
13. A school wants to award its students for regularity and handwork with a total cash award of Rs 6000.If three times
the award money for hard work added to that given for regularity amounts to Rs 11000, represent the above situation
algebraically and find the award money for each value, using matrix method. Suggest two more values, which the
school must include for award.
1−𝑥 1
14. Find the real solutions of the equation tan-1(1+𝑥 ) = 2 tan-1x(x>0)
𝑑𝑦
15. If y =(cos x)x + sin-1√3𝑥 , find 𝑑𝑥 .
OR
𝑑2 𝑦 𝑑𝑦
If y=(sec-1)2, then show that x2(x2 – 1) 𝑑𝑥 2 +(2x3 – x)𝑑𝑥 =2.
16. Find the intervals in which the function given by f(x) = 2x3 – 3x2 -36x+7 is
(a)Strictly increasing (b)Strictly decreasing
𝜋 𝑥𝑠𝑖𝑛𝑥
17. Evaluate : ∫0 dx
1 +𝑐𝑜𝑠2 𝑥
18. Find : ∫(𝑥 − 3) √3 − 2𝑥 − 𝑥 2 dx
OR
𝑥 2 +𝑥+1
Find: ∫ (𝑥+1)2(𝑥+2) dx

𝑑𝑦
19. Find the general solution of the D.E 2xy 𝑑𝑥 = x2 + y2.
20. If 𝑎⃗ , 𝑏⃗⃗ and 𝑐⃗ are mutually perpendiculars vectors of equal magnitudes, find the angles which the vectors 2𝑎⃗ 𝑏⃗⃗ + 𝑐⃗
makes with the vectors If 𝑎⃗ , 𝑏⃗⃗ and 𝑐⃗.
21. Find the co-ordinates of perpendicular drawn from a point A(1, 8, 4) to the line joining the points B(0, -1,3) and C(2,
-3, -1).
22. Solve the following linear programming problem graphically:
Minimize : Z= 3x+9y
When: x+3y ≤ 60 , x+y ≥10 , x ≤ y and x, y≥ 0.
23. Bag A contains 3 red and 2 black balls, while bag B contains 2 red and 3 black balls. A ball drawn at random from
bag A is transferred to bag B and then one ball is drawn at random from bag B. if this ball was found to be red ball,
find the probability that the ball drawn from bag A was red.
OR
If A and B are two independent events, then prove that the probability of occurrence of at least one of A and B is
given by 1 –P(A’).P(B’)
SECTION D (6 MARKS EACH)
𝑎 𝑏 𝑐
24. If a+b+c ≠ 0 and |𝑏 𝑐 𝑎| =0 then using properties of determinants, prove that a = b = c.
𝑐 𝑎 𝑏
25. Given a non-empty set X, consider the binary operation *:P(X) X P(X) → P(X) given by A*B= A∩B, for every A, B
∈ P(X), where P(X) is the power set of X. Show that * is commutative and associative and X is the identity element
for this operation and X is the only invertible element in P(X) with respect to the operation *.
OR
4 4𝑥
Let f:R – {- 3} →R be a function defined as f(x)= 3𝑥+4 .Show that f is one-one function. Also check whether f is onto
4
function or not. Hence find f-1 in (Range off) → R - {- 3}.
26. Show that the right circular cone of least curved surface and given volume has an altitude equal to √2 times the
radius of the base.
OR
Find the equation of tangent to the curve x = acos𝜃 + a𝜃 𝑠𝑖𝑛𝜃 , y = asin𝜃 –a𝜃 𝑐𝑜𝑠𝜃 at any point 𝜃 of the curve. Also
show that at any point 𝜃 of the curve the normal is at a constant distance from origin.
𝑥−5 𝑦−4 𝑧−8
27. Find the distance between the line = = and the plane determined by the points A(2, -2, 1),
3 3 1
B(4, 1, 3) and C(-2, -2, 5).
28. A coin is biased so that the head is 4 times as likely to occur as tail. If the coin is tossed thrice, find the probability
distribution of number of tails. Hence find the mean and variance of the distribution.

29. Using method of integration find the area of the triangle ABC, co-ordinates of whose vertices A(1,-2), B(3, 5) and
C(5,2).
OR
Evaluate the following definite integrals as limit of sums.
4
∫0 3𝑥 2 + 2𝑥 + 1 dx.
CBSE BOARD QUESTION PAPER-2016
SECTION A (1 MARK EACH)
𝑥 + 3 −2
1. If x 𝜖 N and [ ] = 8, then find the value of x.
−3𝑥 2𝑥
2. Use elementary column operation C2 → C2 + 2C1 in the following matrix equation:
2 1 3 1 1 0
[ ]=[ ][ ]
2 0 2 0 −1 1
3. Write the number of all possible matrices of order 2 X 2 with each entry 1, 2 or 3.
4. Write the position vector of the point which divides the join of points with position vectors 3𝑎⃗ -2𝑏⃗⃗ and 2𝑎⃗ +3 𝑏⃗⃗ in the
ratio 2:1.
5. Write the number of vectors of unit length perpendicular to both the vectors 𝑎⃗ =2i +j +2 k and
𝑏⃗⃗ = j + k.
6. Find the vector equation of the plane with intercepts 3, -4 and 2 on x, y and z-axis respectively
SECTION B (4 MARKS EACH)
7. Find the co-ordinates of the point where the line through the points A(3, 4, 1) and B(5, 1, 6) crosses the XZ plane.
Also find the angle which this line makes with the XZ plane.
8. The two adjacent sides of a parallelogram are 2i -4 j -5k and 2i + 2j +3 k. Find the two unit vectors parallel to its
diagonals. Using the diagonal vectors, Find the area of the parallelogram.
9. In a game, a man wins Rs 5 for getting a number greater than 4 and loses Rs 1 otherwise, when a fair die is thrown.
The man decided to throw a die thrice but to quit as and when he gets a number greater than 4. Find the expected
value of the amount he wins/loses.
OR
A bag contains 4 balls. Two balls are drawn at random (without replacement) and are found to be white.What is the
probability that all balls in the bag are white?
10. Differentiate xsinx + (sinx)cosx with respect to x.
OR
𝑑2 𝑦 𝑑𝑦
If y =2cos(logx) + 3 sin(logx), prove that x2 𝑑𝑥 2 + x + y = 0.
𝑑𝑥
𝑑𝑦 𝜋
11. If x = asin2t(1 + cos2t) and y = bcos2t(1 – cos2t), find 𝑑𝑥 at t = 4 .
12. The equation of tangent at (2,3) on the curve y2 = ax3 + b is y= 4x – 5. Find the values of a and b.
𝑥2
13. Find : ∫ 𝑥 4 +𝑥 2 −2cdx.
𝜋
𝑠𝑖𝑛2 𝑥
14. Evaluate: ∫02 𝑠𝑖𝑛𝑥+𝑐𝑜𝑠𝑥 dx
OR
3
Evaluate: ∫02|𝑥𝑐𝑜𝑠𝜋𝑥| dx.
15. Find : ∫(3𝑥 + 1) √4 − 3𝑥 − 2𝑥 2 dx.
𝑑𝑦 𝑑𝑦
16. Solve the Differential equation : y + x 𝑑𝑥 = x – y 𝑑𝑥 .
17. Form the differential equation of the family of circles in the second quadrant and touching the coordinate axes.
18. Solve: the equation for x : sin-1x + sin-1(1 – x) = cos-1 x.
OR
𝑥 𝑦 𝑥2 𝑥𝑦 𝑦2
If cos-1𝑎 + cos-1𝑏 = a, Prove that 𝑎2
- 2 𝑎𝑏 cosa + 𝑏2 =sin2a.
19. A trust invested some money in two type of bond. The first bond pays 10% interest and second bond pays 12%
interest. The trust received Rs 2800 as interest. However, if trust had interchanged money in bonds, they would have
got Rs 100 less as interest. Using matrix method, find the amount invested by the trust. Interest received on this
amount will be given to Helpage India as donation. Which value is reflected in this question?
SECTION C (6 MARKS EACH)
20. There are two types of fertilizers ‘A’ and ‘B’. ‘A’ consists of 12 % nitrogen and 5 % phosphoric acid whereas ‘B’
consists of 4% nitrogen and 5% phosphoric acid. After testing the soil conditions, farmer finds that he needs at least
12kg of nitrogen and 12 kg of phosphoric acid for his crops. If ‘A’ costs Rs 10 per Kg and ‘B’ cost Rs 8 per kg, then
graphically determine how much of each type of fertilizer should be used so that nutrient requirements are met at a
minimum cost.
21. Five bad oranges are accidently mixed with 20 good ones. If four oranges are drawn one by one successively with
replacement, then find the probability distribution of number of bad oranges drawn. Hence find the mean and
variance of the distribution.
22. Find the position vector of the foot of perpendicular and perpendicular distance from the point P with position vector
2i +3 j +4 k to the plane 𝑟⃗ . (2i +j +3 k) -26 =0. Also find the image of P in the plane.
23. Show that the binary operation * on A = R – {- 1} defined as a * b =a+ b + ab for all a,b 𝜖 A is commutative and
associative of A. Also find the identity element of * in A and prove that every element of A is invertible.
24. Prove that the least perimeter of an isosceles triangle in which a circle of radius r can be inscribed is 6 √3 r.
OR
If the sum of lengths of hypotenuse and a side of a right angled triangle is given, show that area of triangle is
𝜋
maximum, when the angle between them is 3 .
25. Prove that the curves y2 = 4x and x2 = 4y divide the area of square bounded by x= 0, x = 4, y = 4 and y = 0 into three
parts.
26. Using properties of determinants, show that Triangle ABC is isosceles if:
1 1 1
| 1 + cos 𝐴 1 + 𝑐𝑜𝑠𝐵 1 + cos 𝐶 | = 0.
𝑐𝑜𝑠 2 𝐴 + 𝑐𝑜𝑠𝐴 𝑐𝑜𝑠 2 𝐵 + 𝑐𝑜𝑠𝐵 𝑐𝑜𝑠 2 𝐶 + 𝑐𝑜𝑠𝐶

OR
A shopkeeper has 3 varieties of pens ‘A’, ‘B’ and ‘C’. Meenu purchased 1 pen of each of each variety for a total of
Rs 21. Jeevan purchased 4 pens of ‘A’ variety, 3 pens of ‘B’ variety and 2 pens of ‘C’ variety for Rs 60. While
Shikha purchased 6 pens of ‘A’ variety, 2pens of ‘B’ variety and 3 pens of ‘C’ variety for Rs. 70.Using Matrix
method, find cost of each variety of pen.

****************************************************************************************

You might also like